AAFP questions.

¡Supera tus tareas y exámenes ahora con Quizwiz!

contraceptives with what type of ingredient should be avoided with spirinolactone because of risk of hyperkalemia?

oral drospirenone/ ethinyl estradiol.

People <30 years of age who excerete <2 g protein/ day and normal creatine clearance= test what?

orthostatic proteinuria. --increased protein excretion in upright position but normal excretion when supine. --dxed with split urine collection (daytime increased and night is normal). --> this is benign so no further evaluation needed.

pain in the anterior pelvic area and tenderness over the symphysis pubis in distance runners is what?

osteitis pubis.

Which one of the following is more common in non-Hispanic whites than in the Hispanic population? (check one) A. Obesity B. Osteoporosis C. Diabetes mellitus D. Hypertension E. Neurocysticercosis

osteoporosis.

middle age woman 30-50 with progressive conductive hearing loss most likely diagnosis?

otosclerosis. --if lesions iminge on stapes footplate a sensorineural loss may occur. A. Noise-induced hearing loss= tinnitis. B. Meniere's disease= fluctuating hearing loss. D. Acoustic neuroma= tinnitis and gradual hearing impairment. E. Perilymphatic fistula= sudden unilateral hearing loss with tinnitis and vertigo. all of these are sensorineural causes for hearing loss.

Patients with mild symptoms of pneumonia community with low level of macrolide resistance in community tx?

outpatient azithromycin. if high level resistance, comorbidities in patient (DM or COPD) or if hx of immunosuppresing drug duse= tx outpatient with levofloxacin. Patients with more severe symptoms, such as an elevated pulse rate or respiratory rate, should be treated on an inpatient basis with ceftriaxone or azithromycin. Patients who have more severe symptoms along with bronchiectasis should be treated with piperacillin/tazobactam plus levofloxacin.

Dribbling and increased post-void residual volume (>100 mL) are signs of

overflow incontinence.

signs and symptoms of lateral epicondylitis (tennis elbow) is due to? tx?

overuse tendinopathy of the common extensor tendon origin of the lateral elbow. modify work routine.

Tx for hemorrhagic shock?

packed RBCs.

most common presenting symptom of hodgkin lymphoma is?

painless lymphadenopathy. then you can have unexplained fever, night sweats and weight loss= b symptoms.

Supraclavicular lymph node + painless jaundice + palpable non tender gall bladder=

pancreatic cancer.

A. Bupropion (Wellbutrin) B. Paroxetine (Paxil) C. Fluoxetine (Prozac) D. Sertraline (Zoloft) highest rate of sexual dysfunction? lowest?

paroxetine highest. buproprion lowest.

Tamoxifen se? Raloxefine se? Tamoxifen more beneficial for the use of breast cancer.

partial agonist in endometrium, which the risk of endometrial cancer; "hot ashes." no in endometrial carcinoma (so you can relax!), because it is an estrogen receptor antagonist in endometrial tissue. both increase chance of thromboembolic event.

Pseudoaddiction?

patient behaviors that may occur when pain is undertreated. Pseudoaddiction can be distinguished from true addiction because the behaviors will resolve when the pain is effectively treated.

A 45-year-old female presents to your office with a 1-month history of pain and swelling posterior to the medial malleolus. She does not recall any injury, but reports that the pain is worse with weight bearing and with inversion of the foot. Plantar flexion against resistance elicits pain, and the patient is unable to perform a single-leg heel raise. Which one of the following is true regarding this problem?

patient has tendinopathy of posterior tibial tendon. fx of this tendon is to plantar flex foot, invert and stabalize the medial longitudinal arch. pain and swelling of ankle is normally noted. 2-3 weeks of immobilization in a cast boot.

What is the thessaly test?

patient stands flat footed on one leg. hold his/ her hands. flex the affected knee to 20 degrees. ask patient to twist body side to side thrice. pain at medical or lateral joint lines, licking= meniscal tear.

Hookworm larvae third stage you see?

penetrate hte skin and migrate through the dermis and create serpiginous and erythematous tracks characteristic of larva migrans. larvae do not develop in human so self limited infection that resolves in weeks-months.

The Infectious Diseases Society of America recommends which one of the following as the drug of choice for group A streptococcal pharyngitis?

penicillin because of it's proven efficacy, safety, narrow spectrum and low cost. Penicillin-resistant group A Streptococcus has never been documented

what are some natural remedies used for IBS?

peppermint, psychological tx. exercise, and probiotics.

PPV means?

percentage of patients with a positive test for a disease that actually have the disease. --Proportion of patients with negative test who do not have the disease is NPV. --percentage of patients with a disorder who have a positive test for the disorder is sensitivity. --percentage of patients without a disorder who have a negative test for the disorder is specificity.

supine chest pain can be? dyspnea with this can increase suspicion for?

pericarditis= check with EKG. pneumonia, PE, pneumothrorax, and MI.

terbutaline and ritodrine have potential to cause respiratory distress in form of?

pulmonary edema.

echinacea is used for prevention and tx of?

purple coneflower= URI.

PMS reduce symptoms with what natural remedy?

pyridoxine and chasteberry have been shown to reduce the symptoms.

The ophthalmopathy of Grave's disease may initially flare and worsen when treated with

radioactive iodine.

Dx of hereditary hemochromatosis requires?

random measurement of serum ferritin and calculation of transferritin saturation (serum iron/ TIBC). If wacky then check HFE.

All children younger than 13 years should ride in the car where?

rear seats of vehicles for optimal protection

what does FEV1 and FEV1/FVC look like for restrictive lung disease?

reduced and increased.

airflow obstruction is asthma. What is the FEV1 and FEV1/FVC ratio?

reduced/ decreased.

ab tx for bordetella pertussis does what?

reduces the risk of transmission to others. does not reduce duration or severity. azithromycin or clarithromycin. if less than 1 month old, azithromycin. do not use erythromycin in young infants= pyloric stenosis. tmp/smx good for people that cannot take macrolides but not if <2 months.

After several unsuccessful attempts to remove an object deep in the ear canal of an uncooperative child, it is best to

refer the patient to an otolaryngologist for removal under anesthesia. A loop curette cannot be safely placed behind a foreign body that is close to the tympanic membrane.

why was the accountable care organization introduced in 2009?

to encourage doctors, hospitals and other health care providers to work together to deliver high quality care and spend health savings more wisely.

Postherpetic neuralgia tx?

topical agents such as lidocaine patches and capsaicin cream/patch. so have gabapentin, pregabalin and amitryptyline.

allergic rhinitis top medication/ most efficacious?

topical intranasal glucocorticoids. cromolyn sodium is effective if started before the season of peak symptoms.

Nonbullous impetigo due to S. aureus or strep pyogenes tx?

topical mupirocin (bactroban).

Which one of the following is the preferred treatment for scabies?

topical permethrin.

open and closed comedones without evidence of surrounding inflammation on his face and upper back= initial tx?

topical retinoids. --no papules, pustules and nodules. if it had been inflammatory then you would have considered antibiotics.

best initial dx test for male hypogonadism is measurement of:

total testosterone in serum in a morning sample. 1. if low, confirm with repeat. sex binding hormone can be messed with if liver disease, obesity and diabetes.

A 76-year-old male with metastatic cancer, diabetes mellitus, and stage IV chronic renal disease develops confusion and myoclonus. His current medications include enalapril (Vasotec), 10 mg/day; glipizide (Glucotrol), 10 mg/day; and morphine sulfate, 30 mg every 4 hours for pain. The morphine was started 4 weeks ago and the dosage was gradually increased until the pain was controlled. Which one of the following is the most likely cause of his symptoms?

toxic metabolites of morphine. avoid morphine in someone with renal insufficiency because toxic metabolite cannot be eliminate by the kidney.

A 3-year-old male is carried into the office by his mother. Yesterday evening he began complaining of pain around his right hip. Today he has a temperature of 37.6°C (99.7°F), cries when bearing weight on his right leg, and will not allow the leg to be moved in any direction. A radiograph of the hip is normal. Dx? Which one of the following would be most appropriate at this time?

transient synovitis or septic arthritis of the hip. so do a CBC and ESR.

Kid OD on diltiazem tx?

transport to hospital and ICU observe. no charcoal or gastric lavage.

what do you need to evaluate for cardiac issues in kawasaki?

transthoracic echocardiography. coronary artery aneurysm.

best non-invasive test of choice for endometriosis? definitive test?

transvaginal US. laproscopy with histology.

a patient with painless postmenopausal bleeding what is the next diagnostic test?

transvaginal US= <3-4mm r/o endometrial carcinoma. If unavailable do a endometrial biopsy.

First-line treatment for diabetic peripheral neuropathy, according to the American Diabetes Association, is ...

tricyclic antidepressants.

important to monitor what for vancomycin?

trough serum levels= should be >10 to prevent bacterial resistance.

what causes waldenstrom's macroglobulinemia?

uncontrolled proliferation of lymphocytes and plasma cells= produce IgM proteins with kappa light chains. Weakness, fatigue, weight loss, bleeding, and recurrent infections are common presenting symptoms. unlike MM no lyptic bone lesions and bone marrow biopsy reveals mostly lymphocytes.

major orthopedic surgery, the American College of Chest Physicians recommends outpatient thromboprophylaxis for a duration of

up to 35 days.

If kid has delayed vocabulary what is another symptoms that could also indicate autism?

upset with normal noises; Delayed social development is typically the first sign of autism. Language delay can be another finding, but it is less specific. Of the behaviors listed, only abnormal sensitivity to sound is consistent with autism.

A 36-year-old male laborer presents to an urgent care center 5 hours after falling off a ladder. He was 7-8 feet off the ground, and he fell directly on his anterolateral leg as he landed. Weight bearing is painful. Foot pulses are normal, as is a sensorineural examination of the foot and leg. The anterolateral lower leg is quite tender but only slightly swollen, and there is exquisite pain in that area with passive plantar flexion of the great toe. Radiographs of the lower leg and ankle are negative. In addition to ice, elevation, and analgesia, which one of the following would be most appropriate?

urgent orthopedic referral for possible fasciotomy= acute compartment syndomre. The fasciotomy happens if there is muscle necrosis (>40mm Hg). if all five P's present then outcome really bad.

intrahepatic cholestasis of pregnancy tx?

ursodiol= controlling pruritis and decreasing liver function helping. safe for mom and fetus.

recommendation for melasma?

use of high potency broad spectrum sunscreen.

what years do you have to be born in for hepatitis C screening?

'45-'65.

what are the four things you need for decision making capacity?

(1) have the ability to understand the relevant information about proposed diagnostic tests or treatment (2) appreciate their situation (including their values and current medical situation) (3) use reason to make a decision (4) communicate their choice.

for who is a moderate intensity statin recommended? four groups.

(1) individuals with clinical atherosclerotic cardiovascular disease (ASCVD) (2) those with primary elevations of LDL-C >190 mg/dL (3) patients age 40-75 with diabetes mellitus, an LDL-C level of 70-189 mg/dL, and no clinical ASCVD (4) patients age 40-75 without clinical ASCVD or diabetes, an LDL-C level of 70-189 mg/dL, and an estimated 10-year ASCVD risk >7.5%. For patients age 40-75 with diabetes, an LDL-C level of 70-189 mg/dL, and no clinical ASCVD, a moderate-intensity statin is recommended.

Severe sepsis criterisa?

+ chest radiograph. organ hypoperfusion (mental status changes), septic shock (mean arterial pressure <60 mm Hg).

do you need antibiotic prophylaxis for MR for dental cleanings?

No.

recommendation for aaa?

65-75 yo who has ever smoked.

What is the only hyperthyroidism that causes high radioactive iodine uptake on thyroid scan?

graves.

What are he current guidelines for screening for colonoscopy with UC?

. Current guidelines recommend beginning screening colonoscopy 10 years after the initial diagnosis and continuing every 2-5 years, with the interval based on the findings. medication should be continued indefinitely unless the disease resurfaces.

when do you see a low DLCO?

. Reduced values are seen with severe interstitial fibrosis, or when the capillary surface has been compromised by vascular obstruction (pulmonary embolism) or is destroyed by emphysema. Chronic pulmonary embolism causes a low diffusing capacity of the lung for carbon monoxide (DLCO) with normal pulmonary function tests. comparatively, COPD would have abnormal PFT and low DLCO.

within 1 hour of ingestion of a potentially toxic amount of medication what can you do as tx?

1 dose of activated charcoal. if the medicine was iron/ lithium then use: Gastric lavage, cathartics, or whole bowel irrigation. ipecac is bad.

tx for chlamydia? during pregnancy?

1 gram dose of oral azithromycin even during pregnancy. --partners for past 60 days. --avoid sexual intercourse for 7 days after tx starts.

Which one of the following is the recommended duration of dual antiplatelet therapy after placement of a drug-eluting coronary artery stent?

1 year.

Ankle brachial index is normal w/ what numbers? lower means what? higher means what?

1-1.4. boarderline from 0.91-0.99 and abnormal if <0.9. lower the # the more severe the PAD. Values greater than 1.4= incompressible vessels and unreliable (long standing DM, or older). --use toe-brachial index for people with incompressible arteries of the more proximal lower extremity.

Three major treatments for nonspecific or idiopathic back pain?

1. Analgesics= acetaminopen, nsaids, tramadol. 2. multidisciplinary rehabilititation. 3. Acupuncture. Other treatments likely to be beneficial include herbal medications, tricyclics, antidepressants, exercise therapy, behavior therapy, massage, spinal therapy, opioids, and short-term muscle relaxants

A 43-year-old female complains of easy bruising. She is otherwise asymptomatic. A CBC reveals a platelet count of 23,000/mm3 (N 150,000-450,000). A peripheral smear reveals giant platelets. A workup is negative for autoimmune causes, including Graves disease, HIV, Epstein-Barr virus, cytomegalovirus, varicella zoster, hepatitis C, and Helicobacter pylori. She is on no prescription or over-the-counter medications and denies alcohol or drug use. Which one of the following would be the most appropriate initial management?

1. Corticosteroids, IVIG, Rituximab The patient has immune thrombocytopenic purpura. -tx restricted to severe cases like Platelets <50,000. 2nd line: TPO-R agonists and splenectomy.

when is TB induration 5+ mm considered +?

1. HIV-positive patients 2. transplant patients 3. household contacts of a tuberculosis patient.

Immunosuppression is defined as:

1. Treatment with glucocorticoids (treatment with the equivalent of 20 mg/day of prednisone for 2 weeks or more, and discontinuation within the previous 3 months) 2. Ongoing treatment with effective doses of 6-MP/azathioprine or discontinuation within the previous 3 months 3. Treatment with methotrexate or discontinuation within the previous 3 months 4. Treatment with infliximab or discontinuation within the previous 3 months 5. Significant protein-calorie malnutrition

unstable angina patients at high risk include those with what risk factors?

1. angina at rest with dynamic ST elevation >1mm. 2. angina with hypotension. 3. angina with new or worsening MR. 4. Angina with S3 or new or worsening crackles. 5. Prolonged >20 minutes of anginal pain at rest. 6. Pulmonary edema most likely related to ischemia.

For a patient with syncope, you get a thorough history and physical and then what do you do if: 1. prodrome of 5 seconds or less= 2. longer prodrome, nausea, vomiting? 3. patients that pass out after standing? Dx?

1. cardiac arrhythmia. 2. Vasovagal syncope. 3. Orthostatic hypotension. Most common test= EKG. - if normal, dysrhythmia not cause of syncope.

order of symptoms for B19 parvovirus?

1. facial flushing= slapped cheek. 2. exanthem concurrently to the trunk and proximal extremities= diffuse macular erythematous rash. 3. central clearing of the rash= lacy, reticulated apperance. --on extensor surfaces and spares palms and soles. -resolves

If thyroid nodule is >1 cm what do you do compared with <1cm?

1. measure TSH or US. 2. followed up with a radionuclide thyroid scan if results abnormal.

why is there sinus bradycardia with anorexia nervosa?

1. vagal hyperactivity resulting from an attempt to decrease the amount of cardiac work by reducing cardiac output. 2. low serum T3 levels, a common finding in persons with chronic malnutrition. sinus tachycardia can happen with refeeding syndrome.

Distal hyperplastic polyps are not markers for proximal or advanced neoplasia. next colonoscopy?

10 years.

tx for poison ivy dermatitis?

10-14 day taper of oral prednisone starting at 60 mg. However, 40-80 mg of intramuscular triamcinolone (or an equivalent) is an alternative to oral treatment, especially if adherence is an issue. Pruritus can be treated

SCFE is most likely to occur in which age group?

11-13 and 13-15 in girls and boys. Associated with growth spurts where femoral retroversion and steeper inclination of proximal femoral physis. --pain can refer to the knee.

Pneumococcal vaccine for what ages and smoking?

19-64 year olds.

Cow's milk recommended at age of?

12 months.

HbA1c of 6.0 correlates which what blood glucose level?

126. 7.0 correlates with 154.

worsening of preexisting orbitopathy is a well recognized potential complication of?

131I treatment and transient increase in thyroid hormone that can precipitate a thyroid storm.

Critically ill patients target glucose is?

140-180.

USPSTF recommends what for opt out HIV screening in

15-65 year olds. routine HIV screening in patients age 13-64 (SOR A). Opt-out screening is preferred to opt-in screening, as opt-in screening based on demographic, behavioral, or clinical subpopulations only identifies approximately 75% of patients with HIV

how high does serum bili need to be in order to initiate phototherapy in a baby: 29-48 hrs old. 49-72 hours old. 72+?

15. 18. 20. other than that, just increase feedings.

what are the diagnostic characteristics for acute rheumatic fever?

2 major or 1 major + 1 minor: Major criteria include carditis, migratory polyarthritis, erythema marginatum, chorea, and subcutaneous nodules. Minor criteria include fever, arthralgia, an elevated erythrocyte sedimentation rate or C-reactive protein (CRP) level, and a prolonged pulse rate interval on EKG.

When it it considered an H1N1 outbreak in a long term facility?

2 or more laboratory confirmed cased of influenza A.

what is the rome criteria for constipation?

2 or more of the following: 1. straining on defecation. 2. Hard stools. 3. incomplete evacuation. 4. less than three bowel movements per week.

Rear facing child safety seats recommended until?

2 year of age or 20 lbs.

usual site of stress fracture of foot?

2-4 metatarsals.

You test a patient's muscle strength and find that his maximum performance consists of the ability to move with gravity neutralized. This qualifies as which grade of muscle strength, on a scale of 0 to 5?

2. The inability to contract a muscle =0. Contraction without movement constitutes grade 1 strength. Movement against gravity only is grade 3 strength. Movement against gravity plus some additional resistance indicates grade 4 strength. Normal, or grade 5, strength is demonstrated by movement against substantial resistance.

grade 2 ascities (visible clinically by abdominal distention, not just with ultrasonography tx? Grade 3 (gross ascites with marked abdominal distention) tx?

2. diuretics with salt restriction. Aldosterone antagonists such as spirinolactone > loop. chlorthalidone is not recommended. 3. Large-volume paracentesis is the recommended .

Which one of the following is the best diagnostic test for vitamin D deficiency?

25- hydroxyvitamin D. vitamin D deficiency is defined by most experts as a 25-hydroxyvitamin D level <20 ng/mL (50 nmol/L).

NRT increases the chances of quitting smoking by how much? how long should tx be?

2X.8 weeks.

asymptomatic microscopic hematuria?

3 or more RBCs/ hpf. not based on dipstick reading.

Risk factors for proximal neoplasia include high-grade dysplasia, three or more adenomas, adenomas with villous features, and an adenoma ≥1 cm in size. For patients with one or more of these findings when should follow up colonoscopy be?

3 years. Distal hyperplastic polyps are not markers for proximal or advanced neoplasia.

when do you do surgical repair on umbilical hernia?

3-5 years if not resolved or if incarcerated at any age.

For a single rescuer performing CPR on a 2-year-old, the ratio is

30:2. 100 beats/min 1/3-1/2 of the depth.

maximum dose for hf of lisinopril?

40 mg/day.

Which one of the following patients is unlikely to benefit from vaccination against hepatitis A? (check one) A. A missionary traveling to Mexico B. A man who has sex with men C. A methamphetamine addict D. A patient with chronic hepatitis E. A 40-year old recent immigrant from India

40 year old recents immigrant from india.

In asymptomatic patients with no RF for DM american diabetes association says to screen for diabetes at what age?

45.

Prone to supine by age? Supine to prone by age?

4; 6 months.

Head banging has been estimated to be present in 3%-15% of normal children and usually begins between the ages of...extinction by how many years?

5-11 months. extinction by 3 yo.

how does calcaneal apophysitis (sever disease) present in runners? what age group?

5-11 yo= bones grow faster than muscle or tendons. -the tight achilles tendon pulls on its insertion site at the posterior calcaneus with repetitive running/ jumping= microtrauma in the area. -pain at posterior pad of foot.

COPD exacerbation that began with IV methyprednisolone is now able to tolerate oral. How many days of oral do you give the patient?

5-days total of systemic corticosteroids therapy. inhaled corticosteroids not that great.

what is alosetron and se?

5-hydroxytryptamine 3 antagonist and is FDA approved to treat severe diarrhea-predominant IBS only in women who have not improved with conventional therapy. (ischemic colitis, constipation, death) and its use is restricted in the United States.

Prostatic volume >40 mL (PSA >2.0) should have what tx?

5-reductase inhibitor treatment. b-blocker for symptomatic relief. no benefit from saw palmetto.

ectopic pregnancy or therapeutic abortion pose risk of fetomaternal hemorrhage. If the estimated gestational age is 12 weeks or less... mcg of RhoGAM is recommended. If the estimated gestational age is greater than 12 weeks, ... µg of RhoGAM is recommended.

50 mcg of RhoGAM. 300.

what is the screening test for lung cancer?

55-80 current smokers of quit in the past 15 years and have 30 pack year history. use low dose chest CT.

Which one of the following basal cell carcinomas is associated with the highest risk of recurrence? (check one) A. A 7-mm lesion on the nose B. A 9-mm lesion on the forehead C. A 12-mm lesion on the shoulder D. A 17-mm lesion on the arm

7 mm lesion on the nose. high risk locations= central face, eyelids, eyebrows, periorbital area, nose, lips (cutaneous and vermilion), chin, mandible, preauricular and postauricular skin/sulci, temple, and ear. genitalia, hands, feet. moderate risk with cheeks, forehead, scalp and neck.

A study finds that the positive predictive value of a new test for breast cancer is 75%, which means: (check one) A. if 100 patients with known breast cancer have the test, 75 (75%) will have a positive test result B. if 100 patients with no breast cancer have the test, 75 (75%) will have a negative test C. 75% of patients who test positive actually have breast cancer D. 75% of patients who test negative do not have breast cancer

75% of patients who test positive usually have breast cancer.

A 5-year-old male is scheduled for elective hernia repair at 11:00 a.m. Which one of the following would be the most appropriate recommendation? solid food. formula. breast milk. clear liquids.

8 hours for solid food 6 hours for formula 4 hours for breast milk 2 hours for clear liquids.

Acute otitis media what is the recommended dosage for amoxicillin?

80-90 mg/kg/day.

sitting position by age?

9 months.

to avoid osmotic demyelination syndrome how fast should hyponatremia be corrected?

<10-12 mmol/L in 24 hrs and <18 mmol/L in 48 hrs.

benign nodule characteristics include:

<5mm. Smooth border. Solid appearance. Concentric calcifications (no or eccentric calcifications malignant). double time of less than 1 month or more than a year.

target uric acid levels that should be kept to prevent joint damage?

<6.0 mg/dl.

When do you have to administer Hg?

<7g/dL.

patients with very complex health problems or poor health should have a goal HgbA1C of?

<8.5%.

what does your hgb level need to be in order to give blood transfusion?

<= 7g/dl.

Lab findings for polycythemia vera?

>16g/dL Hgb. >47% Hct. AA. And for white males: >18g/dL Hgb. >52%. Increased Red Cell Mass. splenomegly. >400,000. WBC>12000.

Which one of the following is recommended to test for the eradication of H. pylori in this patient? (check one) A. Immunoglobulin G serology B. A urea breath test C. Upper endoscopy with a biopsy D. An upper gastrointestinal series

A urea breath test. -- test and treat stratergy for H. pylori unless patient is >45 yo or has red flag for malignancy or complicated ulcer.

Which one of the following hospitalized patients is the most appropriate candidate for thromboembolism prophylaxis with enoxaparin (Lovenox)?

A 67-year-old female with hemiparesis, admitted for community-acquired pneumonia enoxaparin is clearly indicated in the 67-year-old who has limited mobility secondary to hemiparesis and is being admitted for an acute infectious disease. severe thrombocytopenia, and the patient with coagulopathy high bleed risk=

What type of pneumonia is usually seen in 3-16 week old kids? Common co-symptom?

Chlamydia trachomatis. -nontoxic afebrile. tachypneic. -diffuse crackles with few wheezes. -possible eosinophilia. --- see 50% cases with conjunctivitis.

A 15-year-old male is seen in the office for ankle pain. While playing basketball he jumped and landed on the lateral edge of his foot. He had immediate pain and did not continue playing, but was able to walk after the injury. On examination his right ankle has tenderness, swelling, and bruising over the anterior talofibular and calcaneofibular ligaments. There is no bony tenderness. Which one of the following would be most appropriate at this point?

A lace-up ankle support reduces pain and recovery time after an ankle sprain.

Pertussis dx?

A nasopharyngeal culture and polymerase chain reaction testing. Serologic testing is useful only in research settings, and direct fluorescent antibody testing is not recommended. Initial therapy= azithromycin= to decrease transmission but does not improve symptoms.

What do you know about death certificates?

A physician can certify a death from a natural cause but a coroner or medical examiner must certify a death due to any other cause The immediate cause of death is a specific etiology, not a general concept. "Uncertain" is not a manner of death, but "undetermined" may be used by coroners and medical examiners. The death certificate is a public document when filed.

When compared to a figure-of-eight dressing, which one of the following modalities of treatment has been shown to have similar fracture-healing outcomes and increased patient satisfaction for nondisplaced mid-shaft clavicular fractures?

A shoulder sling= --similar fracture healing rates. --with this patients have increased satisfaction when treated.

tx for gonococcal infection infantile is?

A single dose of 25-50 mg/kg of ceftriaxone administered intravenously or intramuscularly is the recommended treatment. Topical antibiotics are appropriate for prophylaxis, but not for treatment. Silver was used for prophylaxis at one time, but is no longer available. Povidone-iodine has not been studied for prevention.

An otherwise healthy 1-year-old white male has a screening hemoglobin level of 10.5 g/dL (N 11.3-14.1), a mean corpuscular volume of 68 fL (N 71-84), and an undetectable serum lead level. What should be your next step?

A therapeutic trial of iron for 1 month

Terminally ill patient receiving palliative chemotheraphy?

Less likely to die at home, more likely to have CPR and undergo mechanical ventilation, referred to hospice later and no survival benefit.

An otherwise healthy 1-year-old male is brought to your office because of increased respiratory effort, wheezing, and rhinorrhea. He has no fever. On examination he is found to have an increased respiratory rate and mild retractions. A chest film shows no foreign body or infiltrates. His O2 saturation is 94%. Management should include which one of the following?

A. A trial of nebulized albuterol (AccuNeb)= consistent with bronchiolitis, which is a response to a viral respiratory infection.

A 45-year-old female with rheumatoid arthritis has a hemoglobin level of 9.5 g/dL (N 11.5-16.0). Her arthritis is well controlled with methotrexate. Further evaluation reveals the following: Hematocrit............29.0% (N 35.0-47.0) Mean corpuscular volume............78 µm3 (N 80-98) Platelets............230,000/mm3 (N 150,000-400,000) WBCs............6900/mm3 (N 4000-11,000) Differential............normal Serum iron............15 µg/dL (N 50-170) Total iron binding capacity............150 µg/dL (N 45-70) Iron saturation............10% (N 15-50) Serum ferritin............7 ng/mL (N 12-150) Reticulocyte count............8 x 109/L (N 10-100) Stool guaiac............negative x 3 Which one of the following would be the most appropriate next step?

A. Evaluation for a source of blood loss

A 25-year-old female presents with a maculopapular rash that has progressed to multiple areas and exhibits target lesions. A cold sore appeared on her upper lip 2 days before the rash appeared. She is not systemically ill and is on no medications. Which one of the following is true concerning this problem?

A. Herpes simplex virus is a likely cause.

treatment regimens is most appropriate for an HIV-positive 42-year old who has latent tuberculosis infection?

A. Isoniazid daily for 9 months Rifampin is not recommended as monotherapy in patients with HIV infection because of increased rates of resistance and drug interactions with many antiretrovirals.

Studies indicate that patients most frequently want physicians to ask about their spiritual beliefs in which one of the following situations? (check one)

A. When being treated for a potentially fatal illness

According to the U.S. Preventive Services Task Force, multivitamin supplements in the geriatric age group:

A. are not recommended for prevention of any disorder

how can you diagnose someone with dm?

A1c level ≥6.5% be used to diagnose diabetes mellitus. fasting plasma glucose level ≥126 mg/dL a random glucose level ≥200 mg/dL in a patient with symptoms of diabetes a 2-hour oral glucose tolerance test value ≥200 mg/dL. While a urine dipstick may be used to screen for diabetes, it is not a diagnostic test.

Medication that can be used as part of a test for diagnosing renovascular hypertension, but would also increase the risk for azotemia if used for treatment?

ACE inhibitor= worsen renal failure in patient with htn caused by renovascular disease.

systolic heart failure what medications are good? A. Carvedilol (Coreg) B. Digoxin C. Ramipril (Altace) D. Spironolactone (Aldactone) E. Verapamil (Calan)

ACE inhibitors and β-blockers improve mortality in heart failure (HF). Digoxin and furosemide improve symptoms and reduce hospitalizations in systolic HF, and furosemide may decrease mortality. Spironolactone, an aldosterone antagonist, reduces all-cause mortality and improves ejection fractions in systolic HF. Verapamil, due to its negative inotropic effect, is associated with worsening heart failure and an increased risk of adverse cardiovascular events.

Bicuspid aortic valve is inherited how?

AD.

Patients with which rheumatologic condition have the highest relative risk of internal malignancy compared to the general population? (check one) A. Systemic scleroderma B. Systemic lupus erythematosus C. Sjögren's syndrome D. Rheumatoid arthritis E. Dermatomyositis

E. 32% diagnosed had cancer. polymyositis 15%.

most likely to occur with long-term chronic use of opioids? (check one) A. Diarrhea B. Sedation C. Hypoalgesia D. Respiratory depression E. Hypogonadism

E. Hypogonadism.

A 36-year-old female consults you because of concerns about "fatigue." After carefully reviewing her history and performing a physical examination, which one of the following would be LEAST valuable in assessing this patient? A. A baseline serum cortisol level B. An erythrocyte sedimentation rate C. A complete metabolic panel D. A TSH level E. A pregnancy test

Baseline serum cortisol level= only in patients with significant findings of Addison's disease.

for a hyperthyroidism patient what medication is preferred?

methimazole > PTU because of hepatocellular damage with PTU.

what is contraindicated in htn patient with uterine atony?

methyergonovine.

elevated levels of which one of the following would suggest vitamin B12 deficiency?

methylmalonic acid or homocysteine levels.

what antidepressant has shown evidence of improving depression symptoms in children and adolescents?

Fluoxetine. escitalopram and citalopram and sertraline are all good for kids but not as good as fluoxetine.

To avoid inadertent tx of HIV with emtricitabine/ tenofovir is what?

Ab testing for HIV= no more than 7 days before initially prescribing pre-exposure prophylaxis with emtricitabine/tenofovir disoproxil.

A 23-year-old male presents to your office with a 2-day history of dull, achy, right testicular pain. He reports that the pain began gradually, reaching a peak last night. He does not recall any trauma and denies any urethral complaints. Your examination reveals an extremely tender right testis with some tenderness extending to the epididymis. A preliminary report from a stat ultrasound examination shows an enlarged, heterogeneous right testis with increased color flow. Which one of the following is the preferred management?

Ab tx. Acute epididymitis. --most commonly under the age of 35 cause chlamydia. unprotected rectal intercourse= enterobactericae. unilateral absent flow on color and spectral doppler sonography= high sensitive and specific or acute testicular torsion= emergency.

normotensive patient with CAD already on statin, aspirin, lisinopril htzd should be prescribed?

metoprolol.

acute mountain sickness ... can be used for both prevention and treatment.

Acetazolamide or dexamethasone can be used for both prevention and treatment.

A patient is admitted to the hospital for acute deep vein thrombosis of the lower extremity and started on anticoagulation therapy. The nursing staff asks for an activity order. Which one of the following should be ordered?

Activity as tolderated. --early ambulation over initial bed rest in patients with acute DVT of the leg. -- if edema and pain are severe, ambulation maybe deferred. early ambulation as soon as effective anticoagulation has been achieved.

A 42-year-old male with a history of chronic hepatitis C develops left leg cellulitis and is treated with cephalexin (Keflex). He returns to your office 5 days later for follow-up, and the cellulitis is responding favorably to treatment. However, the patient has a generalized maculopapular rash and a low-grade fever, which he says began 3 days ago. He also complains of arthralgias. You admit him to the hospital for further evaluation. His serum creatinine level is 3.2 mg/dL (N 0.6-1.5), which is elevated from his baseline level of 0.8 mg/dL. A urinalysis is normal, except for the presence of occasional eosinophils. The remainder of his evaluation, including liver enzyme levels and renal ultrasonography, is normal. wtf is going on? tx?

Acute kidney injury= -rise in serum creatinine or reduction in urine output. -cr increase by 0.3 mg/dl or to 50% above baseline within 24-48 hrs. -reduction in urine output to 0.5 ml/kg/hr for >6 hrs also meets criteria. --acute interstitial nephritis is an intrinsic cause of aki.

Thyroid tumor location?

midline.

A 22-year-old male presents to your office for evaluation of fatigue, poor appetite, and nausea. He states that when he stands too long he often gets dizzy but this is relieved by sitting. His symptoms have been gradually getting worse over the past year. His vital signs are normal but he is found to be orthostatic. A physical examination is unremarkable except for hyperpigmentation in his palmar creases and around his nipples. A basic metabolic panel is notable for a sodium level of 131 mEq/L (N 135-145) and a potassium level of 5.1 mEq/L (N 3.5-5.0). Which one of the following is the most likely cause of this patient's symptoms?

Addison's disease= autoimmune adrenalitis= adrenal cortex destroyed so no more mineralo/gluco corticoids and adrenal androgen hromones. Common symptoms of Addison's disease include anorexia, weakness, fatigue, gastrointestinal symptoms, hypotension, salt cravings, postural dizziness, vitiligo, muscle pain, and joint pain. Hyperpigmentation is the most common physical finding and is generally distributed diffusely over the entire body. It can also be seen in the palmar creases, at the vermillion border of the lips, on the buccal mucosa, around the nipples, and around scars. lack of mineralocorticoids= hyperkalemia with hyponatremia.

A 20-year-old patient comes to the emergency department complaining of shortness of breath. On examination his heart rate is 180 beats/min, and his blood pressure is 122/68 mm Hg. An EKG reveals a narrow complex tachycardia with a regular rhythm. Which one of the following would be the most appropriate initial treatment?

Adenosine= SVT after vagal maneuver. if that does not work then syncronized cardioversion.

one of the following nutritional interventions should be recommended to accelerate pressure ulcer healing in the elderly?

Adequate protein intake (1.2-1.5 g/kg/day) speeds up the process. Increased caloric intake is also necessary to promote healing. The role of vitamins and minerals in preventing and treating pressure ulcers is unclear.

A 50-year-old female complains of a 6-month history of the insidious onset of right shoulder pain and decreased range of motion. She does not respond to consistent use of prescription strength anti-inflammatory medication. Radiographs are negative. Dx? Treatment of this patient's condition should include:

Adhesive capsulitis or degenerative rotator cuff tendinopathy. do physical therapy with home exercise.

Typically, a high-grade squamous intraepithelial lesion (HSIL) of the cervix is treated with ablation or excision. In which one of the following can treatment be deferred? (check one) A. Adolescents B. Patients attempting to conceive C. Patients with a history of three previous normal Papanicolaou smears D. Patients with a negative DNA test for HPV E. Patients over the age of 70

Adolescents: -if you are attempting to conceive, you cannot be conservative (progressive disease during pregnancy and not being able to tx as much). -A negative test for HPV =assess the risk of patients with atypical squamous cells of undetermined significance (ASC-US) or a low-grade squamous intraepithelial lesion (LSIL); it does not change the management of patients with a high-grade intraepithelial lesion (HSIL). -older patients, when abnormality is found you still tx.

the headaches are unilateral, infrequent, and throbbing; the headaches are associated with nausea and vomiting; and sleep offers relief.

migraine.

Initial approach to consitipation in children?

An oral osmotic agent such as polyethylene glycol 3350 (MiraLax)= are effective, easy to administer, noninvasive, and well tolerated. Rectal therapies are similar in terms of effectiveness but are more invasive and less commonly used as first-line treatment Oral stimulants and bisacodyl rectal suppositories are not recommended for children under 2 years of age.

A higher dosage of warfarin (Coumadin) would be required to achieve a therapeutic INR if the patient were found to have conditions?

Hypothyroidism, visceral carcinoma, increased vitamin K intake, DM, hyperlipidemia.

Which one of the following potential bioterrorism agents requires treatment with 60 days of continuous antibiotics?

Anthrax= oral fluoroquinolones and doxycycline.

A 70-year-old white female complains of two episodes of urinary incontinence. On both occasions she was unable to reach a bathroom in time to prevent loss of urine. The first episode occurred when she was in her car and the second while she was in a shopping mall. She is reluctant to go out because of this problem. The most likely cause of her problem is:

Answer: urge incontinence: sense the urge to void but are unable to inhibit leakage long enough to reach the toilet. overflow incontinence, which occurs when the bladder cannot empty normally and becomes overdistended. The term functional incontinence is applied to those cases where lower urinary tract function is intact but other factors such as immobility and severe cognitive impairment lead to incontinence.

tx for necrotizing fascitis?

Antibiotics essential but not sufficient; aggressive surgery debridement within 12 hours reduces the risk of amputation and death.

Which one of the following is most appropriate for patients with asplenia?

Antibiotics for any episode of fever. PCV13 and PCV 23 should be given at least 8 weeks apart. Live attentuated vaccine okay for aplenia unless sickle cell.

A 50-year-old male presents with a 1-day history of fever and chest pain. The chest pain is worse when he is in a supine position and with deep inspiration, and improves when he leans forward. He has no shortness of breath and has never had this problem before. His vital signs are normal except for a temperature of 37.8°C (100.0°F). He has no other medical problems or allergies, and takes no medications. An EKG reveals widespread ST-segment elevation, upright T waves, and PR-segment depression. His troponin level is normal. An echocardiogram is pending. Which one of the following would be the most appropriate treatment for this patient?

Aspirin; acute pericarditis.

syncope related to changes in position, such as bending, lying down from a seated position, or turning over in bed.

Atrial myxoma.

A 58-year-old male who works with heavy machinery at a local factory presents to your office for evaluation of hearing loss of several years' progression. He notes that the loss is mainly in the left ear and he also has mild tinnitus. He has had no trauma to his head, and he has no history of ear infections. Examination of the ears reveals normal tympanic membranes and a neurologic examination is negative. When a tuning fork is placed in the center of his forehead, he says the sound is much louder on the right side (Weber test). Comparing sound in front of the ear to the sound when the tuning fork is placed on the mastoid (the Rinne test) reveals that air conduction is better than bone conduction in the left ear. Which one of the following is true regarding further evaluation and management?

Audiometry is the best initial screening test. -Acoustic neuroma symptoms are due to cranial nerve involvement and progression of tumor size. Hearing loss is present 95% of the time and tinnitus is very common. diagnosis of acoustic neuroma is based on asymmetric sensorineural hearing loss or another cranial nerve deficit, with confirmation based on MRI with gadolinium contrast or a CT scan.

Tx and prophylaxis of pertussis?

Azithromycin. --TMP/SMX alternative in case of allergy or intolerance to macrolides.

what vaccine is likely to cause febrile seizures 2 weeks after getting it?

MMR; measles part. Postimmunization seizures, especially febrile seizures, occur at a higher rate in children who have a past history of seizures or a first-degree relative with a history of seizures.

Under current guidelines, hospice programs are most likely to serve patients dying from: (check one) A. heart failure B. COPD C. severe dementia D. multiple strokes E. cancer

E. cancer. Diseases such as COPD, end-stage liver disease, and heart failure result in long-term disability with periodic exacerbations, any one of which could result in death, but far less predictably.

In a woman whose group B Streptococcus status is unknown, which one of the following is a risk factor requiring empiric intrapartum antibiotic prophylaxis against early-onset group B streptococcal infection in her newborn? (check one) A. Fetal tachycardia B. Delivery at less than 35 weeks gestation C. Rupture of the membranes 12 hours before delivery D. Gestational diabetes during the pregnancy E. Use of vacuum extraction during delivery

B. Delivery at less than 35 weeks gestation none of the others are risks for early onset GBS in neonates.

if someone otherwise normal having olfactory problem do what for management next step?

MRI of the brain.

treatment for polymorphic ventricular tachycardia is?

B-blockers and implanted cardioverter defibrillator.

causes of anemia is associated with a normal red cell distribution width?

B-thalassemia. uncomplicated heterozygous thalassemia.

patient comes in with hyperkalemia, step 1?

EKG.

An anxious 62-year-old white male comes to the emergency department complaining of extreme shortness of breath and a cough producing blood-tinged sputum. The patient denies chest pain and fever. On examination he is afebrile and has expiratory wheezes and a few rales throughout the chest. The heart is normal except for a rapid rate and an S3 gallop. A chest radiograph reveals a right pleural effusion with enlargement of the cardiac silhouette and redistribution of blood flow to the upper lobes. Which one of the following tests would be best for confirming the diagnosis? (check one) A. Troponin I B. BNP C. D-dimer D. CT angiography of the chest E. Arterial blood gases

B. BNP; heart failure. S3 gallop happens with dilated left ventricle and right pleural effusion (common in HF). --bnp helps to determine if it is cardiac or pulmonary.

Which one of the following, when added to his current regimen, has evidence to support its use in preventing all-cause mortality? (check one) A. Chlorthalidone B. Spironolactone (Aldactone) C. Ezetimibe (Zetia) D. Losartan (Cozaar) E. Fish oil

B. Spironolactone (Aldactone).

A 54-year-old male presents with hearing loss associated with tinnitus. Which one of the following additional characteristics would be an indication for MRI of the brain to assess for an intracranial tumor? (check one) A. A rapid onset of symptoms B. Unilateral symptoms C. Association with pain and otorrhea in the affected ear D. Exposure to loud noise shortly before the symptoms began

B. Unilateral symptoms. =Acoustic neuroma is a slow-growing benign tumor of the Schwann cells surrounding the vestibular cochlear (8th cranial) nerve.

if someone has a low or moderate pretest probability of PE what test should you do?

ELISA based d-dimer.

Which one of the following is true concerning Norwalk virus? (check one) A. Outbreaks occur mostly in settings with large numbers of children, such as schools and day-care centers B. Viral shedding continues long after the acute illness C. The virus does not survive long on most environmental surfaces D. An episode of Norwalk gastroenteritis leads to long-lasting immunity E. It is a less common cause of diarrhea in adults than Shigella

B. Viral shedding continues long after the acute illness & often precede the onset of illness.

which one of the following features would be more characteristic of depression as opposed to a grief reaction? (check one) A. Insomnia B. Loss of interest or pleasure in all activities C. Feelings of guilt D. Thoughts of wanting to die E. Psychomotor agitation

B. anhedonia.

Which one of the following is more likely to occur with glipizide (Glucotrol) than with metformin (Glucophage)? A. Lactic acidosis B. Hypoglycemia C. Weight loss D. Gastrointestinal distress

B. hypoglycemia. --metformin can cause lactic acidosis if patient has renal impairment. --does not cause weight gain (maintains or helps lose).

Nausea and vomiting in pregnancy good tx?

B6 or metoclopramide (no adverse risk to fetus).

when does staph food poisoning begin and end?

Begins 1-6 hrs. Resolves 24-48 hrs.

persistent occiput posterior position is associated with higher risk of?

C and assisted vaginal delivery and lower chance of vaginal.

10-year-old son because of a swollen area in his neck that she first noticed yesterday. He has also had symptoms of an upper respiratory infection. On examination the child has a runny nose but otherwise appears well. Palpation reveals a soft, 1.5-cm, slightly tender mass, inferior to the angle of the mandible and anterior to the sternocleidomastoid muscle. The most likely diagnosis is

Brachial cleft cyst. -20% of neck masses in children. They commonly present in late childhood or adulthood, when a previously unrecognized cyst becomes infected. They are most frequently found anterior to the sternocleidomastoid muscle, but can also be preauricular.

Which one of the following can help to minimize the pain of lidocaine (Xylocaine) injection?

Buffering the solution with sodium bicarbonate --effect enhanced when solution is room air and rapidly inserting he needle through the skin, injecting slowly and steadily while withdrawing the needle and injecting into the subQ tissue also minimizes the pain.

A 21-year-old primigravida at 10 weeks gestation has a negative titer for rubella. The best procedure to follow is to? (check one) A. Institute a (-globulin regimen and maintain it throughout her pregnancy B. Administer rubella vaccine after 12 weeks gestation C. Administer rubella vaccine immediately post partum D. Administer rubella vaccine 12 weeks post partum

C. Administer rubella vaccine immediately post partum.

child has stridor and intercostal retractions + nasal congestion and barking cough/ hoarseness + afebrile. tachypnea, inspiratory and expiratory stridor, tx?

C. Aerosolized epinephrine and intramuscular dexamethasone

The mother of a 16-year-old male calls to report that her son has a severe sore throat and has been running a fever of 102°F. Which one of the following additional findings would be most specific for peritonsillar abscess? A. A 1-day duration of illness B. Ear pain C. Difficulty opening his mouth D. Hoarseness E. Pain with swallowing

C. Difficulty opening his mouth.

Which one of the following nutritional management strategies is associated with better outcomes in patients with mild acute pancreatitis whose pain and nausea have resolved? (check one) A. Waiting until lipase has normalized before beginning oral intake B. Early initiation of a clear liquid diet C. Early initiation of a low-fat diet D. Early initiation of tube feeding E. Early initiation of total parenteral nutrition 70

C. Early initiation of a low-fat diet bowel rest is associated with intestinal mucosal atrophy and increased infectious complications because of bacterial translocation from the gut.

Stretching has NO demonstrable benefit for which one of the following? (check one) A. Hamstring strain B. Chronic neck pain C. Joint contracture D. Osteoarthritis E. Rehabilitation post knee replacement

C. Joint contracture This mobility-impairing condition results from post-healing shortness of noncontractile tissues that are not easily released with stretching.

A 44-year-old male in the intensive-care unit develops acute respiratory distress syndrome (ARDS). Which one of the following has been shown to improve outcomes in this situation? (check one) A. Surfactant B. Lower positive end-expiratory pressure (PEEP) settings C. Lower tidal volumes D. Aggressive fluid therapy E. Pulmonary artery catheters

C. Lower tidal volumes

A 70-year-old white male with hypertension has several abnormal liver function tests on routine testing. He says he does not drink alcohol, and the prescription medications he is taking are unlikely to cause hepatotoxicity. However, during more extensive history taking, he tells you that he does use some over-the-counter medications. Which one of these is most likely responsible for the abnormal laboratory findings? (check one) A. Aspirin, used occasionally for headache B. A fiber supplement taken to promote regular bowel habits C. One long-acting niacin tablet per day D. One 250-mg vitamin C tablet daily E. Chewable simethicone after meals, almost daily

C. One long-acting niacin tablet per day in elderly hepatotoxicity. Aspirin and vitamin C can result in gastrointestinal iron loss and anemia.

The 1990 Patient Self-Determination Act requires that: (check one)

C. hospitals ask patients about advance directives ask patients about advance directives and then incorporate the information into medical records.

Leading cause of death for patients with RA?

CAD. accelerated atherosclerosis related to a chronic inflammatory state. I

tx for chronic fatigue syndrome?

CBT and graded exercise therapy.

recommended for the treatment of patients with obsessive compulsive disorder?

CBT.

drug-induced myopathy caused by the simvastatin, which is associated with elevated

CK.

what can cause SIADH? pathology? Drugs?

CNS tumors, infections (meningitis) and pneumonia. amiodarone, carbamazepine, SSRIs, and chlorpromazine.

who should you not use zanamivir in?

COPD, asthma, or respiratory distress.

Patients with severe sleep apnea (apnea-hypopnea index >29) and concomitant cardiovascular disease benefit the most from treatment for obstructive sleep apnea. Because it is relatively easy to implement and has proven efficacy what is used?

CPAP first line.

A 58-year-old female presents with a 6-month history of persistent intermittent unilateral rhinorrhea. The drainage is clear, and seems to be worse in the early morning when she first gets up. Her past medical history includes hypertension and controlled migraines. Her surgical history includes a total hysterectomy 5 years ago and septal deviation surgery 7 months ago. She has tried oral antihistamines and intranasal corticosteroids without relief. The patient should undergo further evaluation for:

CSF fluid rhinorrhea.

RLQP imaging preferred?

CT IV contrast.

Which one of the following is true regarding the risk of inducing cancer with CT scanning? (check one) A. CT of the chest is associated with a greater risk than CT of the head B. The risk increases with age at the time of the scan C. Males have a greater risk of ultimately developing CT-induced lung cancer than females D. Current techniques with rapid scanners make the risk comparable to that associated with standard radiographs of the same area E. The risk in neonates is markedly reduced because of the efficiency of DNA repair processes at this age

CT of the chest is asosciated with greater risk than CT of the head because more radiation expsoure and cancer risk. younger kids have higher risk of developing cancer after exposure. and ct has more risk than plain radiograph. women higher risk than males.

hematuria in all patients, with the exception of those with generalized renal parenchymal disease, young women with hemorrhagic cystitis, children, and pregnant females.

CT urography or intravenous pyelography

LLQ pain imaging?

CT with oral and IV contrast.

gold standard for the diagnosis of renal colic?

CT.

tx for Severe hyperkalemia (>7.0 mEq/L);:

Calcium chloride or gluconate (rapidly stabilizes the membranes of cardiac myocytes, reducing the risk of cardiac dysrhythmias). Therapies that translocate potassium from the serum to the intracellular space should be instituted next, as they can quickly (albeit temporarily) lower the plasma concentration of potassium. = sodium bicarbonate, glucose with insulin, and albuterol. Total body potassium can be lowered with sodium polystyrene sulfonate

A 74-year-old male presents with a 4-day history of diarrhea that he had initially thought was "a 24-hour virus." He states that the onset of his illness included nausea, one episode of vomiting, and profuse diarrhea. He has felt feverish and has been having abdominal cramps. He does not recall eating anything unusual and has not traveled recently. On examination he appears uncomfortable, but in no real distress. His oral temperature is 37.1°C (98.8°F), blood pressure 134/82 mm Hg, and pulse rate 100 beats/min. He has lost 4 kg (9 lb) since his last visit 2 months earlier. His abdomen is soft, with hyperactive bowel sounds and mild diffuse tenderness on palpation. A CBC and basic metabolic profile are normal. Which one of the following is the most likely cause of this patient's illness?

Campylobacter. Diarrhea is the predominant symptom, with a lesser degree of nausea and vomiting. Up to 10 days is required for full recovery.

If patient comes in for amoxicillin for pharyngitis and then 8 days into the treatment gets a maculopapular rash what do you do?

Cannot differentiate allergy from viral exanthema so: Discontinue the amoxicillin and note amoxicillin as a potential allergy in her record

A 60-year-old male complains of multiple episodes of lightheadedness over the past 3 months, saying he felt as if he might "pass out" while sitting at his desk. His past medical history and a physical examination are unremarkable. An EKG shows right bundle branch block and left anterior hemiblock. Most appropriate next step?

Cardiac event monitoring. --trifascicular block at risk for tachyarrhythmias and bradyarrhythmias.

Expectant management of preterm labor cases.

Careful monitoring for fetal compromise, consultation with obstetric colleagues, and neonatal intensive-care unit involvement. =expectant management.

what is the recommended antibiotic prophylaxis for orthopedic procedure?

Cefazolin; unless there is a b-lactam allergy.

Epididymitis tx?

Ceftriaxone and doxycycline/ azithromycin.

A 6-year-old male is diagnosed with acute bacterial sinusitis. He has a previous history of a rash 5 days after beginning penicillin treatment. Which one of the following medications is most appropriate for this patient?

Cefuroximen.

Someone with RA getting surgery done under general anesthesia= what is the most important for preoperative assessment of this patient's surgical risk?

Cervical spine imaging= to detect atlantoaxial subluxation would be most important for preventing a catastrophic spinal cord injury during intubation.

otitis externa tx with topical:

Ciprofloxacin 0.3%/dexamethasone 0.1% (Ciprodex) topically. --nothing oral unless it has spread beyond the ear canal or if the patient has DM/ immunocompromised.

Acute pyelonephritis commonly caused by e.coli emperic tx?

Ciprofloxacin.

Prolonged QT, what antibiotic do you want to avoid?

Clarithromycin interferes with the delayed rectifier potassium current, which results in the accumulation of potassium ions in cardiac myocytes and thereby delays cardiac repolarization. This leads to prolongation of the QT interval and therefore the risk of fatal arrhythmia. Clarithromycin is metabolized by the cytochrome P450 3A enzyme. When using clarithromycin it is important to avoid any other medications that may inhibit this enzyme, leading to higher clarithromycin levels.

A 68-year-old white male with diabetes mellitus is hospitalized after suffering a right middle cerebral artery stroke. A nurse in the intensive-care unit calls to advise you that his blood pressure is 200/110 mm Hg. You should: A. continue monitoring the patient B. administer labetalol (Trandate) C. administer nicardipine (Cardene) D. administer nitroprusside (Nitropress) E. administer nitroglycerin

Continue monitoring the patient; no additional treatment for patients with a systolic blood pressure <220 mm Hg or a diastolic blood pressure <120 mm Hg. elevated BP= protective to increase cerebral perfusion and by lowering it you increase morbidity. tighter control after first 24 hrs.

best initial treatment for an itchy rash with an oval 12 cm erythematous plaque on her buttocks. Plaque is covered with silvery scales. she has tried OTC hydrocortisone cream with no improvement. Tx?

Clobetasol 0.05% cream applied twice daily. -- good for mild plaque psoriasis. Topical corticosteroids, vitamin D analogs, and tazarotene effective only for mild psoriasis.

acute pericarditis should be treated empirically with?

Colchicine and/or NSAIDs for the first episode of mild to moderate pericarditis.

A 26-year-old gravida 3 para 2 was diagnosed with gestational diabetes mellitus at 24 weeks gestation. She was prescribed appropriate nutritional therapy and an exercise program. After 4 weeks, her fasting plasma glucose levels remain in the range of 105-110 mg/dL. Which one of the following would be the most appropriate treatment for this patient at this time?

Combination of an intermediate acting insulin (NPH) and short acting (lispro) twice a day. start tx for patients that have above 100 despite diet and exercise. insulin is best pharmacologic tx for GD.

Varenicline with nicotine replacement does this need to happen?

Combining these medications has not proven to be beneficial. --varenicline blocks the benefit a patient would receive from nicotine replacement products. --both together can actually lead to increase in nausea, headaches, dizziness and fatigue.

According CDC guidelines, does a patient need to use backup contraception if her IUD is inserted today within 7 days of last menstraul bleeding? what if it has been more than 7 days since her menstrual bleeding starting?

No. abstain from sex/ or use additional protection for 7 days.

what can help help acidify urine?

Cranberry juice or betaine can lower urine pH.

A 35-year-old female with a history of chronic abdominal pain and diarrhea develops tender red nodules on her shins. These findings are most consistent with which one of the following?

Crohn's disease is associated with many extragastrointestinal conditions: erythema nodosum (as with this patient), anemia, inflammatory arthropathies, uveitis, and venous thromboembolism

most accurate for estimating gestational age?

Crown rump length 10 weeks.

neuropathy of the ulnar nerve caused by compression or traction as it passes through the cubital tunnel of the medial elbow. The onset of pain is more insidious than UCL injury, occurring with repetitive activity, and is usually accompanied by numbness and tingling in the ulnar border of the forearm and hand. If it has existed for some time, the intrinsic hand muscle may become weak.

Cubital tunnel syndrome.

for a patient coming in for HF that needs to get rid of liquid fast. should a catheter be placed?

No. A catheter should not placed for managing incontinence or for staff convenience or for monitoring output in patietns who are not critically ill.

A 91-year-old white male presents with a 6-month history of a painless ulcer on the dorsum of the proximal interphalangeal joint of the second toe. Examination reveals a hallux valgus and a rigid hammer toe of the second digit. His foot has mild to moderate atrophic skin changes, and the dorsal and posterior tibial pulses are absent. Appropriate treatment includes which one of the following?

Custom made shoes to protect hammer toe. because they are so old. symptomatic supportive, conservative therapy.

AUB tx?

Cyclic administer of progesterone for 3 months. Once pregnancy, hematologic disease, and renal impairment are excluded, administration of intramuscular or oral progesterone will usually produce definitive flow and control the bleeding. No further evaluation should be necessary unless the bleeding recurs.

In frail elderly patients, starvation can be distinguished from cachexia by which one of the following? (check one) A. An inflammatory response seen in starvation B. A normal appetite in the early stages of cachexia C. A rapid decrease in albumin in the early stages of starvation D. A reversal of changes with refeeding in starvation

D. A reversal of changes with refeeding in starvation Appetite is decreased early in cases of cachexia but remains normal in the early stages of starvation.

Which one of the following is the most appropriate next step in ruling out deep vein thrombosis in a patient with mild risk of DVT? A. D-dimer B. Ultrasonography C. Venography D. Impedance plethysmography

D-dimer. has a high NPV for DVG. -negative D-dimer does not rule out dvt. but + does rule it in.

In the secondary prevention of ischemic cardiac events, which one of the following is most likely to be beneficial in a 68-year-old female with known coronary artery disease and preserved left ventricular function?

Effective treatments include aspirin, β-blockers after myocardial infarction, ACE inhibitors in patients at high risk after myocardial infarction, angiotensin II receptor blockers in those with coronary artery disease, and amiodarone in patients who have had a myocardial infarction and have a high risk of death from cardiac arrhythmias.

RF for intermittent claudication: A. Hyperthyroidism B. Hypercalcemia C. Diabetes mellitus D. Hypogonadism E. Elevated angiotensin-converting enzyme

DM and cigarrete smoking (htn and dyslipidemia too).

Skin tags, or acrochordons, are associated with?

DM and obesity. -most commonly neck and axillae.

Which one of the following is the most common secondary cause of nephrotic syndrome in adults? A. Diabetes mellitus B. Systemic lupus erythematosus C. Hepatitis D. NSAIDs E. Multiple myeloma

DM.

necrobiosis lipoidica associated with

DM.

50-year-old male presents to your office with erythroderma and fever. He has not had a sore throat, rhinorrhea, cough, or urinary tract symptoms. His current medications include lisinopril (Prinivil, Zestril), atenolol (Tenormin), and allopurinol (Zyloprim). temperature of 38.6°C (101.5°F). The skin is remarkable for marked erythema over 90% of the body, with tenderness to touch. His mental status is clear and his neck is supple. Mildly tender adenopathy is noted in the neck, axillae, and groin. He has no oral ulcerations or ocular symptoms. WBC count of 15,000/mm3 (N 4300-10,800) with 20% eosinophils. A metabolic profile shows an AST (SGOT) level of 100 U/L (N 10-40) and an ALT (SGPT) level of 110 U/L (N 10-55), but is otherwise normal.

DRESS= erythroderma accompanied by fever, lymphadenopathy,elevation of liver enzymes and eosinophilia. --allopurinol induced DRESS has the highest mortality rate.

are usually mobile, moving with the overlying skin. They can be located in the submental or midline region.

Dermoid cysts.

false-positive urine drug screen for opioids:

Dextromethorphan, diphenhydramine, ibuprofen, and even fluoroquinolones.

what medications out of these below are harmful for severe CHF? A. Diltiazem (Cardizem) B. Lisinopril (Prinivil, Zestril) C. Carvedilol (Coreg) D. Atorvastatin (Lipitor)

Diltiazem. Nondihydropyridine calcium channel blockers with negative inotropic effects (verapamil and diltiazem) may be harmful in patients with low left ventricular ejection fractions. Evidence-based $-blockers (carvedilol or metoprolol succinate) should be used in all patients with a history of myocardial infarction.

Capacity is done by Competency is one by?

Doctor. Lawyer.

A 52-year-old male presents with a small nodule in his palm just proximal to the fourth metacarpophalangeal joint. It has grown larger since it first appeared, and he now has mild flexion of the finger, which he is unable to straighten. He reports that his father had similar problems with his fingers. On examination you note pitting of the skin over the nodule. The most likely diagnosis is:

Dupuytren's contracture.

Mid substance achilles tendinopathy with symptoms for 8 weeks. First line tx?

Eccentric exercise. patient should stand on the ball of the injured foot with the calcaneal area of the foot over the edge of a stair step.

In a patient with chronic hepatitis B, which one of the following findings suggests that the infection is in the active phase?

Elevated ALT.

5-year-old daughter to see you because she found a mass in the child's neck. The mass appeared over the past week and was preceded by a sore throat. Her pharyngitis is now resolved but she still has a fever, although it is not as high. The mother is most concerned because the mass developed over a short span of time, and it is warm, red, and tender. When asked, she says that her daughter has had no recent exposure to cats. When you examine the child you note that her temperature is 38.0°C (100.4°F). You also find shotty adenopathy in both anterior cervical lymph node chains, and a 2.5-cm warm, firm, moderately tender lymph node in the right anterior cervical chain. The overlying skin is also erythematous. Which one of the following would be the most appropriate management at this time?

Empiric antibiotic therapy with observation for 4 weeks. Cervical lymphadenitis= systemic symptoms, unilateral lymphadenopathy, skin erythema, node tenderness and size 2-3 cm.

In older patients with aortic stenosis and a systolic murmur, which one of the following would be most concerning?

Exertional dyspnea.

A 39-year-old female presents with a 4-month history of gradually worsening left elbow pain. She does not recall an injury but frequently lifts and holds her 10-month-old son in her left arm. She has tenderness over the lateral epicondyle. Her elbow range of motion is normal but she has pain with supination and pronation. The remainder of the examination is normal. For long-term pain relief, the best evidence supports which one of the following?

Expectant/ conservative management. --lateral epicondylitis due to degeneration of extensor carpi radialis originating at lateral epicondyle. --self limited and resovles in 12-18 months without tx. --corticosteroid injection= poorer outcome.

... a type of strabismus with an inward or nasal deviation of the eye that would be evidenced by a corneal light reflex lateral to its normal position.

Esotropia is.

FDA approval, imiquimod is approved only for treatment of

External anogenital warts if 12+.

The risk of heat wave related death is highest in those who?

Factors associated with a higher risk of heat-related deaths include being confined to bed, not leaving home daily, and being unable to care for oneself.

heel pain in the geriatric patient causes pain as the day progresses.

Fat-pad atrophy. versus plantar fascitis bad in the morning.

Preferred medications for patients with end stage renal disease?

Fentanyl and methadone. fentanyl metabolized by the liver and no active metabolites.

... is most often seen with delirium tremens and is less common with less severe forms of alcohol withdrawal.

Fever.

HyperTG tx?

Fibrate.

You are evaluating a 68-year-old male with obstructive urinary symptoms. Which one of the following medications may lead to falsely depressed levels of prostate-specific antigen (PSA)?

Finasteride. helps with obstructive symptoms.

androgenic alopecia (male-pattern baldness) tx option?

Finasteride= 5-beta reductase type 2 inhibition= decreased dihydrotestosterone levels. Spirinolactone.

is 1:4 antibody titer better or worse than 1:64 for rhogam?

For example, an antibody titer of 1:4 poses much less risk to the fetus than a titer of 1:64. .

Go over the different time periods for re-colonoscopy.

For patients at low risk the recommended interval between screening colonoscopies is 10 years. Reductions in this interval are recommended for patients with one or two small tubular adenomas (5-10 years) or those with three or more tubular adenomas (3 years); the interval for more extensive disease is best individualized but can be as often as annually in unusual cases.

A 4-year-old male sees you for pre-kindergarten screening. On corneal light reflex testing, the light reflex in the patient's right eye is in the center of the pupil. In the left eye it is located below the pupil, over the inferior-lateral portion of the iris. This clinical finding is associated with a congenital palsy of which one of the following cranial nerves?

Fourth; in patients with esotropia the reflex will be over the lateral portion of the iris in the affected eye. in exotropia the light reflex is over the medial iris. Hypertropia the reflex it is over the inferior iris. Hypotropia it is over the superior iris.

A 49-year-old male feels fine, but his bilirubin level was 2.5 mg/dL (N <1.0). 5 alcoholic beverages per week and takes no medications other than occasional ibuprofen. not jaundiced and has no scleral icterus, and the remainder of the examination is within normal limits, including palpation of the liver and spleen. Laboratory testing reveals a normal CBC, normal liver enzyme levels, and normal serum haptoglobin. Bilirubin fractionation reveals an indirect level of 2.0 mg/dL and a direct level of 0.5 mg/dL (N <0.4). The most likely diagnosis is

Gilbert's syndrome: --unconjugated bilirubin level (usually <5.0 mg/dl). bilirubin increases with infection, exertion, fasting. --asymptomatic with normal LFT. hemolytic anemias, which cause a decrease in serum haptoglobin, an increase in lactate dehydrogenase, and/or CBC abnormalities, particularly on the peripheral smear.

A 46-year-old female presents to your office with a 2-week history of pain in her left shoulder. She does not recall any injury, and the pain is present when she is resting and at night. Her only chronic medical problem is type 2 diabetes mellitus. On examination, she has limited movement of the shoulder and almost complete loss of external rotation. Radiographs of the shoulder are normal, as is her erythrocyte sedimentation rate. Which one of the following is the most likely diagnosis?

Frozen shoulder. Frozen shoulder= between the ages of 40 and 60. DM is the most common risk factor for frozen shoulder. Symptoms include shoulder stiffness, loss of active and passive shoulder rotation, and severe pain, including night pain. Laboratory tests and plain films are normal; the diagnosis is clinical (SOR C). Frozen shoulder is differentiated from chronic posterior shoulder dislocation and osteoarthritis on the basis of radiologic findings. Both shoulder dislocation and osteoarthritis have characteristic plain film findings. A patient with a rotator cuff tear will have normal passive range of motion. Impingement syndrome does not affect passive range of motion, but there will be pain with elevation of the shoulder.

common monomicrobial cause of type II necrotizing skin infections, which are often referred to as necrotizing fasciitis and warrant immediate attention.

GAS.

most common cause of chronic cough?

GERD.

herbal supplement Kava associated with?

GI SE and skin rash.

Supraclavicular lymph node is associated with?

GI system malignancy.

An elderly male patient takes aspirin, 81 mg daily, for prevention of a heart attack. He also takes herbal supplements. Which one of the following supplements can have a negative interaction with aspirin?

Ginkgo biloba: associated with serious intracerebral bleeding especially when concurrent anticoagulants are being used.

patient with known exposure to hepatitis B (pregnant) tx?

HBIG and hepatitis B vaccine.

hemolysis, elevated liver enzymes, and low platelets in pregnant woman should lead to what?

HELLP syndrome= induction of labor with oxytocin if the cervix is favorable.

What should be monitoring with testosterone replacement therapy?

Hct.

A 32-year-old male smoker presents with a 4-day history of progressive hoarseness. He is almost unable to speak, and associated symptoms include a cough slightly productive of yellow sputum, as well as tenderness over the ethmoid sinuses. He is afebrile and has normal ear and lung examinations. His oropharynx is slightly red with no exudate, and examination of his nasal passages reveals mucosal congestion. Which one of the following would be the most appropriate treatment?

He has acute laryngitis that should be managed with symptomatic tx. --nothing for acute laryngitis but for chronic you can give tx for acid reflux.

Which one of the following is the best INITIAL management for hypercalcemic crisis? A. Intravenous furosemide B. Intravenous pamidronate (Aredia) C. Intravenous plicamycin (Mithramycin) D. Intravenous saline

IV saline.

Some medications that cause acute interstitial nephritis ARF?

Ibuprofen (NSAID). Penicillin/ cephalosporin. Loops/ diuretics. Sulfonamides. aminoglycosides. rifampin.

what can effect HbA1c?

Hemoglobin variants and iron deficiency, kidney failure, and liver disease can also affect HbA1c results. and hemolytic anemia.

An obese, hypertensive 53-year-old physician suffers a cardiac arrest while making rounds. He is resuscitated after 15 minutes of CPR, but remains comatose. Which one of the following is associated with the lowest likelihood of neurologic recovery in this situation?

Myoclonic status epilepticus at 24 hrs= no possibility of recovery.

which of the ethnic groups in the US is at greatest risk for complications from influenza?

Native american.

A 55-year-old female has severe symptoms of gastroesophageal reflux disease. Upper endoscopy with a biopsy shows severe esophagitis and Barrett's esophagus. Which one of the following is true regarding this patient? (check one) A. The severity of her symptoms is due to the presence of Barrett's esophagus B. Follow-up screening endoscopy will reduce her risk of death from esophageal cancer C. Her risk of developing esophageal adenocarcinoma is >90% D. Her risk of developing esophageal adenocarcinoma is <1%

Her risk of developing esophageal adenocarcinoma <1%.

56-year-old female comes in for evaluation of gradually worsening right hip pain. She describes her pain as located in the groin and dull in nature, and with activity often notes a clicking sensation associated with sharp pain. On examination her hip range of motion is intact but pain is elicited with extremes of internal and external rotation and her groin pain is exacerbated with the FABER test (knee flexion, abduction and external rotation of the leg until the ankle rests proximal to the contralateral knee) and FADIR test (knee flexion, adduction, and internal rotation of the leg). Which one of the following is the most likely diagnosis?

Hip Labral Tear= causes dull or sharp groin pain, which in some patients radiates to the lateral hip, anterior thigh, or buttock. The pain usually has an insidious onset, but occasionally begins acutely after a traumatic event. Half of patients also have mechanical symptoms, such as catching or painful clicking with activity. FADIR and FABER tests are effective for detecting intra-articular pathology good sensitivity but poor specificity. Dx with MRI.

A 40-year-old male respiratory therapist presents for a health examination prior to hospital employment. His history indicates that as a child he lived on a farm in Iowa. His examination is unremarkable, but a chest radiograph shows that both lung fields have BB-sized calcifications in a miliary pattern. No other findings are noted. A PPD skin test is negative. The findings in this patient are most likely a result of

Histoplasmosis. (midwestern US). --exposure to bird/ bat poop. tx not usually needed.

Eye findings for SS?

Horizontal movements of the eye.

Greatest rf for developing T2D? A. age B. obesity C. history of a completed pregnancy D. history of gestational diabetes E. family history of diabetes

Hx of GD. even more than family hx and obesity.

Which one of the following medications has been shown to reduce bone loss and may reduce her risk of future hip fractures? (check one) A. Amlodipine (Norvasc) B. Hydrochlorothiazide C. Lisinopril (Prinivil, Zestril) D. Losartan (Cozaar) E. Metoprolol (Lopressor, Toprol-XL)

Hydrochlorothiazide.

A 45-year-old male presents with shortness of breath and a cough. On pulmonary function testing his FVC is <80% of predicted, his FEV1/FVC is 90% of predicted, and there is no improvement with bronchodilator use. The diffusing capacity of the lung for carbon monoxide (DLCO) is also low. Based on these results, which one of the following is most likely to be the cause of this patient's problem?

Idiopathic pulmonary fibrosis.

for enlarged lymph node when do you do something? what size does the lymph node need to be?

If 3 or less then Monitoring clinically for 4-6 weeks, then a biopsy if the node persists or enlarges. If >3 cm immediate biopsy if no inflammatory symptoms, if the node is in the supraclavicular area, or if the patient has coexistent constitutional symptoms such as night sweats or weight loss.

AD PCKD most common manifestation?

Hypertension. --worsening RF and increased risk for CV disease and death. first line tx for this: ACE inhibitors.

Circumflex occlusion change in leads: Left anterior descending occlusion change in: Right coronary occlusion causes changes in:

I, AVF and possibly V5,6. V1 to V6. II, III, avF.

screening test for TB preferred for a patient with the TB vaccine?

IFN-gamma.

A 47-year-old male is hospitalized for severe lower-extremity methicillin-resistant Staphylococcus aureus (MRSA) cellulitis. He is started on intravenous vancomycin and his home medications, which include metoprolol and escitalopram (Lexapro), are continued. On day 3, in preparation for discharge, he is transitioned to oral trimethoprim/sulfamethoxazole (Bactrim). Two hours after taking his first dose he reports severe swelling of his lips, wheezing, hoarseness, and hives. His blood pressure, which was previously normal, is now 84/62 mm Hg. You order emergent therapy with intramuscular epinephrine, 0.3 mg; intravenous methylprednisolone sodium succinate (Solu-Medrol), 125 mg; and intravenous diphenhydramine (Benadryl), 50 mg. However, no clinical improvement is noted after 15 minutes. Which one of the following should you recommend now?

IM glucagon. because patient is taking B-blocker and can be resistant to treatment with epinephrine. should be administered to anaphylactic patients on p-blockers when their response to epinephrine is either poor or absent.

A 2-year-old female is brought to the emergency department with a 2-day history of fever and increasing redness on the left forearm. She is otherwise healthy. On examination her temperature is 39.9°C (103.8°F), pulse rate 140 beats/min, and respiratory rate 42/min. She is irritable, and the left forearm has a 4-cm erythematous, warm, tender area, with a fluctuant area centrally. Her WBC count is 21,000/mm3 (N 4300-1 3 0,800), with 14% immature bands. In addition to incision and drainage, which one of the following is the best initial treatment in this patient?

IV Vancomycin. Community acquired MRSA should be considred the cause of the infection until definitive cultures are obtained. So incision and drainage and then if severe= IV Vancomycin.

If patient with known adrenal insufficiency secondary to hypopit is undergoing period of stress what do you do?

IV corticosteroids.

tx for endocarditis with gram + coccus until sensitivities are figured out? if prosthetic valve: If no prosthetic valve:

IV coverage for enterococcus, streptococcus, and MS and MR SA. if prosthetic valve: - vancomycin and gentamicin. Enterococcus and methicillin-resistant Staphylococcus are often resistant to cephalosporins. If the organism proves to be Staphylococcus sensitive to nafcillin, the patient can be switched to a regimen of nafcillin and gentamicin.

best initial diagnosis for cat scratch disease?

IgG testing for bartonella henselae.

A school nurse discovers head lice on a fourth-grade student. When should the student be permitted to return to class?

Immediately; remain in class but be discouraged from close contact with others until treated appropriately with a pediculicide.

A 35-year-old male amateur rugby player seeks your advice because right hip pain of several months' duration has progressed to the point of interfering with his athletic performance. The pain is accentuated when he transitions from a seated to a standing position, and especially when he pivots on the hip while running, but he cannot recall any significant trauma to the area and finds no relief with over-the-counter analgesics. On examination his gait is stable. The affected hip appears normal and is neither tender to palpation nor excessively warm to touch. Although he has a full range of passive motion, obvious discomfort is evident with internal rotation of the flexed and adducted right hip. Which one of the following is most strongly suggested by this clinical picture?

Impingement. Gradually worsening anterolateral hip joint pain that is sharply accentuated when pivoting laterally on the affected hip or moving from a seated to a standing position is consistent with femoroacetabular impingement. if you can reproduce pain with flexion adduction, and internal rotation then sensitive test +.

tmj disorder in a large number of adults but usually includes dental malocclusion, bruxism (teeth grinding), anxiety, stress disorders, and, rarely, rheumatoid arthritis. tx?

In fact, the majority of patients with TMJ disorders have spontaneous resolution of symptoms, so noninvasive symptomatic treatments and tincture of time are the best approach for most.

A new drug treatment is shown to reduce the incidence of a complication of a disease by 50%. If the usual incidence of this complication were 1% per year, how many patients with this disease would have to be treated with this medication for 1 year to prevent one occurrence of this complication?

In the case in question, the absolute risk reduction is 0.5% (0.5×.01). Thus, the number-needed-to-treat for the example cited is 200 (100/0.5).

how can you reduce reoccurance of diverticulitis?

Increased intake of dietary fiber and increased exercise and weight loss if BMI is 30+.

if you use inhaled corticosteroids in COPD what are you at risk of? A. increase cataract formation B. increase the incidence of fracture C. increase the risk of pneumonia D. slow the progression of the disease E. improve overall mortality from the disease

Increased the risk of pneumonia. FEV1<60%. reduce the number of exacerbations and the rate of decline in the quality of lifee

Who can get a second dose of pneumococcal polysaccharide vaccine in children?

Individuals with sickle cell disease, those with anatomic or functional asplenia, immunocompromised persons with renal failure or leukemia, and HIV-infected persons should receive polysaccharide vaccine on this same schedule and should also be revaccinated at least 3 years after the first dose. Otherwise for the rest it is one: chronic illness, diabetes mellitus, cerebrospinal fluid leaks, chronic bronchopulmonary dysplasia, cyanotic congenital heart disease, cochlear implants

Concussion. what do you do?

Initial complete cognitive and physical rest followed by an individualized graded return to activity. --complete physical and cognitive rest required for 1-2 days.

A 47-year-old female presents to your office with a complaint of hair loss. On examination she has a localized 2-cm round area of complete hair loss on the top of her scalp. Further studies do not reveal an underlying metabolic or infectious disorder. Which one of the following is the most appropriate initial treatment?

Intralesional triamcinolone (glucocorticoid). patient has alopecia areata= localized autoimmune reaction to hair follicles. --spontaneous recovery in 6-12 months but areas of regrowth may be affected differently. If the alopeica areata is very severe then you can use topical immunotherapy (most effective) but severe SE.

A 72-year-old previously healthy male presents with a 3-week history of mild, intermittent chest pressure that occurs when he walks up a steep hill. Which one of the following EKG abnormalities would dictate the use of a pharmacologic stress test as opposed to an exercise stress test? (check one) A. First degree atrioventricular block B. Left bundle branch block C. Poor R-wave progression in leads V1 through V3 D. Q-waves in the inferior leads E. Ventricular trigeminy

Left bundle branch block. EKG uninterpretable during an exercise stress test, and can also interfere with nuclear imaging performed during the test.

Tx for rhabomyolysis?

Isotonic saline.

where is behcet's syndrome most common?

Japan, Korea and Eastern mediterranean. YA. 2/3 with occular manifestations will progress to blindness.

preferred anticoagulant for venous thrombosis during pregnancy?

LMWH= less monitoring required compared to unfractionated and less risk of HIT.

A 50-year-old male presents to your office with a 1-hour history of an intense retro-orbital headache. started while he was jogging and eased when he stopped, but has persisted along with some pain in his neck. Other than a blood pressure of 165/100 mm Hg, his examination is unremarkable. Noncontrast CT of the head is also unremarkable. His pain has persisted after 2 hours in the emergency department. Which one of the following would be most appropriate at this time?

LP. You want a noncontrast CT and then LP to check for xanthochromia. This is to r/o subarachnoid hemorrhage.

PCOS first line treatment?

Lifestyle modifications and then metformin to help. metformin does not improve hirsutism or acne. --low dose OCP used to reduce the risk of endometrial cancer in patients with chronic anovulation.

A 13-year-old male presents with a 3-week history of left lower thigh and knee pain. There is no history of a specific injury, and his past medical history is negative. He has had no fevers, night sweats, or weight loss, and the pain does not awaken him at night. He tried out for the basketball team but had to quit because of the pain, which was worse when he tried to run. Which one of the following physical examination findings would be pathognomonic for slipped capital femoral epiphysis?

Limited internal rotation of the flexed hip. There is pain with physical activity, most commonly in the upper thigh anteriorly, but one-third of patients present with referred lower thigh or knee pain, which can make accurate and timely diagnosis more difficult. Hip extension and abduction are also limited in SCFE, but these findings are nonspecific.

An 88-year-old male nursing-home patient is having problems with constant overflow incontinence. Intermittent catheterization has proven difficult due to urethral obstruction and his resistance to such procedures. He has dementia and generalized weakness as a result of multiple strokes and is bedbound, requiring total care for most activities of daily living. Examination shows a grade 3 coccygeal ulcer that has been present for several months, and a digital rectal examination demonstrates a large, irregular prostate. Which one of the following is the best choice to quickly correct his incontinence?

Long term indwelling Foley catheter placement. -provides immediate relief of this patient's urinary retention.

Which one of the following should be given intravenously in the initial treatment of status epilepticus?

Lorazepam (Ativan).

RF for latent to active TB?

Lung cancer. DM. Alcoholism. Recent contact with active TB person. any condition treated with immunosuppressive therapy, and lung parenchymal diseases such as COPD, silicosis, or lung cancer.

CD4<50 prophylaxis for?

MAC with azithromycin.

A PPD may be falsely negative if administered 2-30 days after what vaccine?

MMR.

With rapidly progressing symptoms of paraparesis and sensory abnormalities in a diabetic patient, an epidural abscess is likely. neurologic emergency. tx?

MRI of the thoracic spine, which is the level of her deficits, is the best next step in the evaluation of this patient. Intravenous antibiotics are a necessary treatment, along with surgical debridement, if the MRI demonstrates the underlying problem.

if ankle brachial index is >1.4 then what do you do?

MRI or CT arteriography, duplex scanning and hemodynamic localization are noninvasive methods for lesion localizing and help when symptoms or findings do not correlated with ABI.

... is now considered the imaging study of choice when osteomyelitis is suspected;

MRI.

Which one of the following is the most accurate imaging study for assessing early osteomyelitis?

MRI= The 90% sensitivity and 80% specificity of MRI is superior to all other imaging modalities.

The most common identifiable cause of skin and soft-tissue infections presenting to metropolitan emergency departments is:

MRSA.

most common cause of unintentional deaths in children?

MVA.

A 50-year-old female sees you for follow-up of uncontrolled hypertension. Her recent blood pressure measurements average >175/105 mm Hg. The patient has diabetes mellitus and a BMI of 32.3 kg/m2. Physical findings are otherwise noncontributory. Recent laboratory studies include three different potassium levels <3.5 mEq/L (N 3.5-5.0) despite increasing dosages of oral potassium supplements, with the dosage now at 100 mEq daily. Which one of the following would be most appropriate at this point?

Measure peripheral aldosterone concentration and peripheral renin activity. PAC >15And PAC/PRA>20 suggests adrenal cause.

recent increase in occupational or recreational activities. Patients also often report weakened grip strength. The point of maximal tenderness is 5-10 mm distal to and anterior to the medial epicondyle. It is most often a tendinopathy of the flexor carpi radialis and the pronator teres.

Medial epicondylitis.

Your patient is moving to another state and requests transfer of his medical records. Which one of the following is true regarding this patient's request? (check one) A. The medical record should be released only with written permission from a patient or legal representative B. Although it is kept by the physician, the physical paper or electronic medical record is the property of the patient C. A physician may withhold medical record information that could cause undue stress to a patient D. In spite of a patient request, the physician may withhold information from a third party E. A physician has the right to withhold the medical record until medical bills are paid in full

Medical records should be released only with permission in writing. Although the actual medical record is the property of the physician, the information in the chart is the property of the patient.

A 67-year-old female has started receiving home hospice care. Her attending physician can bill through which one of the following?

Medicare Part B. As long as the attending physician is not employed by hospice, medicare Part B can be billed. Part A: inpatient care in hospitals and skilled nursing facilities, hospice, and home health services, but not custodial or long-term care. Part B: outpatient physician services, including office visits and home health services. Part C: private companies, and combines Part A and Part B coverage. Part D: prescription drug coverage.

What can cause CSF rhinorrhea cause?

Meningitis or other infections by serving as a pathway for bacteria.

A 68-year-old female presents with a 2-month history of painful, swollen wrists and knees. The pain is always present and is accompanied by stiffness in these joints for 2-3 hours every morning. Her past medical history, family history, and social history are unremarkable. She takes a daily multivitamin. A complete physical examination is notable only for symmetric, moderately swollen, slightly erythematous, and very tender wrists and knees. Range of motion is intact but increases her pain. Plain radiographs of these joints show erosions at the ulnar styloids. Lyme disease serologies are negative. Anti-cyclic citrullinated peptide (CCP) antibody testing is positive. Which one of the following would be appropriate for this patient as a sole therapy for her joint condition?

Methotrexate= RA disease modifying drug.

Acute flare up of MS tx?

Methylprednisolone (oral or parenterally). -- INF b, glatiramer, and natilizumab may decrease the frequency of exacerbations but not first choice for tx of acute flare up.

A 45-year-old female presents with a rash on the central portion of her face. She states that she has intermittent flushing and intense erythema that feels as if her face is stinging. She has noticed that her symptoms can be worsened by sun exposure, emotional stress, alcohol, or eating spicy foods. She has been in good health and has taken conjugated estrogens (Premarin), 0.625 mg daily, since a hysterectomy for benign reasons. A general examination is normal except for erythema of the cheeks and chin. No pustules or comedone formation is noted around her eyes, but telangiectasias are present. Which one of the following would be appropriate in the management of this problem?

Metronidazole gel; patient has Rosacea= common in women 30-60 yo. Central facial erythema and telangiectasias are prominent early features that may progress to a chronic infiltrate with papules and sometimes sterile pustules. Facial edema; rhinophyma due to hypertrophy of the subcutaneous glands of the nose. The usual= central facial erythema and flushing that many patients find socially embarrassing. Flushing can be triggered by food, environmental, chemical, or emotional triggers. Ocular problems occur in half of patients with rosacea, often in the form of an intermittent inflammatory conjunctivitis with or without blepharitis. tx: sunscreen, metronidazole, doxycycline, or tetracycline also can be used, especially if there are ocular symptoms. These are often ineffective for the flushing, so low-dose clonidine or a nonselective β-blocker may be added.

a contraindication to the use of combined hormonal contraceptives?

Migraine headaches with aura.

Daily symptoms, nighttime awakenings greater than once weekly but not nightly, daily use of a short-acting β-agonist, some limitation to daily activity, FEV1 >60% but <80% of predicted, and FEV1/FVC ratio reduced by 5%

Moderate persistent.

Using albuterol tx 3 x a week and then 2x a night waking up due to symptoms tx?

Moderately persistent asthma; add inhaled medium dose corticosteroids on top on PRN albuterol.

for lesions of basal cell carcinoma involving the nose what needs to be done? Especially if >11mm?

Mohs micrographic surgery is the preferred treatment. This is a microscopically controlled technique that facilitates removal of the entire lesion with the least amount of tissue removed.

... is the best first choice for chronic potent opioid therapy (SOR B). It is reliable and inexpensive, and equivalent doses can be easily calculated if the patient must later be switched to another medication.

Morphine= reliable and inexpensive and equivalent doses that can be easily calculated if the patient must later be switched to another medication.

Sepsis vasopressor of choice?

NE.

A 65-year-old female develops gram-negative septicemia from a urinary tract infection. Despite the use of fluid resuscitation she remains hypotensive, with a mean arterial pressure of 50 mm Hg. Which one of the following would be the most appropriate treatment for this patient?

NE; more effective than dopamine.

Maximum 24 hour dose of morphine sulfate that you can safely titrate in order to relieve this patient's pain?

NO limit.

accumulation of toxic metabolite of morphine in patient with renal insufficiency causes what?

Neuroexcitatory effects, including confusion, sedation, respiratory depression, and myoclonus.

First line tx for primary dysmenorrhea=

NSAIDs.

A 50-year-old female presents to your office for evaluation of a 2-month history of dyspnea on exertion and a nonproductive cough. She has a previous history of hypertension, overactive bladder, gastroesophageal reflux disease, and recurrent urinary tract infections. Vital signs are unremarkable and she has an oxygen saturation of 94%. She has inspiratory crackles in the posterior lung bases that do not clear with coughing. Office spirometry shows that the FVC is only 80% of normal, but the FEV1/FVC ratio is 0.85. Which one of the patient's current medications is most likely to be the cause of her problem?

Nitrofurantoin also could be amiodarone. can induce pneumonitis. Interstitial lung disease= chronic dyspnea. accompanied with nonproductive chronic cough. FEV1/FVC < 0.7= obstructive. >0.7 restrictive.

Which one of the following has good evidence of effectively improving borderline personality disorder?

No currently available pharmacotherapy dbt.

Trivial mitral regurgitation. Which one of the following is the recommended follow-up for this patient if he remains asymptomatic?

No repeat echocardiography.

When should you get imagining for back pain?

Nonspecific back pain does not require imaging (SOR B). An initial plain film would be appropriate if there were a history of recent significant trauma, or even a history of minor trauma in an elderly patient. Immediate MRI would be appropriate in the presence of other red flags such as bladder dysfunction, areflexia, saddle anesthesia, progressive motor weakness, a history of cancer, or the presence of fever, unexplained weight loss, or night sweats.

acute pancreatitis recommended initial imaging is?

acute pancreatitis.

What is a sports hernia?

Not true; tearing of tissue fibers.

A 5-year-old white male is brought to your office with a chief complaint of chronic nocturnal limb pain. His mother states that his pain is often severe enough that it awakens him at night and she often gives him ibuprofen to help alleviate his calf pain, but she has never seen him limp or heard him complain of pain during the day. She also has not noticed any grossly swollen joints, fever, rash, or weight change. She is concerned because of a family history of juvenile rheumatoid arthritis in a distant cousin. The physical examination is within normal limits, as are a CBC and an erythrocyte sedimentation rate. Most appropriate next step?

Nothing. This is growing pains. Occur in the thigh, calf or shin in up to 35% of 4-6 year olds and can continue to 19. Nocturnal without any limping or signs of inflammatory process. ESR and CBC normal.

If you get a cut <5 years since your past tetanus shot what do you do? If it has been >5 years what do you do?

Nothing; give Tdap. TIG= in addition to tetanus vaccine for wounds that are tetanus-prone due to contamination and tissue damage in person with uncertain primary vaccine hx. TT (toxoid)= indicated only when diphtheria component is contraindicated. if it has been >5 years then they would get a Tdap unless previous booster was Tdap.

6 yo with OSA tx?

adenotonsillectomy.

A 23-year-old female presents with menstrual irregularity, increased facial hair, and acne. Your evaluation leads to a diagnosis of polycystic ovary syndrome. Which one of the following is the first-line management for her constellation of symptoms? (check one) A. Clomiphene (Clomid) B. Hormonal contraceptives C. Metformin (Glucophage) D. Pioglitazone (Actos) E. Spironolactone (Aldactone)

OCP.

Name some common drugs that cause galactorrhea?

OCP; risperidone. but not >200.

which one of the following is most likely associated with resistant htn in adults? A. Obstructive sleep apnea B. Primary aldosteronism C. Renal artery stenosis D. Renal parenchymal disease E. Thyroid disease

OSA= 60-70% of resistant htn due to OSA.

Significant RS for esophageal adenocarcinoma?

Obesity,, GERD.

A 3-year-old male was treated for acute otitis media last month. His mother brings him in for follow-up because she believes his hearing has not been normal since then. He attends day care and has had several upper respiratory infections. On examination the tympanic membranes are not inflamed, but the membrane is retracted on the right side. An office tympanogram shows a normal peak (type A) on the left side, but a flat tracing (type B) on the right side. Which one of the following would be the most appropriate recommendation?

Observation with follow-up . unilateral serous otitis and is unlikely to have delayed language from decreased hearing on one side.

hydocele in 4 week old tx?

Observe: A hydrocele of the tunica vaginalis testis occurs frequently at birth but usually resolves in a few weeks or months. No treatment is indicated during the first year of life unless there is a clinically evident hernia.

uveitis presenting as an extra articular manifestation of ankylosing spondylitis tx?

Ocular corticosteroids. Oral or parenteral corticosteroids and NSAIDs are also effective.

children with bacterial conjunctivitis should be allowed to remain in school once?

Once therapy is initiated.

Dyspnea in a terminally ill patient with absence of hypoxia. mainstay tx?

Opiates= mainstay of symptomatic tx.

A 58-year-old male with COPD presents with a 5-day history of increased dyspnea and purulent sputum production. He is afebrile. His respiratory rate is 24/min, heart rate 90 beats/min, blood pressure 140/80 mm Hg, and oxygen saturation 90% on room air. Breath sounds are equal, and diffuse bilateral rhonchi are noted. He is currently using albuterol/ipratropium by metered-dose inhaler three times daily. In addition to antibiotics, which one of the following would be most appropriate for treating this exacerbation?

Oral Prednisone for 5 days= -- nothing serious at this time so tx outpatient. already on a reasonable inhaled bronchodilator/ anticholinergic dose tx with oral antibiotics and corticoteroids. Oral corticosteroid therapy initiated early in a COPD exacerbation reduces the rate of treatment failure, decreases hospitalization rates, improves hypoxia and pulmonary function, and shortens the length of stay for patients requiring hospitalization. Short courses of oral corticosteroids (5-7 days) are as effective as longer ones

A 45-year-old male is seen in the emergency department with a 2-hour history of substernal chest pain. An EKG shows an ST-segment elevation of 0.3 mV in leads V4-V6. In addition to evaluation for reperfusion therapy, which one of the following would be appropriate?

Oral clopidogrel (Plavix)= begin fibrinolysis within <30 minutes after first contact with health system. enteric aspirin is delayed. b-blocks not routine and warfarin not indicated.

A 65-year-old male who has been in good health presents to your office with a 2-day history of a sensation of pressure and hearing loss in his left ear. A physical examination and a thorough neurologic examination are both unremarkable. Both tympanic membranes are normal. An audiogram shows a 30-decibel hearing loss at three consecutive frequencies in the left ear, with normal hearing on the right. Placing a vibrating tuning fork in the midline of the forehead reveals sound lateralizing to the right ear. Which one of the following would be most appropriate at this point?

Oral corticosteroids may be offered as initial therapy, and hyperbaric oxygen therapy may be helpful within 3 months of diagnosis. The guidelines also strongly recommend against routine laboratory tests or CT of the head as part of the initial evaluation.

Patients with depleted iron stores will benefit from replenishment with what?

Oral ferrous iron. They do not need the EPO level checked if ferritin is present.

Toe nail fungus fast tx?

Oral terbinafine daily for 12 weeks= highest cure rate and best long term therapy. --topical creams are not appropriate because the fungus is inside the cells.

The most common cause of proteinuria in children is:

Orthostatic proteinuria.

An 86-year-old mildly demented male nursing-home resident rarely leaves the facility. He has frequent fecal incontinence that is disturbing to both him and his family. He has diet-controlled diabetes mellitus and hypertension, and a history of transurethral resection of the prostate (TURP) for benign prostatic hypertrophy. An examination is remarkable only for an empty rectum and no focal neurologic findings. Most likely cause of this patient's fecal incontinence?

Overflow incontinence due to constipating medication. Reduced storage capacity = inflammatory bowel disease. Mild diabetes mellitus does not cause decreased rectal sensation, and puborectalis and internal sphincter weakness are uncommon in males, as they usually result from vaginal delivery.

cluster tx abortive therapy=

Oxygen and sumatriptan are first-line abortive therapies.

early evaluation of patients with chest pain what is a good test?

PA and lateral chest radiograph. do not confirm or rule out the presence of myocardial ischemia, other causes of chest pain may be evident, such as pneumothorax, pneumonia, or heart failure. And clues about PE, aortic disease or neoplasia.

leading cause of death post bariatric surgery?

PE.

before starting a TNF inhibitor what test needs to be performed?

PPD test. induration 5mm+ should be considered + and assess whether tx for latent tb is necessary prior to tfn inhibitor use.

A patient is sent to you by his employer after falling down some steps and twisting his ankle and foot. Which one of the following would be the most appropriate reason to obtain foot or ankle radiographs? (check one) A. Notable swelling and discoloration over the anterior talofibular ligament B. A complaint of marked pain with weight bearing as he walks into the examining room C. Pain in the maleolar zone and bone tenderness of the posterior medial malleolus D. The absence of passive plantar foot flexion when the calf is squeezed (Thompson test)

Pain the maleolar zone and bone tenderness of the posterior medial malleolus. Not be because they can still walk into the examining room. They should have inability to bear weight for it to count.

A 42-year-old male presents with anterior neck pain. His thyroid gland is markedly tender on examination, but there is no overlying erythema. He also has a bilateral hand tremor. His erythrocyte sedimentation rate is 82 mm/hr (N 1-13) and his WBC count is 11,500/mm3 (N 4300-10,800). His free T4 is elevated, TSH is suppressed, and radioactive iodine uptake is abnormally low. Which one of the following treatment options would be most helpful at this time?

Painful subacute thyroiditis= painful thyriod + hyperthyroid + elevated ESR. --thyroid function returns back for most people or can be temporarily hypothyroid for a while. Tx= symptomatic. -- use high dose glucocorticoids for quick relief for the most severe symptoms.

A 30-year-old male complains of the gradual onset of anterior right knee pain on climbing the stairs. On examination there is no effusion, but there is tenderness over the medial retinaculum. There is good ligament strength, and range of motion is normal. When the knee is extended from 90° flexion to full extension, the patella deviates laterally. Which one of the following would be the best initial treatment for this condition? (check one) A. Bracing B. Taping C. NSAIDs D. Arthroscopic surgery E. Physical therapy

Patient has patellofemoral stress syndrome= runner's knee or anterior knee pain. best tx= PT.

What are the Ottawa Ankle Rules?

Patients who were able to bear weight immediately following their injury and who can take 4 steps independently in a clinical setting require radiographic study only when the following criteria are met: 1. pain is present in the malleolar zone and bony tenderness of the posterior edge or tip of either malleolus is elicited (ankle radiograph) 2. pain is present in the midfoot zone and bony tenderness of either the base of the fifth metatarsal or the navicular region is present.

A 44-year-old female is distressed because of incontinence. She reports frequent episodes of an immediate need to urinate, which cannot always be deferred. She admits to urinating more than 10 times a day, but denies any urine leakage with coughing, laughing, or straining. Which one of the following is the most appropriate initial treatment for this patient?

Pelvic floor muscle training and bladder training= urge incontinence or overactive bladder, as well as in stress and mixed incontinence.

The diagnosis should be considered when a patient has fever and persistence of flank pain. patient had a UTI 2 weeks ago txed and now spiking fevers. The diagnosis is?

Perinephric abscess that is usually an extension of an ascending urinary obstruction. -higher risk if with anatomical abnormalities or DM

A 52-year-old male has a skin lesion removed from his arm with appropriate sterile precautions. Which one of the following would be most appropriate to use on this surgical wound?

Petrolatum. topical antibiotics have not been shown to reduce the rate of infection in clean surgical wounds compared to the use of non antibiotic ointment or no ointment. Topical antibiotics can aggravate open wounds, hinder the normal wound healing process.

Diabetic foot ulcer severe tx? --usually polymicrobial.

Piperacillin/ tazobactam and vancomycin. -- if patient has systemic inflammatory signs= IV antibiotics.

how does plantar fascititis present in a runner?

Plantar fasciitis is the most common cause of heel pain in runners and often presents with pain at the beginning of the workout. The pain decreases during running only to recur afterward. pain at the plantar surface.

fever, chills, cough, and pleuritic chest pain. A sudden onset of severe hypoxia is less common. Radiologic findings typically include lobar infiltrates or bronchopneumonia (with a segmental pattern of infiltrate), whereas diffuse bilateral infiltrates are much less common. cause?

Pneumococcal pneumonia.

acute onset of hypoxia and respiratory failure, associated with a dry cough and fever. Characteristic radiographic findings include diffuse bilateral interstitial infiltrates. patient also has immunocompromised. what is the cause?

Pneumocystitis pneumonia.

patient with repeated ekgs showing QTc >489 ms with syncopal episode or >500 ms in the absence of symptoms diagnosed with long QT if no secondary causes such as medication are present. associated with?

Polymorphic ventricular tachycardia, torsades de pointes, and sudden cardiac death.

A 44-year-old African-American female reports diffuse aching, especially in her upper legs and shoulders. The aching has increased, and she now has trouble going up and down stairs because of weakness. She has no visual symptoms, and a neurologic examination is normal except for proximal muscle weakness. Laboratory tests reveal elevated levels of serum creatine kinase and aldolase. Her symptoms improve significantly when she is treated with corticosteroids. Diagnosis? Tx?

Polymyositis. Corticosteroids.

A 4-year-old female has had three urinary tract infections in the past 6 months. She complains of difficulty with urination and on examination is noted to have labial adhesions that have resulted in near closing of the introitus. What is this?? Which one of the following is the most appropriate management?

Prepubertal labial adhesions= idiopathic. application of estrogen cream to the site 2x daily if symptomatic.= tx. common problems with this= a pulling sensation, difficulty with voiding, recurrent urinary tract infections, or vaginitis.

hearing loss associated with aging, is gradual in onset, bilateral, symmetric, and sensorineural.

Presbycusis.

Which one of the following is most likely to be seen with diastolic dysfunction?

Preserved EF; nondilated L ventricle with preseved EF and evidence of structural heart disease such as diastolic dysfunction on echocardiography.

Which one of the following is the most concerning early symptom of a dangerous drug reaction?

Pruritus around the mouth and on the palms of the hands and soles of the feet

A 52-year-old mechanic complains of an irritation in his right eye lasting for 2 days. On direct visualization you see a small, dark foreign body on the periphery of the cornea and are able to remove it with no complications. However, there is a patch of reddish-brown discoloration extending several millimeters around the area where the foreign body had been. Which one of the following is most appropriate for this patient?

Prompt ophthalmologic evaluation because it is a rust ring. --requires the use of slit lamp and specialized ophthalmic equipment.

Which one of the following is considered first-line therapy for migraine prophylaxis in adults? (check one) A. Gabapentin (Neurontin) B. Propranolol (Inderal) C. Fluoxetine (Prozac) D. Vitamin B2 (riboflavin) F. Naproxen (Naprosyn)

Propranolol. Other first-line agents include timolol, amitriptyline, divalproex sodium, sodium valproate, and topiramate. Second line to be used if no first line agents or combinations: Gabapentin, fluoxetine, vitamin B , and naproxen.

A 4-year-old male is brought to your office by his parents who are concerned that he is increasingly "knock-kneed." His uncle required leg braces as a child, and the parents are worried about long-term gait abnormalities. On examination, the patient's knees touch when he stands and there is a 15° valgus angle at the knee. He walks with a stable gait. Which one of the following should you do now? (check one)

Provide reassurance; physiologic genu valgus. Toddlers under 2 years of age typically have a varus angle at the knee (bowlegs). This transitions to physiologic genu valgus, which gradually normalizes by around 6 years of age. physiologic so no therapy needed.

A 23-year-old female presents with recurrent unprovoked epistaxis. The patient's mother is known to have hereditary hemorrhagic telangiectasia. Contrast echocardiography is recommended to screen for which one of the following frequently associated conditions?

Pulmonary ateriovenous malformation found in 15-30% of patients with hereditary hemorrhagic telangiectasia (osler weber rendu syndrome).

someone comes in with stroke. First thing you want to make sure to do when they come into the er?

Pulse ox. Not too high or too low bp. not too hypovolemic otherwise problem.s no heparin until r/o hemorrhagic.

Which one of the following medications is associated with a higher risk of death due to stroke or sudden cardiac death in patients with dementia? (check one) A. Diazepam (Valium) B. Fluoxetine (Prozac) C. Paroxetine (Paxil) D. Quetiapine (Seroquel) E. Venlafaxine

Quetiapine: black box warning about elderly patients with dementia, related in part to sudden cardiac death and also to stroke.

In early February, you receive a call from your office nurse. Her 5-month-old daughter has been ill for several days. What started as a mild upper respiratory infection has progressed and she now has profuse rhinorrhea, a temperature of 100.2° F (37.9° C), and audible wheezing. In spite of an almost nonstop cough, she does not appear acutely ill. The organism responsible for this child's illness is most likely to be:

RSV.

What virus is rarely associated with bacterial co-infection?

RSV; self limited.

first line therapy for all patients with recurrent SVT?

Radiofrequency ablation.

A 66-year-old male with known GOLD stage 3 COPD is admitted to the hospital with pneumonia. His pneumonia improves and he is discharged with home oxygen because of hypoxemia. He did not require home oxygen before this. Which one of the following would be most appropriate regarding his future use of home oxygen?

Reassess the need for oxygen within 3 months.

A 29-year-old female presents with redness of her left eye. She has just returned from a summer beach vacation with her children and woke up with a red eye. Your examination reveals a watery discharge, a hyperemic conjunctiva, and a palpable preauricular lymph node. Her cornea is clear on fluorescein staining. Which one of the following is most appropriate for this patient?

Reassurance only; viruses 80% of the cases and requires no tx. most caused by adenoviruses= pharyngeal conjunctival fever, epidemic keratoconjunctivitis. --pharyngeal conjunctivitis= high fever, pharyngitis, and bilateral eye inflammation. --high fever, pharyngitis, and bilateral eye inflammation.

Complications of hypoparathyroidism include?

Refractory HF. Classic symptoms are those of insufficient Ca. , tetany, seizures, altered mental status, and stridor.

A 68-year-old male with end-stage lung cancer is being treated for pain secondary to multiple visceral and skeletal metastases. He has been on oral ibuprofen and parenteral morphine. However, over the past few weeks he reports progressive worsening of his pain. In order to achieve better pain control his morphine dosage has been continuously titrated up. In spite of this increase he continues to report severe pain that is now diffuse and occurs even when his caregivers touch him. Which one of the following would be most appropriate at this time?

Reduce the morphine= opioid induced hyperalgesia= paradoxical increase in sensitivity to pain despite an increase in the opioid dosage.

A 78-year-old female presents with a red eye. She reports drainage and pain in her left eye since she woke up today, but no photophobia. Examination of the eye shows conjunctival erythema and a mucopurulent discharge. The pupil is normal in size and reactive to light. Which one of the following should prompt immediate referral to an ophthalmologist?

Reduction of visual acuity= symptom of acute angle closure glaucoma that requires immediate ophthamologist referral.

best course of tx for primary hyperparathyroidism?

Referral to surgeon for consideration of parathyroidectomy.

As part of routine care for a 31-year-old female you obtain a Papanicolaou (Pap) test for cervical cancer screening. The cytology results are normal, and the sample is positive for the presence of HPV but negative for serotypes 16 and 18. Which one of the following is the most appropriate management for this patient?

Repeat Pap and HPV testing in 1 year. If her cytology remains negative but her HPV test remains positive, she should be advised to have colposcopy at that time regardless of the serotype of the HPV. If her current test had shown evidence of either strain 16 or 18 immediate colposcopy would be indicated.

Effective tx for OCD?

Repetitive exposure to fearful stimuli= --most effective then come TCA and SSRI. Atypical antipsychotics do not tx the disorder itself.

The "Get Up and Go Test" evaluates for which one of the following?

Risk of falling: rise from a chair, walk 10 feet, turn, return to the chair, and sit down. also remember medication review for risk of falling.

Most common cause of otitis media?

S. pneumoniae penicillin resistant= also H influenza and Moraxella.

how do you assess mortality based on GI bleeding?

Rockall risk scoring= low risk if the following are evident. 1. age <60. 2. Systolic BP>100+. 3. HR <100/min. 4. No shock. 5. No major comorbidities. nothing for mallory weiss tear. ulcer is one poitn.

Peripheral arterial disease (PAD) reducing PAD symptoms and preventing death

Routine exercise. Smoking cessation and aspirin . Statin drugs. ACE inhibitors have been shown to reduce symptoms of PAD directly. atenolol and nifedipine has actually been shown to worsen symptoms of PAD.

Limited internal rotation of the flexed hip is indicated with?

SCFE= young adolescents during the growth spurt, when the femoral head is displaced posteriorly through the growth plate.

This patient has hypotonic hyponatremia, manifested by low serum osmolality. She is asymptomatic and has no signs of hypovolemia on her laboratory tests or physical examination. Her urine sodium is high and her urine osmolality is low, which indicates

SIADH REVIEW PLEASE.

Euvolemic patient with hyponatremia, decreased serum osmolality-

SIADH= likely due to SSRI, chlorpromaide, barbituates, carbamazepine, opiods, tolbutamide, vincristine, diuretics, and NSAIDs. tx= stop drug.

Pet reptile is most likely to transmit which of the following to human contracts?

Salmonella.

current methods for reducing the risk of renal failure induced by contrast material include

adequate hydration. N-acetylcysteine.

A 55-year-old overweight male presents with a complaint of pain in the left big toe. He recently started jogging 2 miles a day to try to lose weight, but has not changed his diet and says he drinks 4 cans of beer every night. The pain has developed gradually over the last 2 weeks and is worse after running. An examination shows a normal foot with tenderness and swelling of the medial plantar aspect of the left first metatarsophalangeal joint. Passive dorsiflexion of the toe causes pain in that area. Plantar flexion produces no discomfort, and no numbness can be appreciated. Which one of the following is the most likely diagnosis?

Sesamoid fracture. Pain involving the big toe is a common problem. The first metatarsophalangeal (MTP) joint has two sesamoid bones, and injuries to these bones account for 12% of big-toe injuries. Overuse, a sharp blow, and sudden dorsiflexion are the most common mechanisms of injury. Gout= first big toe would be sudden, warm, red, swelling and pain with movement. Morton's neuroma= numbness involving digital nerve in the area= nerve being pinched between metatarsal heads in the center of the foot.

What is SIRS?

Severe inflammatory response syndrome (fever >38.5, HR >90, RR>20, WBC>12000).

Symptoms throughout the day, nighttime awakenings nightly, short-acting β-agonist usage several times daily, extremely limited daily activities, FEV1 <60% of predicted, and FEV1/FVC ratio reduced by >5%.

Severe persistent.

When treating acute adult asthma in the emergency department, using a metered-dose inhaler (MDI) with a spacer has been shown to result in which one of the following, compared to use of a nebulizer?

Shorter stays in the emergency department. lower pulse rates provide greater improvement in peak-flow rates lead to greater improvement in arterial blood gases decrease required albuterol doses. lower costs shorten emergency department stays significantly lower relapse rates at 2 and 3 weeks compared to nebulizers. There is no difference in hospital admission rates.

An 8-year-old female is brought to your office because she has begun to limp. She has had a fever of 38.8°C (101.8°F) and says that it hurts to bear weight on her right leg. She has no history of trauma. On examination, she walks with an antalgic gait and hesitates to bear weight on the leg. Range of motion of the right hip is limited in all directions and is painful. Her sacroiliac joint is not tender, and the psoas sign is negative. Laboratory testing reveals an erythrocyte sedimentation rate of 55 mm/hr (N 0-10), a WBC count of 15,500/mm 3 (N 4500-13,500), and a C-reactiveprotein level of 2.5 mg/dL (N 0.5-1.0). Which one of the following will provide the most useful diagnostic information to further evaluate this patient's problem?

She has septic arthritis and you want to do a US to make sure there is not an effusion present. --if present, do an apsiration. leggs calve perthes can be identified with MRI.

keloid tx?

Silicone gel sheeting, topical imiquimod, and intralesional fluorouracil can be used when first-line therapy fails, but these methods are more often associated with recurrence. Laser therapy and surgical excision are associated with a high rate of recurrence when used as monotherapy.

A previously healthy 18-month-old male is brought to your office with a 2-day history of cough and fever. On examination the child has a temperature of 38.3°C (100.9°F), a respiratory rate of 30/min, and mild retractions and mild wheezes bilaterally. Oxygen saturation is 90%. The most appropriate initial management would be:

Single tx with aerosolized albuterol, continue only if there is a positive response. -patient has bronchiolitis. -tx= inhaled bronchodilator, albuterol or epinephrine with only continuation if initial dose proves beneficial.

what type of emergency situation should atropine be used for?

Sinus bradycardia with hypotension.

A 44-year-old female presents with a complaint of increasingly dry eyes over the past 3-4 months, and says she can no longer wear contacts due to the discomfort and itching. She also apologizes for chewing gum during the visit, explaining that it helps keep her mouth moist. On examination you note decreased tear production, decreased saliva production, and new dental caries. She stopped taking a daily over-the-counter allergy medication about 1 month ago. Which one of the following is the most likely diagnosis?

Sjogren's syndrome= systemic autoimmune disease. due to lymphocytic infiltration of exocrine glands and= acinar gland degeneration, necrosis, atrophy and decreased fx. +anti-SS-A or anti-SS-B or a positive salivary gland biopsy. In addition to ocular and oral complaints, clinical manifestations include arthralgias, thyroiditis, pulmonary disease, and GERD.

Which one of the following is a physiologic difference between males and females that can affect the pharmacokinetics of medications with a narrow therapeutic index?

Slower gastrointestinal transit times in women. which can diminish the absorption of medications such as metoprolol, theophylline, and verapamil. In addition, women should wait longer after eating before taking medications that should be administered on an empty stomach, such as ampicillin, captopril, levothyroxine, loratadine, and tetracycline. women secrete less gastric acid than men. lower BMI than men= smaller loading/ bolus dosages of medication. women have higher fat storages then men= so lipophilic drugs have longer duration of action. women= lower GFR than men= slower clerance of medication that are eliminated reinally.`

What volume overload drug can cause gynecomastia?

Spirinolactone. --Eplerenone= has greater specificity for mineralocorticoid receptos and less liekly to cause gynecomastia.

foot infection in diabetic superficial common pathogens?

Staphylococcus aureus and β-hemolytic streptococci (groups A, B, and others). if previously txed and deep= polymicrobial.

How should lamotrigine be administered to avoid SJS?

Start on a daily dose of 25 mg nd then titrate every 2 weeks until goal dosage reached.

A 68-year-old male was seen in a local urgent-care clinic 6 days ago for upper respiratory symptoms and was started on cefuroxime (Ceftin). He presents to your office with a 2-day history of 4-5 watery stools per day with no blood or mucus. He is afebrile and has a normal abdominal and rectal examination. A stool guaiac test is negative, and a stool sample is sent for further testing. What is the best initial management for this patient?

Stop cefuroxime. Risk of C. difficile. Treatment should not be initiated unless the stool is positive for toxins A and B.

If you get HIT what is your tx?

Stop heparin and start non heparin anticoagulant like Argatroban or desirudin.

what is the cause of erysipelas? what is a good tx?

Streptococcus pyogenes. --most involve the face but the lesions can occur anywhere on the body. Penicillin.

Asymptomatic adults with sustained BP >135/80 be screened for?

T2D using fasting plasma glucose, 2 hour glucose tolerance or A1c measurements. -no guideline for PAD.

If patient presents with cellulitis and is inpatient what is the initial drug of choice?

Vancomycin= MRSA is the most prominent cause of suppurative skin and soft tissue infection. --TMP/SMX good for outpatient.

If patient has CD4<200 then prophylaxis with what?

TMP/SMX for pneumocystitis.

who repetitively extend their elbows against resistance. Pain occurs at the posterior elbow with resisted extension, and tenderness is located over the triceps insertion.

Tendinopathy of the triceps insertion is more common in weight lifters or athletes

ottawa rules:

The Ottawa Rules state that radiography is required only if there is pain in the malleolar or midfoot zone and either bony tenderness over an area of potential fracture (i.e., distal fibula or tibia, lateral or medial malleolus, base of the fifth metatarsal, or navicular bone) or an inability to bear weight immediately after the injury and when evaluated by a physician.

The relative risk reduction (RRR) is the proportional decrease in disease incidence in the treated group relative to the incidence in the control group. In this example the 3% incidence in the treated group is 40% less than the 5% incidence in the control group: (5%-3%)/5% = 40%. The absolute risk reduction (ARR) is the difference between the incidence of disease in the treatment group and the incidence in the control group. In this example the ARR is 5% minus 3% = 2%.

The number needed to treat (NNT) equals the reciprocal of the ARR: 1/.02 = 50. The RRR is not a very useful statistic in clinical practice. It amplifies small differences and makes clinically insignificant findings appear significant because it essentially ignores the baseline risk of the outcome event. The ARR provides a more useful measure of clinical effect. It answers the question "How much will I decrease my patient's risk of an adverse outcome by this treatment?" The NNT is also very useful for clinicians, as it answers the question, "How many patients will I need to treat to prevent one adverse outcome?" Ref: Demaerschalk BM: Literature-searching strategies to improve the application of evidence-based clinical practice principles to stroke care.

An elevation of serum alkaline phosphatase combined with an elevation of 5'-nucleotidase is most suggestive of conditions affecting

The liver. Alkaline phosphatase can be high for things effecting the bone, placenta, small intestine and liver.

Management for an outbreak based on CDC?

chemoprophylaxis with appropriate medication for all residents who are asymptomatic and tx for all residents who are symptomatic regardless of laboratory confirmatino of infection or vaccine status.

a healthy asymptomatic patient ejection fraction?

The normal predicted value is 55%-75%.

dubin johhnson in high what?

conjugated.

A. Antacids B. Soy milk C. Iced tea D. Bran what do these things do to iron absorption?

decrease it.

Maximal heart rate with exercise generally ... with age.

decreases.

what allows for detecting right ventricular (RV) dysfunction and also allows for estimation of pulmonary artery pressure?

echocardiography.

A 57-year-old male presents to the emergency department complaining of dyspnea, cough, and pleuritic chest pain. A chest radiograph shows a large left-sided pleural effusion. Thoracentesis shows a pleural fluid protein to serum protein ratio of 0.7 and a pleural fluid LDH to serum LDH ratio of 0.8. Which one of the following causes of pleural effusion would be most consistent with these findings? exudative or transudative? and exampels of both.

The protein and lactate dehydrogenase (LDH) levels in pleural fluid can help differentiate between transudative and exudative effusions. Light's criteria (pleural fluid protein to serum protein ratio >0.5, pleural fluid LDH to serum LDH ratio >0.6, and/or pleural LDH >0.67 times the upper limit of normal for serum LDH) are 99.5% sensitive for diagnosing exudative effusions and differentiate exudative from transudative effusions in 93%-96% of cases. Exudative= PE. Transudative: cirrhosis, HF, nephrotic syndrome, SVC obstruction.

A 69-year-old female presents with postmenopausal bleeding. You consider whether to begin your evaluation with vaginal probe ultrasonography to assess the thickness of her endometrium. In evaluating the usefulness of this test to either support or exclude a diagnosis of endometrial cancer, which one of the following statistics is most useful?

The statistics that are clinically useful for evaluating diagnostic tests include the positive predictive value, negative predictive value, and likelihood ratios. Likelihood ratios indicate how a positive or negative test correlates with the likelihood of disease. In the example given, if the patient's endometrial stripe is >25 mm, the likelihood ratio is 15.2 and her post-test probability of endometrial cancer is 63%. However, if it is ≤ 4 mm, the likelihood ratio is 0.02 and her post-test probability of endometrial cancer is 0.2%.

a likelihood ratio of 1 means?

The test can neither confirm nor rule out pulmonary embolism.

ear infection for children 6 months or older include:

The usual treatment for AOM is amoxicillin, but an antibiotic with additional $-lactamase coverage, such as amoxicillin/clavulanate, should be given if the child has received amoxicillin within the past 30 days, has concurrent purulent conjunctivitis, or has a history of AOM unresponsive to amoxicillin (SOR C).

A 55-year-old white male notices a nodular thickening over the flexor tendons in his medial palm. He has no difficulty using his hand, and he is able to lay his palm flat on a tabletop. You suspect Dupuytren's disease. Which one of the following is true regarding this condition? A. There is a strong association with diabetes mellitus B. Surgical intervention is recommended at this point to prevent progression to contracture C. Once a contracture develops, it is irreversible and no treatment is indicated D. A single cortisone injection often leads to disease regression in mild to moderate cases E. A search for an occult malignancy is indicated

There is a strong association with DM, alcohol and smoking. shortening and thickening of palmar fascia. --early asymptomatic disease does not require tx. --series of cortisone injection over a period of a month= disease regression. --surgery if >30 degrees or with a proximal inter-phalangeal of any degree.

Centor criteria:

These include tonsillar exudates, tender anterior cervical lymphadenopathy, absence of cough, and history of fever. The presence of three or four of these criteria has a positive predictive value of 40%-60%, and the absence of three or four of these criteria has a negative predictive value of 80%.

what is the milk alkali syndrome?

hypercalcemia resulting from chronic overdose of calcium and vitamin D supplement. PTH in milk alkali and sarcoidosis should be suppressed.

During rounds, you notice a new rash on a full-term 2-day-old white female. It consists of 1-mm pustules surrounded by a flat area of erythema, and is located on the face, trunk, and upper arms. An examination is otherwise normal, and she does not appear ill. Which one of the following is the most likely diagnosis?

This infant has the typical "flea-bitten" rash of erythema toxicum neonatorum (ETN). Transient neonatal pustular melanosis is most common in African-American newborns, and the lesions lack the surrounding erythema typical of ETN. Acne neonatorum is associated with closed comedones, mostly on the face. As the infant described is not ill, infectious etiologies are unlikely.

in the midline over the hyoid bone. Frequently, they elevate when the patient swallows.

Thyroglossal duct cysts

... most common lower extremity stress fractures in both children and adults, accounting for about half of all stress fractures.

Tibial fractures are the.

electrolyte side effect of spirinolactone?

hyperkalemia.

A 17-year-old male high school football running back is hit on the lower leg by an opposing player's helmet when the other player dives for a fumble. The running back presents to the emergency department after the game with significant swelling and bruising of the lower leg. Symptoms include exceptionally severe pain that is worse with stretching the calf muscles. There is no weakness of the extremity and sensation is intact. You examine the leg and can palpate pulses. Plain radiographs do not show a fracture. Which one of the following should be ordered next?

Tissue pressure studies. MEDICAL EMERGENCY. acute compartment syndrome= after severe injury to extremity, although it can develop even after minor injury. associated problems= fractures, a badly bruised muscle, crush injuries, constricting bandages, and bites with swelling.

Tx for pressure ulcers?

Topical antibiotics should not be used for more than 2 weeks at a time. -nutritional supplement does not definitively show increased ulcer healing. -keep the head elevated <30 degrees to reduce sheering forces.

... is indicated for patients with poorly functioning gastrointestinal tracts who cannot tolerate other means of nutritional support and for those with high caloric requirements that cannot otherwise be met.

Total parenteral nutrition (TPN).

If CD4 <100 prophylaxis with?

Toxo.

universally recommended that pregnant women avoid direct contact with cats' litter boxes to avoid?

Toxo.

Which one of the pharmacologic effects of transdermal medications changes the LEAST with aging? A. Liver metabolism of the drugs B. Renal excretion of the drugs C. Distribution within the body D. Transdermal absorption of the drugs

Transdermal aging. everything else in order: volume of distribution increases, liver/ renal decrease with aging, and serum concentrations increase as a result.

tx for PID before chlamydia results come back?

Treat based on clinical findings.

a classic presentation of an osteoporotic vertebral compression fracture. The diagnosis should be confirmed with a plain radiograph.tx: short term; long term. pain relief?

Treatment is basically symptomatic, with a period of bed rest as short as possible (to avoid complications of immobility), pain medication, and bracing. Salmon calcitonin (injectable or intranasal) is often helpful in providing pain relief. Long-term management of underlying osteoporosis may help prevent future fractures. Subcutaneous or intranasal calcitonin (Calcimar, Miacalcin) may be very helpful for pain relief

Tx for plantar fascitis?

Treatment strategies include relative rest, ice, NSAIDs, and prefabricated shoe inserts that provide arch support, as well as heel cord and plantar fascia stretching. surgery for patients with refractory for 6-12 months of uninterrupted conservative therapy.

Long term medication assisted tx for narcotic addiction is more successfull than detox programs true or false?

True. Methadone drug of choice for pregnant women.

NSAIDS should be avoided in cirrhotic liver disease true or false?

True: While hepatotoxicity with NSAIDs is rare, increase the risk of bleeding in cirrhotic patients, = impair platelet function. In addition, NSAIDs decrease blood flow to the kidneys and can increase the risk of renal failure in patients with cirrhosis. Nsaids compared to aspirin= cardiovascular morbidity, worsen heart failure, increase blood pressure, and increase events such as ischemia and acute myocardial infarction.

right upper quadrant pain initial imaging study?

US of RUQ.

Treatment ofr microscopic hematuria?

US of kidney, urine cytology and cystoscopy. The AUA also recommends that all patients older than 40 and those who are younger but have risk factors for bladder cancer undergo cystoscopy to complete the evaluation.

Imaging for children with neck masses?

US.

38-year-old male comes to your office because of right elbow pain. He recently began participating in a highly competitive adult volleyball league, and 2 weeks after he first began playing he developed mild pain in the medial elbow of his right arm. While completing an overhead serve last night he felt an acute worsening of the elbow pain. After the match he noted bruising over his medial elbow. When you examine him you find bruising and pain to palpation around the medial elbow. With his shoulder in 90° of abduction and external rotation you rapidly flex and extend the elbow while maintaining valgus torque on the elbow (the moving valgus stress test). The patient reports pain between 70° and 120° of flexion. This clinical presentation is most consistent with which one of the following causes of elbow pain?

Ulnar collateral ligament injury= UCL is primary restraint to valgus stress on the elbow during overhead throwing. --report a pop followed by immediate pain and bruising around the medial elbow. The moving valgus stress test has 100% sensitivity and 75% specificity for diagnosing UCL injuries.

Which one of the following is recommended with regard to the use of osteoporosis medications in elderly patients? (check one) A. Substitution of denosumab (Prolia) for bisphosphonates in patients planning extensive dental work B. Use of denosumab in patients at increased risk for infection C. Use of denosumab rather than bisphosphonates in patients with class III or IV renal dysfunction D. Continuous use of bisphosphonates for 10 years or more

Use of denosumab rather than bisphosphonates in patients with class III or IV renal dysfunction. Bisphosphonates should not be used in patients with a creatinine clearance <35 mL/min/1.73 m2, but denosumab is not cleared by the kidneys and is safe in patients with chronic kidney disease. The use of bisphosphonates for more than 5 years can increase the risk of atypical fractures and a holiday from the drug is recommended after either 3 or 5 years, depending on the drug used.

A 42-year-old female presents to the emergency department with a 2-hour history of palpitations. Her physical examination is normal except for what seems to be a regular rhythm tachycardia and a blood pressure of 84/54 mm Hg. An EKG reveals a regular narrow-complex tachycardia at a rate of 180 beats/min without clear atrial activity. The optimal treatment for this patient is

Vagal maneuvers and administration of adenosine are useful in the diagnosis and treatment of narrow-complex supraventricular tachycardias. Adenosine, a very short-acting endogenous nucleotide that blocks atrioventricular nodal conduction, terminates nearly all atrioventricular nodal reentrant tachycardias and atrioventricular reciprocating tachycardias, as well as up to 80% of atrial tachycardias. CCB not as good because can cause hypotension.

A 24-year-old gravida 2 para 1 at 9 weeks gestation sees you for a routine prenatal check. She complains of significant nausea, and recommended dietary modifications have not helped. She drives a school bus so she would like to avoid sedating medications. She appears well-hydrated and her examination is otherwise normal. Best tx for relieving this patient's nausea?

Vitamin B6.

Which one of the following would you suggest for improving iron absorption?

Vitamine C or meal high in meat protein. -- calcium and coffee decrease iron absorption but not as much as tea (decreases by about 90%).

Preventing or alleviating nipple pain is important for comfort, but also for promoting breastfeeding in general. The best intervention for nipple pain is

education on proper positioning and attachment of the infant.

MMSE is known to be affected by what factor?

educational attainment.

when should you get screening with colonoscopy if someone in your family has had colon cancer?

either age 40 or 10 years before the age of diagnosis.

symptomatic peripheral arterial disease should be started on

a daily dose of aspirin or clopidogrel to prevent MI or stroke.

Tx of GDM is good why?

a decreased risk for operative delivery, large-for-gestational-age infants, shoulder dystocia, and maternal preeclampsia. does not change chances of T2D after delivery.

A 26-year-old pet groomer sustained a dog bite to her left hand 2 hours ago. On examination a 4-cm × 2.5-cm laceration is noted on the thenar eminence of her palm. Although the wound shows some gaping there is minimal active bleeding. No neurovascular injury is noted. Which one of the following is an indication for antibiotics in this patient?

a high risk include a bite on an extremity with underlying venous and/or lymphatic compromise a bite involving the hand, a bite near or in a prosthetic joint, cat bites crush injuries, delayed presentation puncture wounds, underlying diabetes mellitus, and immunosuppression.

Contraindications to treatment with electrosurgery include the use of ... and the treatment of ....

a pacemaker and melanoma.

In patients with type 2 diabetes mellitus, the single most important predictor of severe hypoglycemia is

a previous history of severe hypoglycemia that required external assistance.

Patient Health Questionnaire (PHQ-2)?

a screening instrument that is specific for depression.

An 18-year-old female basketball player comes to your office the day after sustaining an inversion injury to her ankle. She says she treated the injury overnight with rest, ice, compression, and elevation. You examine her and diagnose a moderate to severe lateral ankle sprain. In addition to rehabilitative exercises, you advise

a semi-rigid stirrup brace (Air-Stirrup, "Aircast"). -allows flexion and axtension, or a soft lace-up brace is recommended over immobilization.

Amiodarone is associated with ...

a subacute cough and progressive dyspnea due to pulmonary toxicity (patchy interstitial infiltrates).

initial evaluation for chronic pelvic pain should include

a urinalysis and culture, cervical swabs for gonorrhea and Chlamydia, a CBC, an erythrocyte sedimentation rate, a β-hCG level, and pelvic ultrasonography.

hematospermia in males <40 what do you do?

examine testes and prostate (usually normal) and if sexually active screen STI.

renal colic?

abdominal pain commonly caused by kidney stones.

most consistent symptom of IBS?

abdominal pain.

best diagnosing method for acute appenendicitis?

abdominal/ pelvic CT.

A 24-year-old female has noted excessive hair loss over the past 2 months, with a marked increase in hairs removed when she brushes her hair. She delivered a healthy baby 5 months ago. She is on no medications, and is otherwise healthy. Examination of her scalp reveals diffuse hair thinning without scarring. An evaluation for thyroid dysfunction and iron deficiency is negative. Which one of the following is the most likely cause of her hair loss? A. Telogen effluvium B. Anagen effluvium C. Alopecia areata D. Female-pattern hair loss E. Discoid lupus erythematosus

about 3 months after a triggering event. The hair loss with telogen effluvium lasts 6 months after the removal of the stressful trigger. Anagen effluvium is the diffuse hair loss that occurs when chemotherapeutic medications cause rapid destruction of anagen-phase hair. Alopecia areata, which causes round patches of hair loss, is felt to have an autoimmune etiology. Female-pattern hair loss affects the central portion of the scalp, and is not associated with an inciting trigger or shedding. Discoid lupus erythematosus causes a scarring alopecia.

what is a felon?

abscess of the distal pulp of the fingertip.

Systolic hf patient medications that improve symptoms and survival?

ace inhibitior and b blocker. -if you cannot tolerate ace inhibitor direct acting vasodilators like isorbide and hydralazine is perferred.

HSP management?

acetaminophen (avoid ibuprofen in abdominal pain or known renal involvement). --Prednisone has been found to help in those with renal involvement or other complications of the disease such as significant abdominal pain, scrotal swelling, or severe joint pains (SOR B). However, it is not effective for preventing renal disease or reducing the severity of renal involvement, as was once thought.

febrile seizure in a kid and normal exam tx?

acetaminophen.

is iron absorbed better in an acidic or basic environment? so what should you give with Fe for absorption purposes? iron better on empty or full stomach?

acidic. vitamin C. empty; but can get constipation then.

a fall or a direct blow to the point of the shoulder with the shoulder abducted. The pain associated with this injury is over the acromioclavicular joint margin and there may be swelling. Depending on the severity of the injury there may be full range of motion but it may be restricted due to pain. There should not be any weakness associated with this injury. what is this?

acromioclavicular seperation.

A 72-year-old white male presents with a complaint of headache, blurred vision, and severe right eye pain. His symptoms began acutely about 1 hour ago. Examination of the eye reveals a mid-dilated, sluggish pupil; a hazy cornea; and a red conjunctiva. Which one of the following is the most likely diagnosis?

acute angle closure glaucoma.

In patients with no known coronary artery disease (CAD), the presence of frequent premature ventricular contractions (PVCs) is linked to

acute myocardial infarction and sudden death. evaluate CAD in patients who have >30 PVCs per hour and cardiac risk factors such as HTN and smoking. May include stress test, echocardio and ambulatory rhythm monitoring.

Based on the 2013 ACOG guidelines for management of hypertension in pregnancy, which one of the following should be the next step in management?

admit patient for induction of labor after gestational hypertension after 37 weeks. Identifying elevated urine protein is not required for this decision, as gestational hypertension and preeclampsia without severe features are managed in the same way at 39 weeks gestation. Twice-weekly office visits with assessment of blood pressure and the other tests mentioned may be appropriate for patients at less than 37 weeks gestation.

RF for biceps tendon rupture?

age >40, deconditioning, contralateral biceps tendon rupture, a history of rotator cuff tear, rheumatoid arthritis, and cigarette smoking. will have weakness supination and elbow flexion may be present. dx: biceps squeeze test and hook test both sensitive and specific for dxing.

sepsis secondary to urinary infection= can cause renal failure due to low perfusion of the kidney (Prerenal azotemia) tx?

aggressive high rate IV normal saline. -low dose dopamine has proven to be ineffective.

0.3% of people taking methimazole develop?

agranulocytosis within first 60 days of starting. other se include: serum sickness. cholestatic jaundice. alopecia. nephrotic syndrome. hypoglycemia. loss of tase.

Which one of the following has the best evidence of effectiveness for preventing fractures in postmenopausal women with osteoporosis? (check one) A. Home-hazard assessment B. Daily supplementation with vitamin D C. Treatment with calcitonin D. Treatment with alendronate (Fosamax)

alendronate= prevents osteoporotic hip and vertebral fractures.

Screening for chronic hepatitis B infection is NOT recommended for which one of the following? (check one) A. Patients on chronic immunosuppressive therapy B. Patients with end-stage renal disease who are on hemodialysis C. Household contacts of individuals with chronic hepatitis B D. Pregnant women with no risk factors for hepatitis B E. All newborns

all newborns.

Allopurinol or colchicine which causes sjs?

allopurinol.

Fibromyalgia is characterized by tender trigger points where?

along the medial scapula borders, posterior neck, upper outer quadrants of the gluteal muscles, and medial fat pads of the knees.

Central retinal artery occlusion findings?

amaurosis fugax, a red conjunctiva, a pale fundus, a cherry-red spot at the fovea, and "boxcarring" of the retinal vessels.

Which one of the following is known to cause hyperthyroidism? (check one) A. Propranolol (Inderal) B. Amiodarone (Cordarone) C. Methimazole (Tapazole) D. Propylthiouracil E. Methotrexate (Rheumatrex, Trexall)

amiodarone= 37% iodine. Methimazole and propylthiouracil interfere with organification of iodine, thereby suppressing thyroid hormone production; they are commonly used as antithyroid agents when treating hyperthyroidism.

most appropriate tx for fibromyalgia?

amitryptaline.

first-line treatment for previously healthy infants and school-age children with mild to moderate community-acquired pneumonia

amoxicillin. --most common CAP is S. pneumonae. If patient was older then it would be azithromycin because of M.pneunoniae.

Good option for diverticulitis for patient allergic to metronidazole?

amoxicillin/ clavulanate. --at least covers the anaerobic coverage.

mild hyperparathyroidism that becomes evident only with

an added calcium load that can occur with thiazide.

4-month history of pain in the lower lumbar region without radiation. He works in retail sales, and the pain and stiffness prevent him from working. some relief with spinal manipulation. negative for red flags indicating a serious cause for his pain. diffuse mild tenderness over the lumbar region and mild limitation of lumbar mobility on forward and lateral flexion/extension maneuvers. tx for the chronic back pain?

analgesics (acetaminphen or NSAIDs) spinal manipulation a multidisciplinary rehabilitiation.

if ct is positive next step for patient with suspected subarachnoid hemorrhage?

angiography and tx.

most specific for diagnosing RA?

anti-ccp.

A 29-year-old white female is hospitalized following a right middle cerebral artery stroke confirmed by MRI. Her past medical history is remarkable only for a history of an uncomplicated tonsillectomy during childhood and a second-trimester miscarriage 3 years ago. The only remarkable finding on physical examination is left hemiplegia.The initial laboratory workup reveals normal hematocrit and hemoglobin levels, a normal prothrombin time, and a platelet count of 200,000/mm3 (N 140,000-440,000). The activated partial thromboplastin time is 95 sec (N 23.6-34.6), and it does not normalize when the patient's serum is mixed with normal plasma. A serum VDRL is positive, and a serum FTA-ABS is nonreactive. Which one of the following is the most likely diagnosis?

anti-phospholipid syndrome= heterogeneous group of circulating antibodies to negatively charged phospholipids, including most commonly a lupus anticoagulant and anti-cardiolipin antibodies.

rf for Clostridium difficile colitis includes?

antibiotic use, it is posing an increasing threat to patients in hospitals and chronic-care facilities who have not been given antibiotics.

COPD exacerbation managed if moderate/ severe?

antibiotics. brief course of systemic corticosteroids shorten hosptial stay and decrease tx failure. inhaled corticostroids not of hlep.

First-generation nonprescription antihistamines can enhance

anticholinergic and sedative effects of other medication.

kava in the short term tx for? up to 24 weeks=

anxiety disorder. GAD.

who is an appropriate candidate for hospice?

any patient with a life expectancy of less than 6 months who chooses a palliative care approach.

heart condition with marfan's?

aortic insufficiency or MVP. Aortic root dilation in 80-100% of affected individuals.

A 36-year-old female presents with a several-week history of polyuria and intense thirst. She currently takes no medications. On examination her blood pressure and pulse rate are normal, and she is clinically euvolemic. Laboratory tests, including serum electrolyte levels, renal function tests, and plasma glucose, are all normal. A urinalysis is significant only for low specific gravity. Her 24-hour urine output is >5 L with low urine osmolality. The most likely cause of this patient's condition is a deficiency of:

arginine vasopressin (antidiuretic hormone). -DI due to a deficiency in the secretion or renal action of arginine vasopressin. -The urine is very dilute, with osmolality <300 mOsm/L. Low levels of aldosterone, plasma renin activity, or angiotensin would cause abnormal blood pressure, electrolyte levels, and/or renal function. Insulin deficiency results in diabetes mellitus.

Radial head subluxation= nursemaid's elbow tx?

as long as no outward signs of fracture or abuse= attempt reduction of the radial head before moving on to imaging studies. -- 90 degree flexed position, supinate the hand and bring the elbow into full supination. --fx back to normal in a few minutes.

A 30-year-old male presents with a 2-week history of swelling of the right posterior elbow. He recalls bumping his elbow against a door, but his pain quickly subsided. He began to notice the swelling over the next 2 days. On examination he has normal range of motion with a boggy, nontender mass over the olecranon. what is this and tx?

aseptic olecranon bursitis= preceded by minor trauma to the elbow followed by a nontender, boggy mass over the olecranon. -septic olecranon bursitis causes not just swelling but erythema, warmth and pain too. if it is septic then aspirate. but if it aseptic then do compression dressings/ ice/ avoid activities that aggravate the problem.

how can you decrease fetal loss with antiphospholipid antibody syndrome?

aspirin and hepatrin combined. prednisone administration= actually increased risk of premature rupture of membranes, preterm, FGR, infection, preeclampsia, diabetes, osteopenia and avascular necrosis.

most common presenting symptoms of OSA?

excessive daytime sleepiness.

severe aortic stenosis f/u how?

exercise treadmill test even if asymptomatic.

weeks 11-16 weeks gestation mom should be screened for?

asymptomatic bacteriuria.

Should it become necessary, the patient's decision-making capacity is determined by:

attending physician. capacity is physician. competency is lawyer.

Which one of the following Papanicolaou (Pap) test results is most likely to indicate a cancerous lesion?

atypical glandular cells not otherwise specified (17%) chance of cancer. --HSIL 3% chance.

Which one of the following is a frequent cause of cross-reactive food-allergy symptoms in latex-allergic individuals? (check one) A. Avocadoes B. Goat's milk C. Pecans D. Pastrami E. Peppermint

avocadoes. plant derived food= cross reacts.

treatment of choice for campylobacter?

azithromycin. Typical treatment for adults is 500 mg daily for 3 days and in children 10 mg/kg/day for 3-5 days.

The preferred antibiotic treatment for community-acquired pneumonia in a young adult in the ambulatory setting is:

azithromycin. -covers the atypicals= most common cause of CAP. Levofloxacin covers atypicals but ciprofloxacin does not.

...the only nasally inhaled anticholinergic recommended by the American College of Chest Physicians for a cough caused by the common cold.

ipratropium.

Ferritin low, TIBC high, serum iron low means what?

iron deficiency.

HTN medication which one is not first line?

b- blocker. . ACE inhibitors, angiotensin receptor blockers (ARBs), calcium channel blockers, and thiazide-type diuretics all yielded comparable effects on overall mortality and cardiovascular, cerebrovascular, and kidney outcomes. They are all recommended for initial treatment of high blood pressure in the nonblack population, including patients with diabetes mellitus. $-Blockers were not recommended for the initial treatment of hypertension because one study found there was a higher rate of the primary composite outcome of cardiovascular death, myocardial infarction, or stroke with use of these drugs compared to the use of an ARB.

prior to surgery when should b blockers be recommended? how about statins?

b-blocks several weeks beforehand and carefully titrated. statins are recommended perioperative period regardless of other cardiac risk.

In the U.S., the most common nutritional deficiency is:

iron.

Dx criteria for fibromyalgia:

baed on patient symptoms (structured symptom hx). not tender points any more.

most effective for long-term weight loss in a patient BMI >40 with no complications or BMI >35 with symptoms=

bariatric surgery.

body lice transmit what disease?

bartonella quintana= trench fever. influenza like syndrome. can be txed with antibiotics.

most effective for obesity?

behavioral counseling- multiple behavioral management activities, such as group and individual sessions, setting weight-loss goals, addressing barriers to change, and active use of self-monitoring. low carbohydrate diets are minimally effective over the long term w/out behavioral intervention.

Children who have reach the weight or height limit of their forward-facing child safety seat should use a belt-positioning booster seat until the seat belt fits properly, typically when the child is 145 cm (57 in) tall and between 8 and 12 years of age

belt-positioning booster seat until the seat belt fits properly, typically when the child is 145 cm (57 in) tall and between 8 and 12 years of age

a history of vague anterior elbow pain and a history of repeated elbow flexion with forearm supination and pronation, such as dumbbell curls. Resisted supination produces pain deep in the antecubital fossa.

biceps tendinopathy.

irritation and microtrauma due to repetitive elevation or abduction of the shoulder, causing an inflammatory reaction in the synovial sheath. is called? common complaint?

bicipital tendinitis. pain in the anterior shoulder that radiates into the upper arm. more painful with activity and is worse at night. abduction and external rotation exacerbate the pain. no weakness.

In order to make polymalgia rheumatica dx what do you need?

bilateral shoulder or hip stiffness and aching for at least one month in order to make the dx. esr should be 40+.

intrahepatic cholestasis of pregnancy has what increased?

bile acids. no primary skin lesion and elevation of serum levels of total bile acids diagnostic.

sevelamer moa?

binds to phosphate in the GI and lowers PTH. --less phosphate= less PTH= less calcium absorbed/ released from bones.

herbal supplement saw palmetto and echinacea not associated with?

bleeding.

Sympathomimetic agents can elevate ..., may worsen existing ..., and adversely interact with... speed up...

blood pressure and intraocular pressure urinary obstruction. B-blockers, methyldopa, tricyclic antidepressants, and oral hypoglycemic agents and MAOIs. HR.

A 45-year-old male was admitted to the hospital for nausea resulting from chemotherapy for colon cancer. He has no other chronic diseases and takes no routine medications. He was mildly dehydrated on admission and has been receiving intravenous fluids (D5 ½-normal saline with potassium chloride) at slightly higher than maintenance rates through an indwelling port for the last 24 hours. The nausea is being controlled by antiemetics, and his condition is improving. Results of routine blood work at the time of admission and from the following morning are shown below. Admission Following Morning Glucose 109 mg/dL (N 65-110) 371 mg/dL BUN 13 mg/dL (N 7-21) 9 mg/dL Creatinine 0.9 mg/dL (N 0.6-1.6) 0.9 mg/dL Sodium 143 mmol/L (N 136-144) 129 mmol/L Potassium 3.7 mmol/L (N 3.6-5.1) 6.6 mmol/L Chloride 110 mmol/L (N 101-111) 108 mmol/L Total CO2 20 mmol/L (N 22-32) 22 mmol/L Which one of the following would be the most appropriate next step? (check one) A. Start an intravenous insulin drip B. Order blood work taken from a peripheral vein C. Restrict the patient's free water intake D. Switch from normal saline to hypertonic saline E. Treat with diuretics

blood work from peripheral vein because they probably drew blood from the indwelling port without discarding the first several milliliters. --blood was contaminated by IV fluids.

if you have symptoms and findings of multiple myeloma then what do you do to confirm?

bone marrow examination with >10% plasma cells. The serum level of monoclonal immunoglobulin is typically >3 g/dL.

Which one of the following is a classic finding in multiple myeloma? (check one) A. Hypokalemia B. Bone pain C. Polycythemia D. Hepatic failure E. Insomnia

bone pain. also renal insufficiency, elevated calcium, anemia and bone disease.

preferred tx for hyperprolactinemia?

bromocriptine or cabergoline (dopamine agonist).

plantar fascititis=

burnign pain in the heel and there is tenderness of the planter fascia where it inserts onto the medical tubercle of the calcaneus.

A lumbar puncture following a seizure is not routinely recommended in a child over 18 months of age, since

by that age a patient with meningitis would be expected to demonstrate meningeal signs and symptoms or clinical findings suggesting an intracranial infection.

A 2-month-old female is brought to your office with tachypnea and a staccato cough. She is afebrile. A chest radiograph shows hyperinflation and bilateral infiltrates, and a CBC reveals eosinophilia. Which one of the following is the most likely etiologic agent?

c. trachomatis. tachypnea, a staccato cough, and no fever. hyperinflation and infiltrates, and a CBC will reveal eosinophilia.

Extracorporeal shock wave therapy is effective for what shoulder problem.

calcific tendinitis of the rotator cuff. se? bruising and pain.

Which one of the following medications used in the treatment of osteoporosis can also be used to treat the pain associated with acute and chronic vertebral compression fractures? (check one) A. Calcitonin-salmon (Miacalcin) B. Raloxifene (Evista) C. Risedronate (Actonel) D. Teriparatide (Forteo) E. Zoledronic acid (Reclast)

calcitonin-salmon is useful in the management of pain associated with acute or chronic vertebral fractures.

tx for magnesium toxicity?

calcium gluconate= antidote. --can use calcium chloride if central line. --calcium gluconate safer with peripheral lines.

For which type of renal calculus is acidification of the urine indicated? (check one) A. Cystine B. Uric acid C C. Calcium oxalate D. Calcium phosphate

calcium phosphate.

a patient without apparent lung disease who develops a spontaneous "small" pneumothorax (<15% of lung volume) can be managed as: A. CT of the affected lung B. Analgesics and a follow-up visit in 48 hours C. Chest tube insertion D. Hospital admission and a repeat chest film in 24 hours

can be managed as an outpatient with analgesics and follow-up within 72 hours. CT of the lung is needed in complicated cases, including patients with known lung disease or recurrent pneumothoraces. A chest tube is required only when the pneumothorax involves >15% of lung volume.

-glucosidase inhibitors moa?

carbohydrate absorption in the small intestine.

uterine atony in asthmatic what is contraindicated?

carboprost.

A 28-year-old male is seen for follow-up of acute low back pain. He has a past history of substance abuse. Ibuprofen and acetaminophen have helped some, but he is experiencing muscle spasms. It is best to avoid which one of the following when treating this patient's problem? (check one) A. Chlorzoxazone (Parafon Forte DSC) B. Metaxalone (Skelaxin) C. Cyclobenzaprine (Flexeril) D. Methocarbamol (Robaxin) E. Carisoprodol (Soma)

carisoprodol. . Carisoprodol is metabolized to meprobamate, which is a class III controlled substance. It has been shown to produce both physical and psychologic dependence.

pain and bilateral paresthesias in the distribution of her median nerve. She says that the pain frequently radiates as far as her shoulder, and that her fingers feel swollen even though they look normal. She states that she has worsening paresthesias at night and often finds herself flicking her wrist in an attempt to alleviate her symptoms. wrist splints at night for the last 8 weeks and has also been taking oral NSAIDs, resulting in only minimal relief. She is in the middle of her concert season and is unable to take time off for a surgical procedure. what is this? longest symptoms relief with what?

carpal tunnel syndrome. local corticosteroid injection.

In this patient, the presence of increasing low back pain and leg weakness, and the findings of anal sphincter laxity and numbness in the perianal area on examination, suggest the presence of a serious neurologic etiology such as next step?

cauda equina syndrome. mri of lumbosacral spine.

acute parotitis do what?

caused by dehydration and can be diagnosed from the hx and examination. give emperic tx= amoxicillin/ clauvulanate. -- f/u with cultures.

primary insomnia tx?

cbt.

gbs + pregnant woman allergic to penicillin tx?

cefazolin. if higher risk for anaphylaxis than a rash then go for vancomycin or clindamycin.

recommended tx regimen for gonorrhea?

ceftriaxone IM 250 mg. -- even if -- NAAT test give 1 gram azithromycin PO.

patient presenting with IBS should be tested for what?

celiac disease.

patient with symptoms of IBS differential diagnosis includes?

celiac spruce. microscopic and collagenous colitis. atypical crohn's disease for patients with diarrhea prominent IBS. Chronic constipation.

Children with diabetes mellitus are at increased risk for retinopathy, nephropathy, and hypertension. They are also more likely to have immune-mediated disorders such as

celiac. hypothyroidism.

painless, unilateral, sudden loss of vision over a period of seconds may be caused by thrombosis, embolism, or vasculitis causing?

central retinal artery occlusion. Occlusion of the central retinal vein causes unilateral, painless loss of vision, but the retina will show engorged vessels and hemorrhages.

when is a tb induration 10+mm positive?

children; employees or residents of nursing homes, correctional facilities, or homeless shelters; recent immigrants; intravenous drug users; hospital workers; and those with chronic illnesses. For individuals who are subject to repeated testing, such as health-care workers, an increase in induration of 10 mm or more within a 2-year period would be considered positive and an indication of a recent infection with Mycobacterium tuberculosis. A nurse with a 9-mm induration would be considered to have a negative PPD.

when should a patient be given the pneumococcal vaccine before the age of 65?

chronic lung disease cardiovascular disease diabetes mellitus chronic liver disease cerebrospinal fluid leaks cochlear implants immunocompromising conditions asplenia residents of nursing homes and long-term care facilities. 50-64 in the following categories: Alaska Natives, Native Americans living in areas of increased risk, persons with asthma, and smokers.

One time revaccination after 5 years is recommended for for who?

chronic renal failure. asplenia. immunocompromised. diabetic patient and nursing home resident can receive the vaccine before 65 but then only 1 more 65.

lichen sclerosis what is it and tx?

chronic, progressive, inflammatory skin condition found in the anogenital region. intense vulvular itching. tx= high potency topical corticosteroids. white, thickened excoriated skin.

what medication that can be used for anxiety can also be used for IBS?

citalopram.

patient taking statin. what antibiotic is contraindicated?

clarithromycin or erythromycin.

A 50-year-old female presents with a 3-week history of a moderately pruritic rash, characterized by flat-topped violaceous papules 3-4 mm in size. The lesions are located primarily on the volar wrists and forearms, lower legs, and dorsa of both feet. Ten days after the rash first appeared she went to the emergency department and was treated for "possible scabies," but the treatment has made little or no difference. Which one of the following treatments is indicated at this time?

classic lichen planus= pruritis symmetric papular lesions. violaceous flat topped papules, 3-6 mm in size and distinct in appearance that biopsy is not required. 1. tx is clobetasol- high potency topical corticosteroid (triamcinolone medium potency ineffective). 2. if high potency not effective use calcinurien inhibitors like tacrolimus topical.

cilostazol helps with?

claudication symptoms; PDE inhobitor and antiplatelet.

A 63-year-old female with type 2 diabetes mellitus presents to the emergency department with unstable angina. Her blood pressure is 150/90 mm Hg, her pulse rate is 70 beats/min, and her lungs are clear to auscultation. The patient expresses a preference for conservative (i.e., noninvasive) therapy. In addition to aspirin, which one of the following agents should be administered at this time?

clopidogrel= asap in unstable angina/ nstemi. --daily for 1 month w/ aspirin.

as a very severe, constant, agonizing orbital pain, usually beginning within 2 or 3 hours after falling asleep.

cluster headache.

Headaches occurring in cycles unilateral. what type of headache?

cluster.

a treatment process that helps patients correct false self-beliefs that can lead to negative moods and behaviors

cognitive therapy,

if lesion is suspicious for melanoma based on asymmetry, irregular border, color variegation, and size larger than 6 mm. the preferred method of biopsy is?

complete elliptical excision with small margin of normal- appearing skin. --single punch biopsy is unlikely to capture the entire malignant portion in larger lesions.

What if it is moderate risk?

compression US.

A 30-year-old female who had a deep venous thrombosis in her left leg during pregnancy has an uneventful delivery. During the pregnancy she was treated with low molecular weight heparin. Just after delivery her left leg is pain free and is not swollen. She plans to resume normal activities soon. Which one of the following would be most appropriate with regard to anticoagulation?

continue low molecular weight heparin/warfarin for 6 more weeks because risk of PE still continues postpartum.

A 47-year-old male with chronic kidney disease is being treated with epoetin alfa (Procrit). His hemoglobin level is 11.3 g/dL (N 13.0-18.0). Which one of the following would be most appropriate with regard to his epoetin alfa regimen?

continue the current regimen= if hgb is >13.5 then risk of death and serious CV event is increased. -- maintain levels at 10-12. --set target at <12.

pallidotomy and thalamotomy are effective tx for what?

contralateral to the side of the body most effected; help with parkinsonism tremor and dyskinesia.

Trochanteric bursitis tx?

corticosteroid injection.

Most common cause of Secondary osteoporosis in males?

corticosteroids. others include: excessive alcohol. hypogonadism. vitamin D deficiency. low BMI.

Other recommended testing prior to pre-exposure prophylaxis is?

creatinine clearance calculation, hepatitis B ab testing, screening for STI, pregnancy testing in females capable of pregnancy.

preferred site for emergency airway is?

cricothyroid membrane= directly above the cricoid cartilage through the membrane. -below vocal cords. -not in trachea. the preferred tracheotomy site under controlled circumstances= below the cricoid cartilage= 2-4 tracheal rings and thyroid isthmus but can have excessive bleeding and difficulty finding the trachea in an emergency.

what are some indications for indwelling catheter?

critical illness, obstruction, hospice care, perioperatively for <2 days for urologic procedures.

. A young person, usually in the second or third decade, will present with a period of episodic abdominal pain, largely postprandial and often periumbilical, occasionally with low-grade fever and mild diarrhea. Anorexia, nausea, and vomiting may also be present. Weight loss is frequent. Some patients may be aware of tenderness in the right lower quadrant and even of a palpable mass in that region dx?

crohn's disease.

suspected subarachnoid hemorrhage first study ordered=

ct without contrast. in 100% of cases within 12 hours of the bleed, and it is useful for identifying other sources for the headache, for predicting the site of the aneurysm, and for predicting cerebral vasospasm and poor outcome.

acute lower back pain relief tx?

cyclobenzaprine= non bzd muscle relaxant. bed rest inadvisable.

Symptoms >2 days/week but not daily, nighttime awakenings 3-4 times/month, short-acting β-agonist usage >2 days/week but not more than once daily, minor limitation to daily activities, FEV1 ≥80% predicted, and normal FEV1/FVC ratio?

mild persistant asthma.

A 45-year-old female presents to an urgent care center complaining of left-sided chest pain for the past 2 days. The pain is nonradiating and sharp in character, and increases with deep inspiration. She has no associated shortness of breath, cough, nausea, diaphoresis, or dizziness. She has no significant past medical history or recent travel history. On examination she is afebrile, with a pulse rate of 102 beats/min, a blood pressure of 116/72 mm Hg, and a respiratory rate of 22/min. Her lungs are clear and her heartbeat is regular with no murmurs. Her lower extremities have no edema, tenderness, or varicosities. Which one of the following is the most appropriate next step in her evaluation?

d-dimer.

A 37-year-old graphic designer presents to your office with a history of several months of radial wrist pain. She does not recall any specific trauma but notes that it hurts to hold a coffee cup. Finkelstein's test is positive and a grind test is negative, and there is tenderness to palpation over the radial tubercle. Which one of the following would be most appropriate at this point?

de Quervain's tensosynovitis. Finkelstein's test has good sensitivity and specificity in a patient with a negative grind test. -- Positive grind= scaphoid fracture. --> for this you can get a hand radiograph with a secondary thumb spica splinting.

if a patient wants to go into hospice but has cardioverter defibrillator in his body, then how do you handle the situation?

deactivate the defibrillator= inconsistent with the care goals of the patient and the family.

Medical hyperbaric oxygen is considered reimbursable treatment option by many insurers for a long list of diagnoses but the list of conditions with known benefit from hyperbaric oxygen includes:

decompression sickness. wounds caused by crush injury. short term for diabetic foot ulcers no studies about long term benefit.

tx for cacaneal apophysitis?

decreasing pain-inducing activities, anti-inflammatory or analgesic medication if needed, ice, stretching and strengthening of the gastrocnemius-soleus complex, and the use of orthotic devices.

Which one of the following additional findings would be most consistent with constitutional delay of growth and puberty? (check one) A. Impairment of the sense of smell B. Delayed bone age C. Elevated LH and FSH D. Elevated thyrotropin E. Elevated prolactin

delayed bone age. Bone age is delayed, but growth potential is often normal.

... patients who cannot adhere to the requirement for fluid restriction, or who have recalcitrant hyponatremia despite restriction.

demeclycocline.

When do you need prophylaxis?

dental, gastrointestinal, or genitourinary procedures is now indicated only for high-risk patients with prosthetic valves a previous history of endocarditis unrepaired cyanotic congenital heart disease (CHD) CHD repaired with prosthetic material for cardiac transplant recipients who develop valvular disease. - not MR and acquired vascular disease.

The mother of an 8-year-old female is concerned about purple "warts" on her daughter's hands. The mother explains that the lesions started a few months ago on the right hand along the top of most of the knuckles and interphalangeal joints, and she has recently noticed them on the left hand. The child has no other complaints and the mother denies any unusual behaviors. A physical examination is unremarkable except for the slightly violaceous, flat-topped lesions the mother described. What is the most likely cause for this patient's finger lesions?

dermatomyositis. Gottron's papules, flat topped.

1. absence of cough. 2. swollen, tender anterior cervical nodes. 3. Temperature >38C. 4. tonsillar exudate and swelling. one point for each and then one point for ages between 3-14 yo. point subtracted for >45. what does the cumulative score determine?

determines the likelihood of streptococcal pharyngitis and the need for antibiotics, and guides testing strategies. 0-1= low risk of S. pharnygitis and do not require testing. 2-3= test using rapid antigen test or throat culture and a positive warrants antibiotic therapy. 4 or += high risk for strep pharyngitis and empiric tx.

intense urge to void, which overcomes the patient's voluntary attempt to hold the sphincter closed; hence, the common term urge incontinence.

detrusor instability.

most common cause of urinary incontinence in elderly both men and women?

detrusor instability. incontinence may become worse with surgical relief of obstructive prostatic hypertrophy.

ventricular hypertrophy as a response to chronic systolic hypertension. The ventricle becomes stiff and unable to relax or fill adequately, thus limiting its forward output. The typical patient is an elderly person who has systolic hypertension, left ventricular hypertrophy, and a normal ejection fraction (50%-55%). what kind of HF is this?

diastolic.

acute rectal fissure initial topical tx?

diltiazem, nifedipine and nitroglycerin all dilate the internal sphincter.

what can cause csf fluid rhinorrhea?

direct communication between the subarachnoid space and the paranasal sinuses. Accidental trauma causes 70%-80% of CSF rhinorrhea cases, with 2%-4% of acute head injuries resulting in CSF rhinorrhea.

The scabies mite is predominantly transmitted by ...

direct personal contact.

postpartum urinary retention tx?

discharge with a catheter in place and close follow up. --resolves in 2-6 days post delivery but some can take several weeks. --use the self catheter until you are able to spontaneously micturate.

treatment for someone with low-risk peptic ulcer bleeding based on clinical and endoscopic criteria can be managed how?

discharged from the hospital on the same day as endoscopy with a ppi.

how should asthma be diagnosed in children?

do a spirometry. once the diagnosis is confirmed tx should commence with a SABA.

If you find microalbuminemia in urine for the first time, next step?

do another urine albumin/ creatinine ratio. After you diagnose chronic kidney disease then you do a renal US. -- no 24 hr urine needed because the ratio correlated well with it.

cardiogenic shock treatment?

dobutamine.

A 20-year-old college wrestler is seen for an examination prior to the wrestling season. He tells you that some friends have told him he should start taking dehydroepiandrosterone (DHEA), and he asks for your advice. What is true about the effects of this drug?

does not enhance muscle strength or performance. illegal. In fact, they increase serum estrogen and luteinizing hormone levels.

managing hypotension with sepsis what is the drug of choice?

dopamine.

only FDA approved oral tx for acne rosacea is?

doxycycline at subantimicrobial dosage (40 mg daily).

most effective for prophylaxis for lyme disease after removal of engorged deer tick?

doxycycline.

For how long should his blood pressure be periodically measured before considering the test complete?

drop in BP of at least 20 mm Hg systolic or 10 mm Hg diastolic within 3 minutes of standing up.

A 40-year-old white female lawyer sees you for the first time. When providing a history, she describes several problems, including anxiety, sleep disorders, fatigue, persistent depressed mood, and decreased libido. These symptoms have been present for several years and are worse prior to menses, although they also occur to some degree during menses and throughout the month. Her menstrual periods are regular for the most part. The most likely diagnosis at this time is:

dysthymia.

Individuals on the National Weight Control Registry typically do what in terms of: diet. exercise. weight checking. breakfast.

eat a low-fat diet rich in complex carbohydrates, eat breakfast daily, weigh themselves at least once a week, and are physically active for 60-90 minutes a day.

If you have a history and clinical examination of HF most important to distinguish systolic or diastolic. SO what do you do?

echocardiogram. physical findings and radiograph do not distinguish this.

if patient comes in with marfans what should you perform for screening?

echocardiography.

drug-induced myopathy caused by simvastatin, which is associated with

elevated creatine kinase.

hyperparathyroidism lab values?

elevated hyperparathyroid. serum cl high or mildly elevated. decreased serum phosphate. elevatd ALP.

diagnosis of diabetic ketoacidosis (DKA) is based on?

elevated serum glucose >250 and elevated serum ketone level (pH <7.3) a serum HCO3 <18.

LDH Haptoglobin Indirect bilirubin levels for someone getting hemolysis?

elevated. decreased. elevated.

appropriate management of thrombosed hemorrhoid presenting within 72 hours of onset of symptoms=

elliptical excision of thrombosed hemorrhoid and overlying skin under local anesthesia.

Female athlete triad=

energy availability, dietry resitrction and substantial energy expenditures disrupt pituitary and ovarian function. --history of stress fracture resulting from minimal trauma.

severe pancreatitis patient what method of nutrition is preferred?

enteral= lower complication of rates and shorter hospitalization.

For which one of the following respiratory infections should antibiotic therapy be initiated immediately upon diagnosis? (check one) A. Bronchitis B. Epiglottitis C. Laryngitis D. Rhinosinusitis E. Tracheitis

epiglottitis= because if H. flu gets bad then they can have swelling of airways and cause death. others you can see if it goes away in 4-10 days and if not then antibiotics.

For persistent v fib in addition to electrical defibrillation and CPR what medication can you give?

epinephrine or vasopressin. amiodorone if vasopressin fails. lidocaine if amiodorine fails.

An annular rash with a bright red outer border and partial central clearing is called?

erythema migrans.

2-day-old infant reveals flesh-colored papules with an erythematous base located on the face and trunk, containing eosinophils. Which one of the following would be most appropriate at this time?

erythema toxicum neonatorum, which usually resolves in the first week or two of life tx= just observe.

best tx for erythrasma?

erythomycin.

what fluoresces under wood's lamp? due to porphyrins.

erythrasma. Erythrasma is a superficial gram-positive bacterial infection caused by Corynebacterium minutissimum.

The characteristics of this lesion, including coral-red fluorescence under a Wood's light, suggests Corynebacterium infection, which is associated with erythrasma. tx?

erythromycin.

Polymyalgia rheumatica is usually associated with an elevated

esr.

The arginine vasopressin antagonist conivaptan is approved for the treatment of

euvolemic or hypervolemic hyponatremia, but not in patients who are obtunded or in a coma, or who are having seizures.

The arginine vasopressin antagonist conivaptan is approved for the treatment of ... or ... hyponatremia, but not in patients who are obtunded or in a coma, or who are having seizures.

euvolemic or hypervolemic.

Incidental discovery of adrenal masses what should you do?

evaluate for adrenal hormone secretion. A morning cortisol level >5 μg/dL after a 1-mg dose indicates adrenal hyperfunction. Additional testing should include 24-hour fractionated metanephrines and catecholamines to rule out pheochromocytoma. do fine needle aspiration of the mass after malignancy and pheochromocytoma have been excluded.

As you age what happens to your: Drug absorption. GFR. Lean body mass. Volume of distribution of water soluble drugs like digoxin. Percent body fat.

everything down except percent body fat.

diagnosis of vocal cord dysfunction should be considered in patients diagnosed with

exercise-induced asthma who do not have a good response to β-agonists before exercise. a normal expiratory portion but a flattened inspiratory phase (SOR C). . A decreased FEV1 and normal FVC would be consistent with asthma.

asympatomatic gallstone tx?

expectantly with no tx unless symptoms of biliary colic develop.

proportion of persons with the condition who test negative

false negatives.

proportion of people without the condition who test positive

false positives.

Studies show that patients who present for treatment of herpes zoster within 72 hours will benefit from antiviral therapy such as

famciclovir= reduces the pain and decreases the risk of postherpetic neuralgia.

low urine 24 hour calcium levels or a low urine calcium to urine creatinine ratio is not characteristic of hyperparathyroidism. This finding should suggest?

familial hypocalciuric hypercalcemia.

for diabetic fasting glucose has to be what? 2 hour plasma glucose and random blood glucose?

fasting= 126+. 2 hour= 200. random= 200 for symptomatic. a1c 6.5+ all for 2+ occasions.

what is chronic fatigue syndrome?

fatigue for 6 months and a minimum of four of the following symptoms: 1. impaired memory. 2. postexertional malaise. 3. muscle pain. 4. polyarthralgia. 5. tender lymph node. 6. sore throat. 7. new headaches. 8. unrefreshing sleep.

what is herpangina?

febrile disease caused by coxsackieviruses and echoviruses. Vesicles and subsequent ulcers develop in the posterior pharyngeal area (SOR C).

Which one of the following surgical procedures is associated with the highest risk for perioperative myocardial ischemia? (check one) A. Femoropopliteal bypass B. Pulmonary lobectomy C. Hip arthroplasty D. Transurethral resection of the prostate E. Mastectomy

femoropopliteal bypass. Surgical procedures associated with a high risk (>5%) of perioperative MI include aortic/ peripheral vascular surgery and emergent major operations. Head and neck surgery, intraperitoneal and intrathoracic surgery, orthopedic surgery, and prostate surgery carry an intermediate risk (1%-5%). Endoscopic procedures and cataract and breast surgeries are considered low-risk (<1%) procedures.

perferred narcotic for end stage renal disase?

fentanyl= 99% hepatic elimination. long hx of safe use in patients with renal failure. morphine, hydromorphone, hydrocodone all have active metabolites that accumulate in renal failure patients. - but can still be used. Meperidine, codeine, and propoxyphene are all contraindicated in chronic kidney disease because of the accumulation of toxic metabolites.

Which one of the following has been shown to be LEAST effective in the treatment of irritable bowel syndrome? (check one) A. Fiber B. Probiotics C. Antispasmodics D. Antidepressants

fiber.

what chronic problems are associated with pseudoseizures?

fibromyalgia and chronic pain syndrome. eye closure throughout the event is uncommon in true seizure.

Until when can OCPs be continued?

fifties.

tx for fracture of the medial third of the clavicle?

figure 8 apparatus- sending the patient home in a sling and swathe. surgery not indicated.

Patient comes in with hiccups that have been a couple of days and waking up from sleep= what is your primary focus for tx?

finding the underlying pathology causing the hiccups. respiratory reflex from phrenic and vagus nerves and thoracic sympathetic chain. tx for symptoms can be mechanical means (stimulating hte pharynx with tongue depressor) or medical tx (chlorpromazine).

A 42-year-old female is found to have a thyroid nodule during her annual physical examination. Her TSH level is normal. Ultrasonography of her thyroid gland shows a solitary nodule measuring 1.2 cm. Which one of the following would be most appropriate at this point?

fine needle aspiration biospy of the nodule. all patient who are found to have a thyroid nodule on a physical exam should have their tsh measured. "hot" nodules are almost never malignant. The first step in the evaluation of a thyroid nodule is to order a TSH level. If the TSH level is suppressed, radionuclide scintigraphy should be ordered to rule out a hyperfunctioning nodule. If the TSH level is either normal or high, the current recommendation is to biopsy only nodules >1 cm. Clinical follow-up is recommended for nodules :1 cm.

Three good medications for orthostatic hypotension?

fludrocortisone, midodrine, and physostigmine

initial tx for mild euvolemic hyponatremia is?

fluid restriction. Intravenous isotonic saline would be indicated for mild hypovolemic hyponatremia. Intravenous hypertonic saline would be indicated for severe hyponatremia with symptoms. Intravenous diuretics would be indicated for hypervolemic hyponatremia, such as in heart failure, along with fluid and sodium restriction.

Most appropriate initial treatment for patients with hypotension in septic shock is?

fluid resuscitation.

A Cochrane review found that ... was the only agent with consistent evidence of effectiveness in decreasing depressive symptoms in adolescents.

fluoxetine. fluoxetine, citalopram, or sertraline as first-line treatments for moderate to severe depression in children and adolescents. Escitalopram is also licensed for the treatment of depression in adolescents age 12 or older.

Which one of the following is most likely to cause hypoglycemia in elderly patients? (check one) A. Metformin (Glucophage) B. Pioglitazone (Actos) C. Glipizide (Glucotrol) D. Sitagliptin (Januvia) E. Glyburide (DiaBeta)

glyburide= sulfonylurea more like to cause hypoglycemia than glipizide. do not use glyburide in elderly.

decrease respiratory secretions but least likely to cause CNS problems?

glycopyrrolate. does not cross BBB. A. Atropine B. Transdermal scopolamine (Transderm Scop) C. Hyoscyamine (Levsin) all cross.

Pretibial myxedema is a cutaneous manifestation of?

graves disease.

A mother brings her 6-year-old son to your office for evaluation because she found a lump in his neck below the jaw on the right side. She first noted it a week ago, about a week after he had recovered from an upper respiratory infection. She reports that her son feels well and is back to full, unrestricted activity. When you examine the child you find an enlarged lymph node in the right anterior cervical chain of nodes. It measures 2 cm in diameter and is somewhat firm, mobile, and nontender. The remainder of the examination is normal. Which one of the following would be most appropriate at this point? (check one) A. Ultrasound evaluation of the lymph node B. A fine-needle biopsy of the node C. An excisional biopsy of the node D. A 10-day course of antibiotics E. Follow-up examination in 1 month

follow up examination in 1 month; Localized or generalized lymphadenopathy is a common complaint in children. Most cases are benign and related to infections or connective tissue disorders. Malignancy should be excluded if lymphadenopathy persists beyond 4 weeks, or if other symptoms develop, such as fever, night sweats, weight loss, hepatosplenomegaly, or orthopnea. In this situation, the evaluation should include a CBC, blood smear, erythrocyte sedimentation rate, and chest radiography (SOR C).

Snuffbox tenderness and tenderness over the scaphoid tubercle are very sensitive for what if you suspect but do not see it on imaging after it happens? tx plan?

fracture of scaphoid. Immobilize in a thumb spica splint for 1-2 weeks and then order repeat radiographs

ulnar gutter splint?

fracture of the fifth metacarpal head= boxer's fracture. Slight volar angulation and no displacement. --gutter splint imobalizes the wrist, hand and 4-5 digits in the neutral position. 3-4 weeks of continuous splinting is adequate. Surgical pinning is indicated in cases of significant angulation (35°-40° or more of volar angulation) or in fractures with significant rotational deformity or displacement. The other options listed are not appropriate treatments for this injury. This injury most commonly results from "man-versus-wall" pugilistics, but other mechanisms of injury are possible.

a fall on an outstretched arm or direct blow to the lateral side of the arm. Generally there is pain or bruising over the fracture site. Movement will be restricted by pain, but there should not be any weakness.

fracture of the humeral head.

A positive dipstick for hemoglobin without any RBCs noted in the urine sediment indicates either

free hemoglobin or myoglobin in the urine.

A 75-year-old white female presents with hyponatremia, with a serum level of 118 mEq/L, a urine osmolality >100 mOsm/kg H2O, and a serum osmolality of 242 mOsm/kg H2O. She complains of some fatigue, but is alert and oriented. Her blood pressure is 136/82 mm Hg. She has normal thyroid, adrenal, cardiac, hepatic, and renal function. You admit her to the hospital for treatment and observation. Which one of the following is the most appropriate initial treatment?

free water restriction. She has SIADH.

constitutional growth delay= being below the fifth percentile for height for most of childhood. Usually these children are thin and have a family history of delayed development. Bone age would be expected to be at least 2.5 standard deviations below the mean for age matched peers of the same chronologic age. However, most experts agree that if no signs of puberty are seen by 14 years of age (no breast development in girls, no testicular enlargement in boys), then

further workup for a more serious condition should be sought.

A 62-year-old male on hemodialysis develops a pruritic rash on his arms and chest, with erythematous, thickened plaques and edema. He had brain imaging with a gadolinium-enhanced MRI for neurologic symptoms 10 days ago. what is the problem and how can you diagnose it?

gadolinium-associated nephrogenic systemic fibrosis, which is associated with the use of gadolinium-based contrast material in patients with severe renal dysfunction, often on dialysis. No effective tx and mortality is 30%. deep biopsy of the affected area is diagnostic.

A 40-year-old male with acute pancreatitis has an alanine transaminase (ALT) level that is five times normal. Which one of the following is the most likely diagnosis? (check one) A. Gallstone pancreatitis B. Pancreatic necrosis C. Pancreatic pseudocyst D. Hepatitis C E. Alcohol-induced pancreatitis

gallstone pancreatitis=alanine transaminase has a positive predictive value of 95% for acute gallstone pancreatitis. high levels of crp= pancreatic necrosis. hepatitis c= antibody detection or polymerase chain reaction testing.

most common cause of hearing loss in newborns?

genetic disorders= waardenburg syndrome, usher's syndrome, alports and turners. more than 50% of hearing impaired children.

You receive a telephone call from the mother of a 5-year-old female. The child has had diarrhea and a decreased appetite for the past 2 days. She is still playing some. The mother reports no vomiting, but says her daughter has complained of a dry mouth and does not have tears when she cries. You suspect that the child may be mildly dehydrated. Which one of the following would you advise? (check one) A. Increased water intake B. Clear liquids with sodium, such as chicken broth C. An over-the-counter oral rehydration solution D. Intravenous fluids in the emergency department E. Loperamide (Imodium)

give OTC rehydration solution; 1. a capillary refill time >2 seconds 2. the absence of tears 3. dry mucous membranes 4. an ill general appearance; the presence of two or more of these signs indicates a fluid deficit of at least 5%.

Give what before the 23 PVR vaccine?

give the 13.

osteoarthritis of the knee best evidence for dietary supplement?

glucosamine sulfate.

Advanced dementia with concern of aspiration. Not adequate intake for nutritional requirements. No advance directives and waiting on family meeting. what do you do for her feeding in this case?

hand feeding. -as good as tube feedings in regards to outcomes of death, aspiration pneumonia, functional status and patient comfort.

if transfusion reaction occurs then what color is the serum and why?

haptoglobin binds enough free hgb and serum is pink.

You are examining a patient with a chronically painful shoulder. You forward flex the arm to 90° with the elbow bent to 90°. You then internally rotate the arm, which causes pain in the shoulder. this test is called? What is it testing for?

hawkin's impingement test. --rotator cuff tendinopathy/ tear.

Polycythemia vera weird lab findings?

hct >55. normal arterial O2 sat >92%. splenomegaly. thrombocytosis >400,000. leukocytosis >12000. Low serum EPO. Elevated LAP. high carboxyhemoglobin associated with secondary polycythemia.

best study to confirm the diagnosis of urinary tract stone in a patient with acute flank pain=

helical CT scan of the abdomen and pelvis without contrast.

patient with chronic mild liver disease can take nsaids but should be avoided in all patients with cirrhosis due to risk of precipitating?

hepatorenal syndrome.

A 75-year-old male with a history of hypertension sees you after experiencing an episode of numbness on his right side and loss of strength in his right arm. The numbness and weakness resolved spontaneously within 20 minutes. Carotid Doppler ultrasonography and cerebral angiography both reveal significant carotid stenosis. In addition to starting aspirin, which one of the following would be the most appropriate next step for this patient?

high dose statin therapy. -statins can help prevent strokes. they can help stabilize the intimal wall.

When draining a felon, which one of the following incisions is recommended?

high lateral incision or a volar longitudinal incision is recommended.

According to the American Heart Association's 2007 guidelines, prophylaxis to prevent bacterial endocarditis associated with dental, gastrointestinal, or genitourinary procedures is now indicated only for

high-risk patients with: 1. prosthetic valves 2. a previous history of endocarditis 3. un-repaired cyanotic congenital heart disease (CHD) or CHD repaired with prosthetic material, 4. cardiac transplant recipients who develop valvular disease. specifically exclude mitral valve prolapse and acquired valvular disease, even if they are associated with mitral regurgitation.

Risk of what due to omeprazole use?

hip, wrist, spine fracture. CAP. Hypomagnesemia. Cardiac events when coadministered with clopidogrel. --no problem with kidney stones or uti.

Obtaining a...is the most sensitive and accurate method of determining a patient's risk.

history for evidence of prior bleeding problems.

3 week old with history of not breathing for 20-25 seconds, having a blue tongue and lips, and being limp next step?

hospital admission for observation= this is an apparent life threatening event.

Status asthmaticus is a medical emergency and requires emergent treatment in a

hospital setting.

herbal supplement yohimbine associated with?

htn.

A 62-year-old African-American male is admitted to the hospital for the third time in 6 months with heart failure. He has dyspnea with minimal activity. Echocardiography reveals an ejection fraction of 40%. Which one of the following combinations of medications is most appropriate for long-term management of this patient? (check one) A. Enalapril (Vasotec) plus digoxin B. Hydralazine plus isosorbide dinitrate C. Losartan (Cozaar) plus amlodipine (Norvasc) D. Spironolactone (Aldactone) plus bisoprolol (Zebeta)

hydralazine + isosorbide nitrate. treatment with diuretics, β-blockers, and an ACE inhibitor (or ARB) is insufficient to control symptoms or cannot be tolerated.

Patients with advanced-stage cancer (especially of the lung or breast) may develop acute adrenal insufficiency from metastatic infiltration of the adrenal glands. Intravenous ... is the treatment of choice in the management of adrenal crisis.

hydrocortisone.

The finding of a normal parathyroid hormone (PTH) level in a patient with hypercalcemia is diagnostic for

hyperparathyroidism since PTH should be suppressed in the presence of elevated calcium. Symptomatic hypercalcemia causes dehydration because of both intestinal symptoms and diuresis.

Which one of the following metabolic abnormalities is most likely to be seen in patients with stage 4 kidney disease?

hyperparathyroidism. -- serum calcium and phosphate levels every 3-6 months and bone specific alkaline phosphatase activity every 6-12 months with the goal of normalizing these values.

reeclampsia affects as many as 5% of first pregnancies and is manifested as

hypertension, proteinuria, edema, and rapid weight gain after 20 weeks gestation. have a fourfold increased risk of hypertension and a twofold increased risk of ischemic heart disease, stroke, and venous thromboembolism.

Amiodarone side effects?

hyperthyroidism; hypothyroidism. -- have baseline TFT with follow up testing every 6 months to monitor.

A 55-year-old male is brought to the emergency department because of confusion and seizures. He has a history of hypertension and obstructive sleep apnea due to obesity. He is not conscious and no other history is available. An examination shows no focal neurologic findings, but a general examination is limited because of his size. Breath sounds are diminished, and heart sounds are difficult to hear. He has venous insufficiency changes on his lower extremities, with brawny-type edema. Laboratory testing reveals a sodium level of 116 mmol/L (N 135-145), but normal renal and liver functions. A chest radiograph shows mild cardiomegaly. A BNP level is pending, but immediate treatment is felt to be indicated. Which one of the following is the treatment of choice for this patient?

hypertonic saline= until seizures stop. This patient has severe hyponatremia manifested by confusion and seizures, a life-threatening situation warranting urgent treatment with hypertonic (3%) saline. The serum sodium level should be raised by only 1-2 mmol/L per hour, to prevent serious neurologic complications

Renal failure does what to your calcium?

hypocalcemia and can cause hypercalcemia= tertiary hyperparathyroidism. -This develops after severe hyperphosphatemia and vitamin D deficiency eventually produce hypersecretion of PTH.

b agonists can cause what electrolyte abnormality? other drugs causing the same abnormality?

hypokalemia= they activate potassium uptake by the cells. pseudoephedrine and insulin. diuretics=thiazides.

SIADH symptoms in labs?

hyponatremia and hypo-osmolality associated with increasing concentrated urine. --increased level of AVP. --euvolemic and not on diuretic for 24-48 hrs. --fractional excretion of sodium >1%.

Dipyridamole is associated with ... in elderly patients, but it benefits some individuals by preventing ....

hypotension strokes.

when would you do an arteriography with embolization for a gi bleed?

if it was persistently bleeding.

Which one of the following is most characteristic of the pain associated with acute pericarditis?

improvement when sitting up and leaning forward. worse with inspiration or in positions that put traction on the pleuropericardial tissues, such as lying supine.

Calcium carbonate is the least expensive and most commonly used supplement, but it is constipating and stomach acid ... its absorption. Calcium citrate is ... stomach acidity for absorption and it may be used with long-term gastric acid suppression agents. It may be taken without regard to food or meals.

improves. less dependent on.

Asthma symptoms occurring more than 2 days per week, but not daily, and use of her albuterol inhaler more than 2 days per week, but not daily. what level of asthma?

mild persistant.

rational approach to evaluating weight loss in an elderly patient. The workup should be directed by

including a CBC, chemistry panel, stool for occult blood, and TSH level.

exercise-related oligomenorrhea, but does not have the eating disorder that characterizes the female athlete triad tx?

increase caloric intake. Menstrual problems in athletes do correlate with bone density loss and impaired recovery from exercise. Hormonal manipulation has not been shown to affect bone density, though it may produce withdrawal bleeding.

Which one of the following effects of antioxidant supplementation has been demonstrated in randomized clinical trials?

increase in all-cause mortality associated with supplementation with vitamin E, vitamin A, and β-carotene. No benefits or reductions in all-cause mortality were demonstrated for vitamin C or selenium (SOR A).

in pregnancy does the levothyroxine dose need to be increased or decreased?

increased at 4-6 weeks of gestation.

combination of aspirin and clopidogrel is not suggested for stroke patients why?

increased bleed risk.

postoperative delirium associated with?

increased mortality. manage pain.

SE of tx of COPD with inhaled corticosteroids?

increased risk of bruising, candidal infection of oropharynx and pneumonia, increasing bone loss and fracture. --no benefit on mortality and do not improve FEV1 on a consistent basis.

cigarrete smoking does what for psoriasis?

increases the likelihood of it. psoriasis is associated with obesity, DM and Metabolic syndrome.

what is acute paronychia? tx?

infection around a fingernail beginning as cellulitis but can progress to a definite abscess. topical antibiotics w/ or w/out topical corticosteroids. -warm compress, oral antibiotics, incision/ drainage (not always necessary). -removal of proximal nail fold used for chronic paronychia that does not respond to other treatment.

Metformin has multiple mechanisms of action, but its main effect on serum glucose results

inhibition of gluconeogenesis in the liver.

mallet fracture=

injury to the thin tendon that straightens the end joint of a finger or thumb. - ie baseball finger.

Sulfonylureas and meglitinides stimulate

insulin release from the pancreas.

Symptoms less than or equal to twice weekly, nighttime awakenings ≤2 times/month, short-acting β-agonist usage ≤2 days/week, no interference with daily activities, and normal FEV1 and FEV1/FVC ratio at baseline?

intermittent asthma.

Complaints of discomfort in the eye with blurred vision and a history of striking steel should arouse strong suspicion of:

intra-ocular foreign body.

what pediatric thing has current jelly stool?

intussception. also sausage shaped thing.

Dizziness is most likely to have a serious etiology when it

is associated with diplopia and other neurologic symptoms= weakness or difficulty with speech= central cause of vertigo and do complete workup.

An 82-year-old male nursing-home resident is sent to the emergency department with lower abdominal pain and bloody diarrhea. He has a history of multi-infarct dementia, hypertension, and hyperlipidemia. On examination he is afebrile, and a nasogastric aspirate is negative for evidence of bleeding. Which one of the following is the most likely cause of this patient's bleeding?

ischemic colitis= abdominal pain + bloody diarrhea and CV risks. peptic ulcer disease unlikely with nasogastric aspirate --. and diverticular bleeding and angiodysplasia are painless. infectious colitis has fever.

Evidence supports the use of which one of the following to reduce the risk of contrast-induced nephropathy in this patient? (check one) A. Intravenous furosemide B. Ascorbic acid C. Calcium antagonists D. Isotonic bicarbonate infusion E. High osmolar contrast media

isotonic bicarbonate infusion.

Patients with diverticulosis change what about their diet?

just increase fiber. Nothing else.

Which one of the following medications would be most useful pain due to a fractured ankle for a patient that takes naltrexone ? (check one) A. Fentanyl B. Hydromorphone (Dilaudid) C. Ketorolac D. Meperidine (Demerol) E. Morphine

ketorolac= non-opiod= NSAID.

The drug most often recommended as first-line therapy for hypertension in pregnancy is

labetolol. metoprolol with fetal growth restriction have given rise to the recommendation to avoid its use in pregnancy. Immediate-release nifedipine is not recommended due to the risk of hypotension.

patient whose hoarseness does not resolve within 3 months how should you proceed?

laryngoscopy.

When a screening test identifies a cancer earlier, thereby increasing the time between diagnosis and death without prolonging life, this is called:

lead time bias.

appropriate management of frostbite?

leaving simple nontense areas of clear blistering intact to avoid risk of infection. Tense or hemorrhagic blisters may be carefully aspirated, but only under sterile conditions. Rapid rewarming of affected areas is best done in a whirlpool bath containing a mild antiseptic at 40°C-41°C. Freezing injuries are extremely painful and analgesics are indicated. Vigorous rubbing of affected tissue is contraindicated, as it may lead to further tissue loss.

About a month after returning from the Middle East, an American soldier develops a papule on his forearm that subsequently ulcerates to form a shallow annular lesion with a raised margin. The lesion shows no signs of healing 3 months after it first appeared. He has no systemic symptoms. The most likely diagnosis is:

leishmaniasis= indolent course.

When considering a diagnosis of pancreatitis, amylase levels (check one) A. can help determine the severity of the disease B. are less likely to be elevated in alcoholics C. are more sensitive and specific than serum lipase levels D. are less likely to be affected by nonpancreatic conditions such as renal insufficiency

less likely to be elevated in alcoholics. serum lipase is more specific and sensitive than amylase (amylase can be high for inflammation of small bowel).

Salmonella+ high-risk conditions that predispose to bacteremia, and those with severe diarrhea, fever, and systemic toxicity or positive blood cultures should be treated with

levofloxacin or ceftriaxone.

Which one of the following drugs would be the most appropriate empiric therapy for nursing home-acquired pneumonia in a patient with no other underlying disease?

levofloxacin= concern about gram - oropharyngeal colonization. -- empiric tx must cover strep pneumo, s. aureus, h. flu, and gram --. LEVOFLOXACIN is the best.

pruritic, reddish purple plaques on her wrist, ankle, bad and bucacal mucosa. patient has hepatitis C. what is it?

lichen planus.

56 year old DM and HTN presents with 6 months of generalized pruritus. scratchs frequently and skin is dry and scaly. multiple linear excoriations and thickened skin on his forearm, legs and neck. what is this? treatment?

lichen simplex chronicus, consisting of lichenified plaques and excoriations that result from excessive scratching. stopping the itch-scratch cycle. topical corticosteroids can be helpful under occlusive dressing.

Which is not first line for head lice? A. Lindane 1% B. Malathion 0.5% (Ovide) C. Permethrin 1% (Nix) D. Pyrethrins 0.33%/pipernyl butoxide 4% (RID)

lindane= efficacy has waned over the years. -has neurotoxicity. malathion has triple action. pyrethins has some resistance but permethrin is first line.

breast cancer screening has caused an increase in the dx of?

localized disease.

A 32-year-old female who is an avid runner presents with knee pain. You suspect patellofemoral pain syndrome. Which one of the following signs or symptoms would prompt an evaluation for an alternative diagnosis? (check one) A. Peripatellar pain while running B. Knee stiffness with sitting C. A popping sensation in the knee D. Locking of the joint E. A positive J sign (lateral tracking of the patella when moved from flexion to full extension)

locking of the joint.

A health-care worker has a negative tuberculin skin test (Mantoux method). A second test 10 days later is positive. This result indicates:

long standing latent infection.

Young kid with hypertension what should you be prompted to do?

look for secondary causes of htn. -blood and urine testing. -renal US.

Antidiarrheal agents such as ... are generally safe and effective in the management of diarrheal symptoms in IBS.

loperamide.

A 35-year-old white male presents to the emergency department with chest pain of 30 minutes duration. He describes the pain as feeling like pressure on his chest, and says it radiates into his left arm. It is accompanied by dyspnea, diaphoresis, anxiety, and palpitations. His past medical history is unremarkable and he has no family history of premature heart disease. He smokes 2 packs of cigarettes per day and admits to intranasal cocaine use 2 hours ago. Vital signs include a blood pressure of 180/110 mm Hg, a pulse rate of 110 beats/min, a respiratory rate of 24/min, and a temperature of 37.2°C (99.0°F). Other than the anxiety and diaphoresis, the general examination is unremarkable. An EKG shows sinus tachycardia with an early repolarization pattern. Aspirin and nitroglycerin have been administered, as well as oxygen via nasal cannula. Which one of the following would be most appropriate at this point? (check one) A. Nifedipine (Procardia) B. Enalaprilat intravenously C. Lorazepam (Ativan) intravenously D. Metoprolol (Toprol) intravenously E. Thrombolytic therapy

lorazepam= Treatment of cocaine-associated chest pain is similar to that of acute coronary syndrome, unstable angina, or acute myocardial infarction, but there are exceptions.

lumbar spinal stenosis can be seen with?

lumbar spine extension= decreases the xsectional area of the spinal canal= compressing the spinal cord further. --walking downhill. pain with internal hip rotation= hip arthritis in the groin. increased pain with uphill walking= vascular claudication.

non specific chronic back pain most likely is?

lumbar strain and sprain.

treatment of severe preeclampsia with intravenous magnesium, the occurrence of apnea and areflexia is most consistent with ...

magnesium toxicity.

The finkelstein test?

make fist with thumb in fist and move lateral. + if pain over the abdutor pollicus and extensor policus brevis.

if your dexa score improves from -2.6 to -2.1 with the use of calcium, vitamin D and alendronate then what should you do for the lady? --she also recently stopped smoking, BMI 21, lumbar vertebral fracture 3 y ago.

make no change. she is still osteopenic + RF for osteoporosis.

fetal tachycardia, defined as a baseline fetal heart rate >160 beats/min for at least 15 minutes is nonreassuring. causes include?

maternal fever, fetal hypoxia, hyperthyroidism, maternal or fetal anemia, medication effects of parasympatholytic or sympathomimetic drugs, chorioamnionitis, fetal tachyarrhythmia, and prematurity.

A 45-year-old female presents to your office with knee pain. She was playing volleyball yesterday when she collided with another player and was unable to continue playing because of pain in her knee. The knee was swollen this morning. She is able to walk but not without pain, and she also has pain when she attempts to bend her knee. On examination there is medial joint line tenderness and a positive Thessaly test. Which one of the following is the most likely cause of her knee pain?

medial meniscus tear. --most likely diagnosis in a patient >40 yo who was bearing weight when the injury occurred, was able to continue activity and has a positive thessaly test.

most common cause of acute interstitial nephritis?

medication induced.

Upper thigh numbness in an adolescent female is a classic symptom of

meralgia paresthetica, which is attributed to impingement of the lateral femoral cutaneous nerve in the groin, often associated with obesity or wearing clothing that is too tight in the waist or groin.

Information derived from which one of the following provides the best evidence when selecting a specific treatment plan for a patient?

meta analyses rcts. --The lowest quality of evidence comes from sources such as expert opinion, consensus guidelines, or usual practice recommendations.

Firstline agents for ovulation induction and treatment of infertility in patients with PCOS include

metformin and clomiphene alone or in combination with rosiglitazone. -metformin improves menstrual irregulatiries in patients with PCOS.

two medications approved for tx of T2D in children?

metformin and insulin.

two drugs that improve fertility?

metformin.

which diabetes drug is not associated with hypoglycemic episodes?

metformin: increase the sensitivity of the liver and peripheral tissues to insulin. This assists the patient with weight loss efforts and, unlike insulin secretagogues, has been proven to reduce mortality with long-term use.

hyperTG tx? Triglyceride levels of 150-199 mg/dL= mild. levels of 200-999 mg/dL=moderate. 1000-1999 mg/dL = severe. levels >2000 mg/dL= very severe.

mild to moderate range may be at risk for cardiovascular disease, but those who have severe or very severe hypertriglyceridemia have a significant risk of pancreatitis. For patients at risk for pancreatitis, fibrates are recommended as the initial treatment for pancreatitis. if moderate elevation= statin also good but do not start them along. Fibrate needs to be there.

A. Red meat B. Milk C. Seafood D. Nuts E. Beans of these which one is less likely to precipitate gout attack?

milk= decreases risk of another attack. things high in purines and proteins worsen.

Familial hypocalciuric hyeprca is what?

moderate hypercalcemia but relatively low urinary calcium excretion. PTH levels can be normal or only mildly elevated despite the hypercalcemia. This mild elevation can lead to an erroneous diagnosis of primary hyperparathyroidism. The conditions can be differentiated by a 24-hour urine collection for calcium; calcium levels will be high or normal in patients with hyperparathyroidism and low in patients with FHH.

Mechanical injury to the globe:

moderate to severe pain, normal or decreased vision, subconjunctival hemorrhage completely surrounding the cornea, and a pupil that is irregular or deviated toward the injury.

The most appropriate initial test to rule out adrenal insufficiency is:

morning serum cortisol. A single morning serum cortisol level >13µg/dL reliably excludes adrenal insufficiency. If lower then do ACTH stimulation test. insulin tolerance and meyrapone tests are no longer used. MRI sucks for this.

Which one of the following medications is most appropriate for treating moderate to severe shortness of breath in a hospice patient with lung cancer?

morphine.

Which one of the following antihypertensive drugs is most likely to cause ankle edema? (check one) A. Hydrochlorothiazide B. Amlodopine (Norvasc) C. Lisinopril (Prinivil, Zestril) D. Losartan (Cozar) E. Atenolol (Tenormin)

most common side effects of calcium channel blockers, such as amlodipine, are due to vasodilation. One result of this may be peripheral edema, but it can also cause dizziness, nausea, hypotension, cough, and pulmonary edema.

An elderly couple is having trouble paying for the considerable number of medications they require. They ask you about the safety of obtaining drugs from Canada. Which one of the following is true concerning Canadian drugs?

most of the drugs from canada come from the same manufacturer as in the US= - fda has approved of more than 90% of the drugs available from canada.

what fluoroquinolone does not work for UTI?

moxifloxacin= inadequate urinary concentration. can use: cipro. levo. norflo. lomeflox. gatiflox.

what are some symptoms that are more common with letrozole than with tamoxifen? tamoxifen is more likely to cause?

myalgias and noninflammatory arthralgias. endometrial cancer.

patient's wishes to not know about a diagnosis or prognosis should be respected. However, it is reasonable to ask the patient to what?

name a proxy with whom you may discuss the issues.

NSAID safest for patient w/ hx of MI?

naproxen.

what nsaid is not associated with increased risk of CV event?

naproxen. all nsaids have deleterious effects on kidneys though.

Which one of the following is the most likely cause of chronic unilateral nasal obstruction in an adult?

nasal septal deviation. Mucosal disease is usually bilateral and intermittent. Adenoidal hypertrophy is the most common tumor or growth to cause nasal obstruction, followed by nasal polyps, but both are less frequent than true anatomic causes of constant obstruction.

Recommendations for the treatment of hospitalized infants with bronchiolitis include

nasal suctioning via bulb or neosucker to clear the upper airways. deep suction is not recommended (beyond nasopharynx).

most common se of varenicline is?

nausea. drug + food reduces the risk.

Severe pain and skin changes outside the realm of cellulitis (bullae and deeper discoloration)= tell you that this is?

necrotizing fascitis.

most common form of child abuse?

neglect.

Currently circulating H1N1 is almost always susceptible to what medication? Resistant to what?

neuraminadase inhibitors (oseltamivir and zanamivir) and resistant to adamantanes (amantadine and rimantadine).

tx for raynauds?

nifedipine. dihydropyridine.s prazosin and terazosin are also good. cilostazol is good for intermittet claudication.

Renal artery stenosis that is in good control tx?

no testing other than monitoring renal function especially if ace inhibitor or arb is part of the regimen. -- screen with ultrasonography, CT angiography or MR cystography. Captopril renography is no longer recommended.

some patients with hypothyroidism will have appropriate dosage of TSH but continue to have fatigue, depressed mood and weight gain. if the TSH is appropriate range then what do you do?

no tx adjustment.

tx for asymptomatic multiple myeloma? how about symptomatic <65. >65. Not candidates for sc transplant? medications. metastatic bone pain/ spinal cord compression.

no tx. symptomatic patients under age 65 is autologous stem cell transplantation. 65+ who are healthy enough= transplantation would also be appropriate candidates. Patients who are not candidates for autologous stem cell transplantation generally receive melphalan and prednisolone with or without thalidomide. Radiotherapy can be used to relieve metastatic bone pain or spinal cord compression.

Midshaft clavicle fractures are usually treated: Higher risk with who?

non-operatively but higher risk of nonunion. Female gender, fracture communicatoin or displacement, clavicle shortening, advanced age, greater extent of initial trauma. --these fractures usually head well even if displaced or communicated because of periosteal regenerative potential.

fat woman comes in with increased ast. wtf is going on?

nonalcoholic fatty liver disease. get skinner.

non-severe salmonella tx?

none.

can bisphosphonates effect olfactory?

nope.

is breast feeding a contraindication for MMR vaccine?

nope.

What would you find with retinal detachment:

normal vision or peripheral or central vision loss; absence of pain; increasing floaters; and a normal conjunctiva, cornea, and pupil.

mild cognitive impairment: motor function is: Functional activity: objective memory: cognitive decline:

normal. normal. decline. worry.

The ... virus are common causes of waterborne epidemics of gastroenteritis, and have been shown to be responsible for outbreaks in nursing homes, on cruise ships, at summer camps, and in schools. Symptomatic treatment by itself is usually appropriate.

noroviruses.

preoperative testing in asymptomatic patients undergoing low-risk procedures such as cataract surgery what do you need to do?

nothing. no ekg or cbc.

mild benign prostatic hyperplasia what do you do?

observe and repeat evaluation in a year.

Femoral anteversion tx for 5yo?

observe. --if 8-10 surgery.

First step in the evaluation of nonmassive hemoptysis is to

obtain a chest radiograph. If this is normal and there is a high risk of malignancy (patient age 40 years or older with at least a 30-pack year smoking history), chest CT should be ordered. If a chest radiograph shows an infiltrate, treatment with antibiotics is warranted. If the chest radiograph is normal the patient is at low risk for malignancy, and if the history does not suggest lower respiratory infection and hemoptysis does not recur, observation can be considered.

if you have a painful distal fifth metacarpal with a superficial abrasion secondary to fight bite what do you have to do?

obtain radiograph and give prophylactic antibiotics. Deeper wounds should be explored by a surgeon, but superficial wounds should not be probed indiscriminately.

The physician counseling a 4-year-old child about the death of a loved one should keep in mind that children in this age group: (check one) A. often feel no sense of loss B. often believe they are somehow responsible for the death C. should not attend a funeral D. should usually be told the loved one is having a long sleep E. usually accept the finality of death with little question

often believe they are somehow responsible for the death of a loved one (2-6 years).

if patient has stress fracture when can they return to paly once again?

once they are pain free and have a normal physical exam even if it is less than 6 weeks (usually takes 6-10 weeks to heal). -Repeat radiograph rare.

for patient with anemia of chronic disease tx?

optimal management of underlying disorder.

antibiotic of choice for CAP for children 60 days - 5 years? 5-16 years?

oral amoxicillin. Strep pneumoniae. azithromycin= activity against M.pneumoniae and chlamydia.

moderate to severe acne tx?

oral antibiotics= minocycline.

decayed left lower molar that is tender when tapped lightly, and surrounding gingival inflammation and tenderness. There is no obvious mandibular swelling, but he does have a tender submandibular lymph node. dx? tx? what is penicillin allergy?

peridontitis of the tooth's root with cellulitis complicated with an apical abscess. --anaerobic oral bacteria. Penicillin VK, amoxicillin or amoxicillin/clavulanate is preferred for antibiotic treatment, but this patient is allergic to penicillin. Clindamycin is a good choice to cover the likely pathogens.

A 42-year-old African-American male recently traveled to the Caribbean for a scuba diving trip. Since his return he has noted brief intermittent episodes of vertigo not associated with nausea or vomiting. He is concerned, however, because these episodes occurred after sneezing or coughing and then a couple of times after straining while lifting something. He has had no hearing loss, and no vertigo with positional changes such as bending over or turning over in bed. The most likely cause of this patients vertigo is

perilymphatic fistula. between middle and inner ear caused by barotrauma from scuba diving and direct blows, weight bearing and straining. Vestibular neuronitis is a more sudden, unremitting syndrome. Menieres disease is manifested by episodes of vertigo, associated with hearing loss and often with nausea and vomiting. Benign paroxysmal positional vertigo is more likely in older individuals, and is associated with postural change. Multiple sclerosis requires symptoms in multiple areas and is not thought to be provoked by climate change.

thiazolidinediones sensitize

peripheral tissues to insulin.

patients with a medial meniscus tear but no osteoarthritis tx?

physical theraphy and exercise regimen. if there is osteoarthritis with meniscus tear then do meniscectomy.

herald patch is?

pityriasis rosea. erythematous, scaly, oval patch a few centimeters in diameter. This is usually followed within a few days by smaller patches on the trunk and sometimes the proximal extremities. Pityriasis rubra pilaris is a rare disease with five types. The classic adult type begins with a small red plaque on the face or upper body that gradually spreads to become a generalized eruption. The other conditions listed typically begin with multiple lesions.

ovoid salmon colored slightly raised herald patch on the back with mild prodrome. Multiple similar lesions trend along Langer's lines.

pityriasis rosea. Guttate psoriasis shares some features with pityriasis rosea in that it can appear suddenly and often follows a triggering incident such as a streptococcal infection, but absence of a herald patch and the smaller but thicker erythematous lesions differentiate psoriasis from pityriasis rosea. Tinea versicolor often involves the upper trunk and may appear as a lightly erythematous, scaling rash, but the onset is more gradual than in this case. Although this patient may be exposed to low levels of radiation in her job, radiation dermatitis requires doses such as those administered in cancer treatment protocols and would generally be limited to the field of exposure. Cutaneous T-cell lymphoma usually presents as a nonspecific dermatitis, most commonly in men over the age of 50. An infectious etiology for pityriasis rosea is strongly suspected, although none has been identified. There is some evidence that the agent may be human herpesvirus 6. The illness generally resolves within 2 months, leaving no residual signs other than postinflammatory hyperpigmentation.

associated with testosterone supplementation in men with hypogonadism?

polycythemia. patient receiving testosterone make sure to monitor Hct. Testosterone also causes an increase in lean body mass, and may increase bone density.

67-year-old male presents with a 10-day history of bilateral shoulder pain and stiffness accompanied by upper arm tenderness. On examination there is soreness about both shoulders and the patient has great difficulty raising his arms above his shoulders. There is no visual disturbance, and no tenderness over the temporal arteries. C-reactive protein is elevated and the erythrocyte sedimentation rate is 65 mm/hr (N 0-17). what does this patient have?

polymalgia rheumatica= increased prevalence with age. --tx: 15 mg of prednisone daily. NSAIDs not useful.

A 76-year-old female presents with a history of bilateral shoulder pain for the past month. She reports stiffness in the morning for about 1 hour and also reports difficulty getting up when seated in a chair. Acetaminophen is ineffective for her pain. Her erythrocyte sedimentation rate is 65 mm/hr (N 1-25). what is it?

polymyalgia rheumatica.

A 73-year-old male sees you for evaluation of a tremor. Based on the history and examination, you suspect Parkinson's disease. Which one of the following would be most helpful for confirming the diagnosis? (check one) A. CT of the brain B. MRI of the brain C. A positive response to levodopa D. Confirming that the tremor occurs with movement E. Confirming that the tremor had a symmetric onset

positive response to levodopa. --asymmetric tremor onset and distal resting tremor of 3-6 Hz.

Joint pain in the presence of fever with no apparent source indicates

possible infection, malignancy or rheumatic condition and requires further workup.

postpartum urinary retention (PUR). PUR is often defined as

post void bladder residual of at least 150 cc that is present 6 hours or more after delivery. --primiparous, prolonged first or second stage of labor, have instrumental vaginal deliveries or require C section for failure to progress.

Fell, landing on his outstretched hand with his elbow in slight flexion at impact. Pronation and supination of the forearm are painful on examination, as are attempts to flex the elbow. tenderness of the radial head without significant swelling. A radiograph of the elbow shows no fracture, but a positive fat pad sign is noted. Appropriate management would include:

posterior splint and repeat radiograph in 1-2 weeks. nondisplaced radial head fractures can be treated by the primary care physician and no referral is required. --sometimes joint effusion can be aspirated for pain relief and to increase mobility.

IBD can induce remission with what medication?

prednisone. Anti-TNF agents such as infliximab should be considered in patients with moderate to severe Crohn's disease who do not respond to initial corticosteroid or immunosuppressive therapy, but these are not recommended for initial treatment.

medicare pays for?

preventative measures. vaccines, mammography, paps.

A 61-year-old female is found to have a serum calcium level of 11.6 mg/dL (N 8.6-10.2) on routine laboratory screening. To confirm the hypercalcemia you order an ionized calcium level, which is 1.49 mmol/L (N 1.14-1.32). Additional testing reveals an intact parathyroid hormone level of 126 pg/mL (N 15-75) and a urine calcium excretion of 386 mg/24 hr (N 100-300). Cause of high Ca?

primary hyperparathryoidism; Intact parathyroid hormone (PTH) will be suppressed in cases of malignancy-associated hypercalcemia, except for extremely rare cases of parathyroid carcinoma.

most common cause of hypercalcemia= how can you diagnose it? reveal it?

primary hyperpth and malignancy. do a serum intact pth. next you can do vitamin D and urine calcium. calcitonin is not something that needs to be measured.

A 60-year-old white female with type 1 diabetes mellitus presents with early satiety, nausea, bloating, and postprandial fullness. Laboratory tests are normal, as are upper endoscopy and biliary ultrasonography. Which one of the following would help confirm the most likely diagnosis? and what is the dx? (check one) A. Pelvic ultrasonography B. An exercise stress test C. Psychiatric consultation D. Gastric emptying scintigraphy E. Colonoscopy

probably gastroparesis; autonomic neuropathy usually in T1D women. --CBC r/o infection. --BMP, endoscopy and biliary tract evauation. CONFIRM: gastric emptying scintigraphy.

what are some drugs that can cause lupus like symptoms?

procainamide. hydralazine. so stop the drug and then For those with severe symptoms, a short course of corticosteroids is indicated. Once the offending drug is discontinued, symptoms seldom last beyond 6 months.

A 36-year-old white male complains of episodic pain in the rectum over the past several years. The pain occurs every 3-6 weeks and is sharp, cramp-like, and severe. It lasts from 1 to 15 minutes. He has no other gastrointestinal complaints. A physical examination, including a digital rectal examination and anoscopy, is normal. The most likely diagnosis is:

proctalgia fugax. 13-19% of general population. The normal physical and anoscopy would also be prominent with sacral nerve neuralgia but it would be intermittent for years and last longer.

Galactorrhea associated with a prolactin level >200 μg/L usually indicates

prolactinoma = requires MRI of the pituitary with gadolinium enhancement.

if you have family hx of sudden death and recurrent syncope you are suspicious of? dx? tx?

prolonge QT syndrome. Do a resting EKG for QTc>460mseconds. in females and >440 in males. b-blocker, implantable cardioverter-defibrillator and no participation in competitive sports.

simple to reduce a lateral patellar dislocation, how?

proper technique is to have the patient sit or lie with the leg in a flexed position and then apply gentle medial pressure to the patella until the most lateral edge is over the femoral condyle. The leg should then be gently extended and the knee brought into full extension. This should cause the patella to slip back into place, and the knee should then be immobilized.

what is rifaximin used for?

prophalaxis for traveler's diarrhea, IBS and hepatic encephalopathy. nonabsorbale antibiotic.

specificity of a screening test is described as?

proportion of people without the condition who test negative.

Chronic lung disease and taking steroids at risk of what bug different from a healthy person for pneumonia?

psuedomonas.

An 85-year-old male is admitted to a nursing home due to weakness, debility, and limitation of activities of daily living (ADLs) after being hospitalized for acute community-acquired pneumonia. He previously lived with his wife independently and his goal is to return home when he is strong enough. He has a history of coronary artery disease, type 2 diabetes mellitus controlled with diet, hypertension, and chronic diastolic heart failure, but he has no symptoms related to these chronic problems. His appetite is poor and he has lost a significant amount of weight. His admission diet order from the hospital was a cardiac diet. Which one of the following would be the most appropriate diet for this patient?

regular diet= promote weight gain in nursing home residents with unintentional weight loss.

Single umbilical artery in a baby is associated with?

renal abnormalities.

bisphosphonate (zolondrate) excreted how?

renally.

initial test for hyperaldosteronism?

renin to aldosterone ratio. <20 effectively rules out hyperalodsteronism. >15 suggests aldosteronism but salt suppression test must be done to confirm. urinary K does not play a role in dx.

if you happen to find a 0.7 cm thyroid nodule and then a couple of weeks later no what do you do? tsh normal.

repeat US in 6-12 months= incidentaloma. If TSH is normal then nuclear scanning and further tests are not needed.

mild aortic stenosis f/u how?

repeat echocardiography every 3-5 years.

asymptomatic aortic stenosis management?

repeat echocardiography in 3-5 years.

A 75-year-old male develops a mild Clostridium difficile infection and is treated with 10 days of metronidazole (Flagyl), 500 mg orally 3 times daily. The diarrhea recurs 10 days after he completes the course of treatment. Which one of the following would be most appropriate?

repeat the course of metronidazole again. --mild recurrent disease repeat the course of the original agent. IV vancomycin not helpful need PO.

tx for uti should be based on?

reported symptoms.

Primary hyperaldosteronism is a relatively common cause of

resistant hypertension.

Magnesium sulfate infusions must be carefully monitored because ... is a potential lethal side effect.

respiratory depression. --reflexes usually lost first.

FVC that falls below the lower limit of normal (fifth percentile of the spirometry data) is consistent with?

restrictive pattern of lung disese. FEV1/FVC < LLN is consistent with Obstructive. Mixed if both are below the LLN. --the patient should now be referred for full pulmonary function test= DLCO.

scattered individual purple macules on the ankles and wrists.

rocky mountain spotted fever.

evidence regarding prevention of preeclampsia. The only medication with sufficient evidence to support its

routine use is aspirin at dosages of 60-80 mg daily.

rhinorrhea and pharyngitis, followed in 1-3 days by cough and wheezing. Auscultation reveals diffuse rhonchi, fine rales, and wheezes. The chest film is often normal. If the illness progresses, cough and wheezing increase, air hunger and intercostal retractions develop, and evidence of hyperexpansion of the chest is seen.

rsv.

most common bugs associated with cervical lymphadenitis?

s. aureus and GAS

+ faber test (flexion, abduction. external rotation) is indicative of?

sacroilitis.

fever, malaise, weight loss, cough, and dyspnea, bilateral hilar lymphadenopathy and discrete, noncaseating epithelial granulomas, Facial nerve palsy is seen in <5% of patients, and usually occurs late in the process., Hemoptysis does not generally occur until late in the course (related to aspergillus).

sarcoidosis.

overweight aa male are risk factors for? pain, painless limp or vague pain. no systemic signs. what is it? tx?

scfe= do not bear weight on it and get prompt surgical referral.

diabetes insipidus iscaused by a deficiency in?

secretion or renal action of arginine vasopressin (AVP)= antidiuretic hormone produced by posterior pituitary. -chief action is concentration of urine in the DCT. profound urinary volume, increased frequency of urination, and thirst. The urine is very dilute, with an osmolality <300 mOsm/L.

bence jones protein seen with what and not seen with what?

seen with 80% waldenstrom and barely any monoclonal gammopathy of undetermined significance.

tramadol should be avoided in a patient with a history of?

seizures; lowers the threshold.

what is lupiprostone?

selective C2 chloride channel activator that can be used for patients with chronic constipation.

grauloma annulare is what?

self limited= not contageous and not needed to be covered. --tx= inject/ topical corticosteroids. oral not so much.

proportion of persons with the condition who test positive

sensitivity.

Copper IUD over levonorgestrel releasing intrauterine device is good for?

severe cirrhosis or liver failure (you have an increased chance of hepatocellular adenoma). both apparently = chance of thromboembolic events.

Elevated BP + 4+ protein on urinalysis pregnant 36 week woman diagnosis?

severe preeclampsia. near term should be placed on magnesium sulfate to prevent seizures, and labor should be induced immediately. An urgent cesarean section is not necessary. Corticosteroids have not been shown to improve neonatal outcomes when given after 34 weeks gestation.

The jaundice of biliary cirrhosis is generally accompanied by

severe pruritis.

illiotibial band syndrome presents how?

stinging pain over the lateral femoral epicondyle.

surgical treatment of severe obesity be considered for any patient with a BMI >40 or those with a BMI >35 who have serious coexisting medical problems. Examples of such coexisting medical problems include ...

severe sleep apnea, Pickwickian syndrome, obesity-related cardiomyopathy, and severe diabetes mellitus.

Blood pressure classification in children is based on

sex, height, and age.

A 70-year-old white female presents with a pruritic rash on her sacrum that has occurred intermittently over the last 6 years. She reports that the area is always very tender just before the blister-like lesions erupt. She is otherwise in good health, and takes no medications. Her past medical history is unremarkable. You provide appropriate treatment for the condition. You should advise the patient to avoid which one of the following during future outbreaks?

sexual contact= genital herpes most common STI in the US.

what are the cut offs for the rotavirus vaccine?

should not be started after 15 weeks of age or continued past 8 months.

Asymptomatic gallstones should be txed with prophylactic cholecystectomy?

sickle cell. other hemoglobinopathies.

All patients with croup including mild disease should get?

single dose of dexamethasone. Humidification therapy has not proven to be beneficial. --nebulized epinephrine is an accepted tx in patients with moderate to severe croup.

what is intertrigo caused by?

skin inflammation by skin on skin friction caused by candida albicans. --facilitated by moisture trapped in deep skinfolds where air circulation is limited. --CANDIDA ALBICANS.

salmon pink rash can be?

sle.

Even the ...used to flush intravenous lines can be a source of thrombocytopenia.

small doses of heparin.

age related macular degeneration is the most common cause for blindness in elderly population. More frequently in light skinned individuals. 2 RF?

smoking and HTN.

A patient with ascites is suspected to have secondary hyperaldosteronism. Which one of the following would be typical levels of electrolytes in an aliquot specimen of urine?

sodium 2 mEq/L. potassium 40 mEq/L.

hydration with ... fluids reduces the risk of contrast-induced nephropathy in those undergoing cardiac catheterization

sodium bicarbonate-containing.

A 39-year-old female presents with lower abdominal/pelvic pain. On examination, with the patient in a supine position, you palpate the tender area of her lower abdomen. When you have her raise both legs off the table while you palpate the abdomen, her pain intensifies. Which one of the following is the most likely diagnosis?

something wrong with the abdominal wall itself: = hematoma within the abdominal wall musculature. tightening of the abdominal muscle= carnett's sign. if the cause was due to visceral problems then a taut abdomen would protect.

Risk factors for venous thromboembolism include which one of the following? (check one) A. Anemia B. The use of oral hypoglycemic agents C. Being underweight D. Young age E. Spinal cord injury

spinal cord injury because it induces immobility.

In addition to the usual laboratory evaluation, which one of the following imaging modalities would be most helpful for confirming a diagnosis of appendicitis?

spiral CT scan of the abdomen.

patient with resistant htn that is not responding with chlorthalidone, carvedilol, amlodipine and lisinopril what else can be added?

spirinolactone.

Treatment for mallet fracture?

splinting the DIP in extension for about 8 weeks. --maintain extension throughout entire duration because flexions can affect healing.

actinic keratoses can progress to?

squamous cell cancer.

usually has radiologic findings of focal, multiple infiltrates or cavitary lesions.

staph pneumonia.

switching to warfarin in a patient treated with dabigatran is to

start warfarin 3 days prior to stopping dabigatran. -inr will not be reflective until 2 days of dabigatran being stopped.

Most appropriate next step of 30 yo female that is having fertility issues?

step one: a luteal- phase progesterone level. -- Ovulation can be confirmed by a progesterone level >5 ng/mL on day 21 of the cycle. If <5 ng/mL, anovulation investigated with TSH, estradiol, FSH and prolactin.

Tx for SS?

stop the offending agent an then supportive care. -- diazepam or lorazepam can be used (do not use propranolol, haloperidol, flumazenil). If the patient does not respond to calming with benzodiazepines then use cyproheptadine.

cover/uncover test when the corneal light reflex is deviated from its normal position slightly nasal to mid-pupil. The misaligned eye then moves to fixate on a held object when the opposite eye is covered. The eye drifts back to its original position when the opposite eye is uncovered.

strabismus.

acl tears can be avoided how?

strength and training can prevent them. women> males. early onset osteoarthritis in affected knee for 50%. acl pops audibly when torn and does not need physical contact.

persistent groin pain, and limited hip flexion and internal rotation in runner/ military recruit. what is it?

stress fracture of the right femoral neck= radiographs can be normal initially.

A 40-year-old runner complains of gradually worsening pain on the lateral aspect of his foot. He runs on asphalt, and has increased his mileage from 2 miles/day to 5 miles/day over the last 2 weeks. Palpation causes pain over the lateral 5th metatarsal. The pain is also reproduced when he jumps on the affected leg. When you ask about his shoes he tells you he bought them several years ago. Which one of the following is the most likely diagnosis?

stress fracture= localized tenderness and swelling in superficial bones.. -pain reproducible by having patient jump on the affected leg.

most frequent cause of seizures in elderly?

stroke (Cerebrovascular disease).

injection for rotator cuff tendinitis is where?

subacromial space.

what can pseudogynecomastia be caused by?

subareolar fat secondary to his obesity.

pregnant woman in MVC you want to rule out spinal injury, what position should the patient be kept? if not in MVC then what position is preggers woman kept in if she is 20 + weeks pregnant?

supine with uterus manually deflected laterally. left lateral decubitus to avoid systolic bp dip secondary to compression of great vessels.

head and neck lymphadenopathy, ... nodes are the most likely to be malignant.

supraclavicular nodes.

rotator cuff tear tx?

surgery; is most beneficial if performed <6 weeks after injury.

A 15-year-old male presents to the emergency department at 10 p.m. with a 2-hour history of severe, acute scrotal pain associated with vomiting. On examination the right testicle is swollen. Ultrasonography is inconclusive. Which one of the following would be most appropriate at this point?

surgical consultation.

using OCP with spirinolactone has a what typeof effect for hirsutism?

synergistic.

Spontaneous pneumothrax demographic? tx?

tall thin individuals <40 yo. minimal chest pain. small pneumothoraces= <15% of the hemithroax with normal physical with occassional tachycardia. diagnosis confirmed with chest radiograph. tx= outpatient with repeat radiograph in 24-48 hrs. -most will have subcutaneous bullae on a CT scan.

Low back pain and pelvic pain are commonly encountered in pregnancy, a time when medication or physical modality use may prove undesirable or difficult. A properly prescribed exercise program is

target the low back, trunk, and abdominal muscles to increase core stability. -pilates. -back extension exercise. -abdominal crunches.

A 56-year-old African-American male has pain and tingling in the medial aspect of his ankle and the plantar aspect of his foot. He jogs 3 miles daily and has no history of any injury. The symptoms are aggravated by activity, and sometimes keep him awake at night. The only findings on examination are paresthesias when a reflex hammer is used to tap just inferior to the medial malleolus. This patient probably has

tarsal tunnel syndrome= entrapment of the posterior tibial nerve or its branches as the nerve courses behind the medial malleolus= neuritis. Causes of compression within the tarsal tunnel include varices of the posterior tibial vein, tenosynovitis of the flexor tendon, structural alteration of the tunnel secondary to trauma, and direct compression of the nerve. pronation of the foot causes pain and paresthesias in the medial aspect o the ankle and heel and sometimes plantar surface.

what has been beneficial for ibs?

tca (amitriptyline) or ssris.

acute trauma or from repetitive shoulder motion. Acute trauma may occur from a dislocation of the shoulder, falling on an outstretched arm, or direct blows to the shoulder. Generally, people with a tear of the labrum will have increased pain with overhead activity, popping or grinding, loss of strength, and trouble locating a specific point of pain.

tear of the labrum.

plan do check act describes what?

the cycle of continuous quality improvement.

The greatest demand for iron is during ...

the latter half of pregnancy.

alcoholic ketoacidosis tx?

thiamine, normal saline and glucose.

7 yo with groin pain and limp?

think more legg calve perethes.

what do the labs with myelodysplastic syndrome look like?

thrombocytopenia, neutropenia, macrocytic anemia.

Which one of the following skin infections should initially be treated with oral antifungal therapy? (check one) A. Tinea capitis B. Tinea corporis C. Tinea cruris D. Erythrasma E. Mycosis fungoides

tinea capitis because it needs to penetrate the hair shafts. ereythrasma and mycosis fungoides are not fungal caused.

A dry, scaling, dark red rash in the groin, with an active border and central clearing, is seen with

tinea cruris.

While playing basketball, a 29-year-old male falls on his outstretched hand with his wrist fully extended. He sees you the following day because of diffuse wrist pain and decreased range of motion. The point of maximal tenderness is on the dorsal aspect of the wrist between the extensor pollicis brevis and extensor pollicis longus tendons. There is no visible deformity. Radiographs show no fracture. Which one of the following is the most appropriate initial treatment of this patient?

use thumb spica splint; occult fracture of scaphoid bone of the wrist. - falling on outstretched hand with wrist extended. -tenderness in anatomical snuff box between the two muscles. -immobilize the wrist and thumb.

calcium levels during pagets?

usually normal despite increased resorption, formation and mineralization.

Regional analgesia in laboring patients increases the risk of ...

vacuum-or forceps-assisted delivery (relative risk = 1.42; 95% confidence interval, 1.28-1.57; 23 trials; n = 735).

ssnri that causes withdrawl symptoms if discontinued abruptly?

venlafaxine.

BNP is directly proportionate to what?

ventricular dysfunction.

cluster tx prophylaxis?

verapamil and lithium are the mainstays of treatment for chronic cluster headaches, but of the options listed, only verapamil is indicated for the prevention of cluster headaches, and it is actually the first-line prophylactic agent.

preferred method for diagnosing psychogenic nonepileptic seizures is:

video-electroencephalography.

Risk factors for delirium in hospitalized patients

vision impairment, hearing impairment, dehydration, immobility, cognitive impairment, and sleep deprivation.

A 65-year-old white female presents with weight loss and fatigue. On examination, she has lymphadenopathy, hepatomegaly, and mild splenomegaly. Her hemoglobin level is 9.0 g/dL (N 12.0-16.0), and a chemistry panel reveals a serum protein level of 9.0 g/dL (N 6.0-8.0). You order a chest radiograph, which shows clear lung fields and no evidence of lytic lesions in the thoracic spine. Serum protein electrophoresis reveals a monoclonal gamma-globulin spike, which on immunoelectrophoresis is found to be due to IgM kappa-protein. Urine for Bence-Jones protein is positive. A bone marrow biopsy from the iliac crest demonstrates hypercellularity, with a large number of lymphocytes, but normal-appearing plasma cells. Which one of the following is the most likely diagnosis?

waldenstrom's macroglobulinemia.

asymptomatic aortic stenosis, including those with severe disease tx?

watchful waiting. because surgical risk of aortic valve replacement outweighs the 1% annual risk of sudden death in asymptomatic patients with aortic stenosis. avoid Peripheral α-blockers (prazosin)= hypotension and syncope risk. Coronary angiography should be reserved for symptomatic patients who do not have evidence of severe aortic stenosis on echocardiography performed to evaluate their symptoms, or for preoperative evaluation prior to aortic valve replacement. Exercise stress testing is not safe with severe aortic stenosis because of the risk of death during the test.

Modified centor scoring system for kids is what?

whatever it is 4+ >50% chance of GABHS. Empiric tx for strep.

length time bias?

when a screening test finds a disproportionate number of cases of slowly progressive disease and misses the aggressive cases, thereby leading to an overestimate of the effectiveness of the screening.

dumping syndrome is what?

when food, especially sugar, moves from your stomach into your small bowel too quickly. early satiety, crampy abdominal pain, nausea, vomiting, and explosive diarrhea.diaphoresis, flushing, dizziness, palpitations, and an intense desire to lie down.

what is the most common time frame during which you get agranulocytosis with metimazole? Other SE of methimazole?

within 60 days of starting; 0.3% of the people that take it. Other SE of methimazole? serum sickness, cholestatic jaundice, alopecia, nephrotic syndrome, hypoglycemia, and loss of taste.

what is Jarisch-Herxheimer reaction

within the first few hours after treatment for syphilis. The condition peaks at 6-8 hours and disappears within 12-24 hours after therapy. The temperature elevation is usually low grade, and there is often associated myalgia, headache, and malaise. It is usually of no clinical significance and may be treated with salicylates in most cases. The pathogenesis of the reaction is unclear, but it may be due to liberation of antigens from the spirochetes.

screening for osteoperosis for men and women guidelines?

women >65 and men >70. For men and women age 50-69, the presence of factors associated with low bone density would merit screening. Risk factors include low body weight, previous fracture, a family history of osteoporosis with fracture, a history of falls, physical inactivity, low vitamin D or calcium intake, and the use of certain medications or the presence of certain medical conditions. COPD, HIV, severe liver disease, renal failure, systemic lupus erythematosus, and RA. type 1 diabetes mellitus, hyperparathyroidism, hyperthyroidism, Cushing's syndrome, and others. anticonvulsants, corticosteroids, and immunosuppressants. celiac disease, vitamin D deficiency, anorexia nervosa, gastric bypass, and increased alcohol or caffeine intake.

Sympathomimetic agents like pseudoephedrine and phenyephrine can be problamatic because they can:

worsen existing urinary obstruction, elevate BP, intraocular pressure, adversely interact with B blockers/ methy dopa/ tca/ hypoglycemic agents and MAOIs/ increase HR.

Migraine without aura is classified as category x in women younger than 35 and category y in women 35 or older. Nonmigrainous headaches are category z, as is chronic hepatitis C.

x= 2. y=3. 5=z.

can pneumonia cause SIADH?

yes.

Good alternatives to bisphosphonates? best evidence for prevention of both vertebral fractures and hip fractures?

zoledronic acid. teriparatide reduces vertebral fracture risk but not hip fracture risk. ditto for raloxifene and calcitonin salmon.


Conjuntos de estudio relacionados

Nursing Process Mastery Level 5

View Set

(N125/3) Ears, Nose, Mouth, Throat

View Set

Cell Surroundings - Biology Test

View Set

American English Vocabulary Profile A1

View Set

AP Statistics: Module 4, combined

View Set

MDSE 4010 Global Sourcing - Comprehensive Final

View Set